You are on page 1of 333

READING SUB-TEST

OET 2.0 READING TASK 1

Part A

TIME: 15 minutes

 Look at the four texts, A-D, in the separate Text Booklet.


 For each question, 1-20, look through the texts, A-D, to find the relevant information.
 Write your answers on the spaces provided in this Question Paper.
 Answer all the questions within the 15-minute time limit.
 Your answers should be correctly spelt.

Text A

Cannabis use disorder

Cannabis use disorder is a problematic pattern of cannabis use which leads to impaired control over cannabis
use and difficulty in ceasing use despite its harm. Drug abuse is a global phenomenon affecting almost every
country, with cigarettes, cannabis, and alcohol being the most commonly used and abused substances. Among
them cannabis is illegal. As compared to other psychoactive substances cannabis, most commonly known as
marijuana, is the most widely used drug worldwide. Even though there is no international consensus, young
adulthood is a period where the transition takes place from childhood to adulthood. Arnett et al. define this
group as “emerging adults” from 18 to 25 years, those who are neither children nor adults and who are in
between with their own identity and behavior. It is the age of instability, self-focus, feeling in between, and
possibilities. Young adults are the most vulnerable group of people than any age group to cannabis dependence
and related problems which produce more years lived with disabilities. In 2013, cannabis was used by 80.6%
of current illicit drug users in the United States.

facebook.com/HonorsGroup
READING SUB-TEST

Text B

Globally 2% cause-specific disability-adjusted life years (DALYs) for young people are attributed to illicit
drug including cannabis. The study conducted revealed that cannabis and other illegal drugs accounted for
approximately $8.2 billion of the nearly $40 billion cost of substance abuse in Canada in 2002. The probability
of cannabis addiction in heavy or daily user is enormous. The vulnerability increases in adolescents whose risk
of addiction is 16%, while adults have 5–10% risk of becoming addicted. Canadian community health survey
reported that the prevalence of cannabis dependence among adolescent and young adult in 2012 was 5%. A
three-year prospective study in the Netherlands on high-risk young adults reporting heavy use in 2013 found
that nearly 40% developed cannabis dependence.

Text C

Another study in Holland revealed that cannabis dependence was 42%. In 2013 the prevalence of cannabis
abuse or dependence was 7.4% among youth in the USA and the rate was about half (3.55) among adolescents.
National Survey on Drug Use and Health (NSDUH) revealed that cannabis was the illicit drug with the largest
number of persons with past-year dependence or abuse in 2013. Of the 6.9 million persons aged 12 or older
who were classified with illicit drug dependence or abuse in 2013, 4.2 million persons had cannabis
dependence or abuse. Another study conducted in the USA reported that 38.5% of daily cannabis users met
criteria for cannabis dependence. A longitudinal cohort study conducted in Australia in 2002 among young
adults shows a 7% prevalence of cannabis dependence according to DSM-IV criteria for cannabis dependence.
A community household-based survey with a cross-sectional design in Rwanda that aimed to determine the
prevalence of cannabis dependence among adolescent and young adults shows 2.54% prevalence of cannabis
dependence.

facebook.com/HonorsGroup
READING SUB-TEST

Text D

A recently published (2015) cohort study which considered cannabis abuse and dependence as cannabis use
disorder (CUD) showed the prevalence of cannabis use disorder throughout the life to be 19.1%, with an
average age of onset of 18.6 years. Cannabis availability, regular use of cannabis, peer pressure, and common
mental disorder were factors having a significant association with cannabis use disorder in different studies.
Gateway hypothesis developed by Kandel explained that the sequence of drug use occurring starts with legal
drug and proceeds to illegal drugs. Above all, Shashemene is a town in which Rastafarians view Ethiopia as a
promised land live. Cannabis use is a common practice among Rastafarians which brought a major challenge
to both youth and law enforcement in the town.

facebook.com/HonorsGroup
READING SUB-TEST

Questions 1-7

For each question, 1-7, decide which text (A, B, C or D) the information comes from. You may use any letter
more than once.

In which text can you find information about;

1 increase of the drug influence.

Answer :

2 Use of the drug in the USA.

Answer :

3 lifetime prevalence of cannabis use disorder.

Answer :

4 use of cannabis despite clinically significant distress or impairment.

Answer :

5 common determinants of drug addiction.

Answer :

6 addiction to drug common among young community.

Answer :

7 Prevalence of the drug abuse, drug usage.

Answer :

facebook.com/HonorsGroup
READING SUB-TEST

Questions 8-14

Answer each of the questions, 8-14, with a word or short phrase from one of the texts. Each answer may
include words, numbers or both.

8 From how many places data on cannabis addiction has been developed?

Answer :

9 As per the report, how many people in Canada are reported to be cannabis addict in the first quarter of the
2nd decade of 21st century?

Answer :

10 How many people are known to be addicted to cannabis as per the report of 2013?

Answer :

11 Who are likely to be addicted more commonly?

Answer :

12 Who are popular for making use of cannabis as its use is customary?

Answer :

13 As per the report, how many people in the USA are reported to be cannabis addict?

Answer :

14 What revealed a 7 percent of young adults were cannabis dependent in Australia during 2002?

Answer :

facebook.com/HonorsGroup
READING SUB-TEST

Questions 15-20

Complete each of the sentences, 15-20, with a word or short phrase from one of the texts. Each answer may
include words, numbers or both.

15 In the United States, in 2013, the drug was used by of people.

16 Commonly, adults may have about risk of becoming a drug addict

17 The studies in the landlocked East African country shows of cannabis dependence.

18 The sequence of the drug use may end with .

19 It is very much likely that is of higher level among drug addicts

20 explained that the sequence of drug use occurring starts with legal drug and proceeds to
illegal drugs.

facebook.com/HonorsGroup
READING SUB-TEST

Part B

In this part of the test, there are six short extracts relating to the work of health professionals. For questions 1-
6, choose the answer (A, B or C) which you think fits best according to the text.

Chronic urticaria

Chronic urticaria is a common condition characterized by recurrent hives lasting several weeks or months and
is usually idiopathic. Approximately half of the individuals with chronic urticaria will present with episodes of
angioedema that can be severe and debilitating. There is a 47-year-old Hispanic male who presented initially
for an evaluation of chronic hives following hospitalization due to hive-induced anaphylaxis. The individual
had a history significant for urticaria and angioedema beginning in his early 30s. Interestingly, both the
individual’s 41-year-old sister and 12-year-old daughter were also affected by chronic urticaria and severe
angioedema. Whole exome sequencing of the proband and several family members revealed a heterozygous
variant of uncertain significance in exon 2 of TNFAIP3, denoted as c.65G>A (p.R22Q), in all affected
members.

Arterial aneurysms

Arterial aneurysms are most commonly caused by atherosclerotic disease, especially in elderly patients aged
over 60. Other etiologies, such as connective tissue disorders, should be investigated in younger patients..
Fibrillin-1 microfibrils, through interactions with elastin and other proteins, provide structure to elastic and
nonelastic connective tissues. In addition to the architectural functions, Fibrillin-1 plays an important role in
regulating TGF-β complexes in the extracellular matrix. TGF-β signaling controls various processes at the
cellular level, such as cellular growth, differentiation, and apoptosis. When Fibrillin-1 is defective, it disrupts
the normal architecture of connective tissues.

facebook.com/HonorsGroup
READING SUB-TEST

Pulmonary embolism

Pulmonary embolism (PE) is regarded as an elusive diagnosis with a non-specific clinical presentation and has
a tendency to be both over- and underdiagnosed in clinical practice. In the United States of America, venous
thromboembolism (VTE) has been reported as the 3rd commonest cause of mortality. Most patients with PE
are clinically asymptomatic. In fact, PE has been shown to be present in 60–80% of individuals with
confirmed deep vein thrombosis (DVT), despite the absence of symptom in more than half of these patients.
Cardiac arrest following PE has an associated mortality of up to 70% within the first hour of presentation and
an overall mortality of up to 95%. Approximately 90% of episodes of cardiac arrests occur within 1-2 hours
after the onset of symptoms of PE.

Clinically, the cephalic vein is preferred for haemodialysis in patients with chronic renal failure (CRF), to
remove waste products from the blood. The cut-down of the cephalic vein in the deltopectoral groove is
preferred when superior vena caval infusion is necessary. However, cephalic veins exhibit a wide array of
developmental variations in terms of formation, course, and termination. During routine gross anatomy
dissection of the neck of the patient, a rare case of variation of the termination of the cephalic vein in both
right and left upper limbs have been observed. Knowledge of the variations of cephalic vein is important not
only for anatomists but also for surgeons and clinicians as the vein is frequently used for different surgical
procedures and for obtaining peripheral venous access as well.

facebook.com/HonorsGroup
READING SUB-TEST

% of dual therapy
% of total
Dual therapies Nb
treatment ()
()

Cycloferon + Adefovir 55 7.9% 43.3%

Cycloferon +
42 6.1% 33.0%
Lamivudine

Adefovir + Lamivudine 9 1.3% 7.1%

Cycloferon + Tenofovir 5 0.7% 4.0%

Tenofovir + Adefovir 4 0.6% 3.2%

Tenofovir + Lamivudine 4 0.6% 3.2%

Pegasys + Tenofovir 3 0.4% 2.4%

Pegasys + Adefovir 2 0.3% 1.6%

Entecavir + Tenofovir 1 0.1% 0.8%

facebook.com/HonorsGroup
READING SUB-TEST

Interferon + Cycloferon 1 0.1% 0.8%

Entecavir + Pegasys 1 0.1% 0.8%

Total 126 18.2%  

Marketed formulations of liposomes.

Product name Marketed by Uses

Removes wrinkles and dark spots and has


Capture Totale Dior
radiance effect with sunscreen

Dermosome Microfluidics Moisturizer

Decorte Moisture Liposome Face


Decorte Moisturizer
Cream

Decorte Moisture Liposome Eye Moisturizes, firms, and brightens the


Decorte
Cream delicate skin around the eyes

facebook.com/HonorsGroup
READING SUB-TEST

Natural Progesterone Liposomal Skin


NOW Solutions Maintenance of healthy feminine balance
Cream

Hydration, boosts collagen synthesis,


C-Vit Liposomal Serum Sesderma enhances the skin’s elasticity and
firmness, and brightens the complexion

Advanced Night Repair Protective


Estée Lauder Skin repair
Recovery Complex

facebook.com/HonorsGroup
READING SUB-TEST

Questions 1-6

1 Chronic urticaria;

A Is caused by an allergic reaction to a food or drug.

B Arises spontaneously and its cause is unknown.

C Is a genetic disorder that is very rare.

2 Arterial aneurysms;

A Are caused by narrowing of arteries.

B are abnormal dilations of the peripheral arteries.

C Occur due to change in structure and function of the Fibrillin-1.

3 What is correct?

A PE occurs when a clump of material, most often a blood clot, gets wedged into an artery in your lungs.

B Patients with DVT are susceptible to PE.

C PE symptoms, in most of the cases, do not get revealed easily.

4 What is the notice talking about?

A Cleaning of the blood of toxins.

B A major a superficial vein in the arm and disease associated with it.

C A case report.

5 What does the table indicate?

A Tenofovir goes well in coordination with Cycloferon.

facebook.com/HonorsGroup
READING SUB-TEST

B Cycloferon goes well in coordination with Tenofovir.

C Adefovir goes well in coordination with Cycloferon.

6 The products which are good moisturizers are marketed by;

A NOW Solutions, Microfluidics, Decorte.

B Microfluidics, Decorte, Dior.

C Microfluidics, Decorte.

6 The products which are good moisturizers are marketed by;

A NOW Solutions, Microfluidics, Decorte.

B Microfluidics, Decorte, Dior.

C Microfluidics, Decorte.

facebook.com/HonorsGroup
READING SUB-TEST

Part C

In this part of the test, there are two texts about different aspects of healthcare. For questions 7-22, choose the
answer (A, B, C or D) which you think fits best according to the text.

Text 1: What is Creutzfeldt-Jakob Disease?

Creutzfeldt-Jakob disease is a degenerative brain disorder that leads to dementia and, ultimately, death.
Symptoms of Creutzfeldt-Jakob disease (CJD) sometimes resemble those of other dementia-like brain
disorders, such as Alzheimer`s, but Creutzfeldt-Jakob disease usually progresses much more rapidly.
Creutzfeldt-Jakob disease captured public attention in the 1990s when some people in the United Kingdom
developed a form of the disease — variant CJD (vCJD) — after eating meat from diseased cattle. However,
"classic" Creutzfeldt-Jakob disease has not been linked to contaminated beef. Although serious, CJD is rare,
and vCJD is the least common form. Worldwide, there is an estimated one case of Creutzfeldt-Jakob disease
diagnosed per million people each year, most frequently in older adults, if not among children.

Creutzfeldt-Jakob disease is marked by rapid mental deterioration, usually within a few months. Initial signs
and symptoms of CJD typically include personality changes, anxiety, depression, memory loss, impaired
thinking, blurred vision, insomnia, difficulty speaking, difficulty swallowing and sudden, jerky movements. As
the disease progresses, mental symptoms worsen; most people eventually lapse into a coma. Heart failure,
respiratory failure, pneumonia or other infections are generally the cause of death. The disease usually runs its
course in about seven months, although a few people may live up to one or two years after diagnosis. In people
with the rarer CJD, psychiatric symptoms may be more prominent in the beginning, with dementia — the loss
of the ability to think, reason and remember - developing later in the course of the illness. In addition, this
variant affects people at a younger age than classic CJD does, and appears to have the slightly longer duration
of 12 to 14 months.

Creutzfeldt-Jakob disease and its variants belong to a broad group of human and animal diseases known as
transmissible spongiform encephalopathies (TSEs). The name derives from the spongy holes, visible under a
microscope, that develops in affected brain tissue. The cause of Creutzfeldt-Jakob disease and other TSEs
appears to be abnormal versions of a kind of protein called a prion. Normally, these proteins are harmless, but
when they`re misshapen they become infectious and can wreak havoc on normal biological processes. The risk
of CJD is low. The disease can`t be transmitted through coughing or sneezing, touching, or sexual contact. The

facebook.com/HonorsGroup
READING SUB-TEST

three ways it develops are: Sporadically: most people with classic CJD develop the disease for no apparent
reason. CJD that occurs without explanation is termed spontaneous CJD or sporadic CJD and accounts for the
majority of cases. By inheritance: in the United States, about 5 to 10 percent of people with CJD have a family
history of the disease or test positive for a genetic mutation associated with CJD. This type is referred to as
familial CJD. By contamination: a small number of people have developed CJD after being exposed to
infected human tissue during a medical procedure, such as a cornea or skin transplant. Also, because standard
sterilization methods do not destroy abnormal prions, a few people have developed CJD after undergoing brain
surgery with contaminated instruments. Cases of CJD related to medical procedures are referred to as
iatrogenic CJD. Variant CJD is linked primarily to eating beef infected with bovine spongiform
encephalopathy (BSE), the medical term for mad cow disease.

Most cases of Creutzfeldt-Jakob disease occur for unknown reasons, and no risk factors can be identified.
However, a few factors seem to be associated with different kinds of CJD: Age: sporadic CJD tends to develop
later in life, usually around the age of 60. The onset of familial CJD occurs only slightly earlier. On the other
hand, vCJD has affected people at a much younger age, usually in their late 20s. Genetics: people with familial
CJD have a genetic mutation that causes the disease. The disease is inherited in an autosomal dominant
fashion, which means you need to inherit only one copy of the mutated gene, from either parent, to develop the
disease. If you have the mutation, the chance of passing it on to your children is 50 percent. Genetic analysis in
people with iatrogenic and variant CJD suggests that inheriting identical copies of certain variants of the prion
gene may predispose a person to develop CJD if exposed to contaminated tissue. Exposure to contaminated
tissue: people who`ve received HGH derived from human pituitary glands or who`ve had dura mater grafts
may be at risk of iatrogenic CJD. The risk of contracting vCJD from eating contaminated beef is difficult to
determine. In general, if countries are effectively implementing public health measures, the risk is virtually
non-existent.

Only a brain biopsy or an examination of brain tissue after death (autopsy) can confirm the presence of
Creutzfeldt-Jakob disease. But doctors can often make an accurate diagnosis based on your medical and
personal history, a neurological exam, and certain diagnostic tests. The exam is likely to reveal such
characteristic symptoms as muscle twitching and spasms, abnormal reflexes, and coordination problems.
People with CJD may also have areas of blindness and changes in visual-spatial perception. In addition,
doctors commonly use the following tests to help detect CJD: Electroencephalogram (EEG): using electrodes
placed on your scalp, this test measures your brain`s electrical activity. People with CJD and vCJD show a

facebook.com/HonorsGroup
READING SUB-TEST

characteristically abnormal pattern. Magnetic resonance imaging (MRI): this technique uses radio waves and a
magnetic field to create cross-sectional images of your head and body. It`s especially useful in diagnosing
brain disorders because of its high-resolution images of the brain`s white matter and gray matter. Spinal fluid
tests: cerebrospinal fluid surrounds and cushions your brain and spinal cord. In a test called a lumbar puncture
— popularly known as a spinal tap - doctors use a needle to withdraw a small amount of this fluid for testing.
The presence of a particular protein in spinal fluid is often an indication of CJD or vCJD.

No effective treatment exists for Creutzfeldt-Jakob disease or any of its variants. A number of drugs have been
tested - including steroids, antibiotics and antiviral agents - and have not shown benefits. For that reason,
doctors focus on alleviating pain and other symptoms and on making people with these diseases as comfortable
as possible.

Text 2: Heat Rash

The skin`s job is to protect the inside of the body from the outside world. It acts as a preventive barrier against
intruders that cause infection, chemicals, or ultraviolet light from invading or damaging the body. It also plays
an important role in the body`s temperature control. One way that the body cools itself is by sweating, and
allowing that sweat or perspiration to evaporate. Sweat is manufactured in sweat glands that line the entire
body (except for a few small spots like fingernails, toenails, and the ear canal). Sweat glands are located in the
dermis or deep layer of the skin, and are regulated by the temperature control centers in the brain. Sweat from
the gland gets to the surface of the skin via a duct. A heat rash occurs when sweat ducts become clogged and
the sweat can`t get to the surface of the skin. Instead, it becomes trapped beneath the skin`s surface causing a
mild inflammation or rash. Heat rash is also called prickly heat or miliaria.

It is uncertain why some people get heat rashes and others don`t. The sweat gland ducts can become blocked if
excessive sweating occurs, and that sweat is not allowed to evaporate from a specific area. Some examples of
how blockage may occur include the following: creases in the skin, for example the neck, armpit, or groin
which have skin touching adjacent skin, making it difficult for air to circulate, therefore preventing sweat
evaporation; tight clothing that prevents sweat evaporation; bundling up in heavy clothing or sheets - this may
occur when a person tries to keep warm in the winter or when chilled because of an illness with fever. Heavy
creams or lotions can also clog sweat ducts. Babies have immature sweat glands that aren`t able to remove the
sweat they produce; they can develop heat rash if they are exposed to warm weather, are overdressed,

facebook.com/HonorsGroup
READING SUB-TEST

excessively bundled, or have a fever. Heat rash may occur as a side effect of some medications, for example,
isotretinoin (Accutane) or clonidine (Catapres).

The most common symptoms of heat rash are red bumps on the skin, and an itchy or prickly feeling to the
skin. These are due to inflammation of the superficial layers of the skin (the epidermis) and the prickly
sensation is similar to the feeling of mild sunburn. The symptoms of heat rash are the same in infants and
adults; however, since an infant can`t complain about the rash sensation, he or she may be fussy. Newborns,
infants, the elderly, and obese individuals with large areas with skin-on-skin contact areas (for example, a large
overlapping area of abdominal fat or panniculus) are at risk of developing a heat rash. They are all especially at
risk if they are immobile for long periods of time and parts of the skin aren`t exposed to circulating air, which
results in the inability of the sweat ducts to "breathe" (evaporative cooling). Heat rashes are more common in
places with hot, humid, climates because people sweat more. Intense exercise associated with lots of sweating
may cause a heat rash, especially if the clothing worn does not allow adequate air circulation.

The appearance of a heat rash depends upon where the excess sweat gets deposited in the skin. Tiny blisters
that look like small beads of sweat are seen if the sweat is blocked at the most superficial layers of the skin
where the sweat duct opens on the skin surface. Called miliaria crystallina, it has no symptoms other than these
"sweat bubbles." Classic heat rash or miliaria rubra occurs if the sweat causes inflammation in the deeper
layers of the epidermis. Like any other inflammation, the area becomes red and the blisters become slightly
larger. Because the sweat glands are blocked and don`t deliver sweat to the skin`s surface, the area involved is
dry and can be irritated, itchy, and sore. This rash is also called prickly heat. Less frequently, after repeated
episodes of prickly heat, the heat rash may inflame the deeper layer of the skin called the dermis, and cause
miliaria profunda. This rash is made up of larger, harder bumps that are more skin colored. The rash begins
almost immediately after exercise, and again no sweat can be found on the affected areas. Rarely, this type of
heat rash may potentially be dangerous if enough skin is involved, since the lack of sweating can lead to heat-
related illnesses like heat cramps, heat exhaustion, or heat stroke.

Heat rash or prickly heat is detected by physical examination. Knowing that the rash appears during sweating
or heat, appreciating the location on the body (in skin creases or where clothes fit tightly) and seeing what the
rash looks like is enough to make the diagnosis. As with many rashes, the health care professional may look at
the involved skin and, because of previous experience, immediately make the diagnosis. An effective recovery

facebook.com/HonorsGroup
READING SUB-TEST

process may depend more on treating heat rash with remedies such as over-the-counter creams and sprays.
Medical treatment for heat rash may involve antibiotics if the sweat glands become infected.

Text 1 : Questions 7-14

7 Which disease progresses faster?

A Alzheimers

B Jakob

C Both Alzheimer`s and Jakob

D Not given

8 Creutzfeldt-Jakob disease is commonly found among adults or elderly people;

A False

B True

C False, because it is found among children too.

D Not given

9 People affected with the disease may die after;

A One year

facebook.com/HonorsGroup
READING SUB-TEST

B Two years

C 12-14 months

D Not given

10 Paragraph 3 talks more about;

A Symptoms

B Occurrence

C Transmission

D Prevention

11 One of the most common risk factors includes;

A Exposure to contaminated tissue.

B Age

C Genetics

D B and C

12 “People who may have the human growth hormone derived from human pituitary glands may
be at risk of iatrogenic CJD”

A 100% true

B 100% false

C 50% true

D 50% false

facebook.com/HonorsGroup
READING SUB-TEST

13 “Confirmation of the Creutzfeldt-Jakob disease can be done only after the death of the
person.”

A True

B False

C True in some cases

D Not given

14 What is most helpful in detecting CJD?

A Electroencephalogram (EEG)

B Spinal fluid exams

C MRI

D All of the above

facebook.com/HonorsGroup
READING SUB-TEST

Text 2 : Questions 15-22

15 Heat rash develops when;

A Sweat ducts become clogged

B Sweat can`t come out onto the skin

C Skin stops developing sweat

D None

16 One of the most common reasons given for the blockage of the sweat glands is;

A Excessive sweat is not allowed to evaporate from the skin

B Creases in the skin which makes circulation difficult

C Tight clothing

D Heavy creams and lotions

17 In babies, heat rash often develops due to;

A Warm weather

B Overdressing

C Fever

D Not given

facebook.com/HonorsGroup
READING SUB-TEST

18 Paragraph 3 talks more about;

A Risk factors

B Who is at risk

C Symptoms

D Development of the disease

19 Heat rash is common in;

A Cold areas

B Areas with higher humidity

C Desert areas

D B and C

20 What is the central idea of paragraph 4?

A Heat rash symptoms.

B Mode of occurrence.

C What heat rash looks like.

DB&C

facebook.com/HonorsGroup
READING SUB-TEST

21 Heat rash may cause “miliaria profunda.”

A True in some cases

B False

C 100% true

D Not given

22 Medical treatment for heat rash is effective when the;

A Blockage is high.

B Blockage is mild.

C Blockage is low.

D Home remedies are ineffective.

facebook.com/HonorsGroup
READING SUB-TEST

ANSWERS

PART A

1. Correct Answer Is: B

2. Correct Answer Is: C


3. Correct Answer Is: C
4. Correct Answer Is: A
5. Correct Answer Is: D
6. Correct Answer Is: A
7. Correct Answer Is: C
8. Correct Answer Is: Six
9. Correct Answer Is: 5%
10. Correct Answer Is: 4.2 Million
11. Correct Answer Is: Young people
12. Correct Answer Is: Rastafarians
13. Correct Answer Is: 7.4%
14. Correct Answer Is: longitudinal cohort study
15. Correct Answer Is: 80.6%
16. Correct Answer Is: 5-“10%
17. Correct Answer Is: 2.54% prevalence
18. Correct Answer Is: illegal drugs
19. Correct Answer Is: cannabis addiction
20. Correct Answer Is: Gateway hypothesis

PART B & C

1. Correct Answer Is: B Arises spontaneously and its cause is unknown.


2. Correct Answer Is: A Are caused by narrowing of arteries.
3. Correct Answer Is: B Patients with DVT are susceptible to PE.
4. Correct Answer Is: A Cleaning of the blood of toxins.
5. Correct Answer Is: C Adefovir goes well in coordination with Cycloferon.
6. Correct Answer Is: C Microfluidics, Decorte.
7. Correct Answer Is: B Jakob
8. Correct Answer Is: B True
9. Correct Answer Is: D Not given
10. Correct Answer Is: C Transmission
11. Correct Answer Is: D B and C
12. Correct Answer Is: A 100% true
13. Correct Answer Is: C True in some cases
14. Correct Answer Is: D All of the above
15. Correct Answer Is: A Sweat ducts become clogged
16. Correct Answer Is: A Excessive sweat is not allowed to evaporate from the skin
17. Correct Answer Is: B Overdressing
18. Correct Answer Is: B Who is at risk
19. Correct Answer Is: B Areas with higher humidity
20. Correct Answer Is: C What heat rash looks like.
21. Correct Answer Is: A True in some cases
22. Correct Answer Is: A Blockage is high.

facebook.com/HonorsGroup
READING SUB-TEST

OET 2.0 READING TASK 2

Part A

TIME: 15 minutes

 Look at the four texts, A-D, in the separate Text Booklet.


 For each question, 1-20, look through the texts, A-D, to find the relevant information.
 Write your answers on the spaces provided in this Question Paper.
 Answer all the questions within the 15-minute time limit.
 Your answers should be correctly spelt.

Text A

DNA Sequencing Technologies

Past efforts at sequencing genes were painstaking, time consuming, and labor intensive, such as when Gilbert
and Maxam reported the sequence of 24 base pairs using a method known as wandering-spot analysis.
Thankfully, this situation began to change during the mid-1970s, when researcher Frederick Sanger developed
several faster, more efficient techniques to sequence DNA. Indeed, Sanger's work in this area was so
groundbreaking that it led to his receipt of the Nobel Prize in 1980.

Over the next several decades, technical advances automated, dramatically sped up, and further refined the
Sanger sequencing process. Also called the chain-termination or dideoxy method, Sanger sequencing involves
using a purified DNA polymerase enzyme to synthesize DNA chains of varying lengths. The key feature of the
Sanger method reaction mixture is the inclusion of dideoxynucleotide triphosphates (ddNTPs). These chain-
terminating dideoxynucleotides lack the 3' hydroxyl (OH) group needed to form the phosphodiester bond
between one nucleotide and the next during DNA strand elongation. Thus, when a dideoxynucleotide is
incorporated into the growing strand, it inhibits further strand extension. The result of many of these reactions
is a number of DNA fragments of varying length. These fragments are then separated by size using gel or
capillary tube electrophoresis. This procedure is sensitive enough to distinguish DNA fragments that differ in
size by only a single nucleotide.

facebook.com/HonorsGroup
READING SUB-TEST

Text B

In the modern era, whole world has experienced tremendous boost in the field of molecular diagnosis by use of
DNA sequencing technology. The human genome contains more than 3 billion base pairs that contain all the
information about our health and well-being. The first whole genome sequence of human was published
successfully before decades. It was very expensive and paid billion dollars to complete. The cost paid for, was
much more worthy as it was providing the first fundamental understanding of the structure and biology of
human genome and relation with diseases. Initially, Next Generation Sequencing technology was generated
huge amount data of human variant later it was proved that this NGS technology is much more effective in the
diagnosis human diseases by use of bioinformatic tools to select pathogenic variants. In present days,
sequencing costs have dramatically declined and therefore it is now routinely using for diagnosis of many rare
inherited diseases including hematology and blood disorder. Genome Wide sequence analysis is already
playing an important role in the hematology field. This new sequencing technology is going to solve the
challenges that researchers in the field of hematology are going forward.

Text C

Nowadays researchers are making disease-specific targeted NGS panel, which is helping more quickly and
precise diagnosis of specific disease in the field of hematology. Keeping in mind the growing research in the
area of molecular diagnosis, how genome-wide analysis has unlocked new avenues of research, diagnosis, and
therapy for benign hematologic disorders. Recent advances in molecular technologies, mainly next-generation
sequencing, inspire us to apply these technologies as a first-line approach for the identification of potential
mutations and to determine the novel causative genes in patients with blood disorders. Researcher have started
preparing targeted NGS panels for diagnosis of hematologic malignancies, Red cell congenital hemolytic
anemia for diagnosis of all rare cause of hemolytic anemia which covers around 70-80 genes associated with
hemoglobinopathies, which will cover gene related alpha (HBA1/2) and beta (HBB) globin gene locus
analysis, HBD sequence analysis, gene related to RBC membrane protein disorders, RBC enzymopathies
genes, congenital dyserythropoietic anemia (CDA) and the inherited bone marrow failure syndromes (IBMFS)

facebook.com/HonorsGroup
READING SUB-TEST

are a group of rare genetic blood disorders in which there is usually some form of aplastic anemia associated
with a family history of the similar disorder.

Text D

A panel of genes already identified by WES and association studies as responsible of CHA or modulators of
the clinical course of the disease is already analyzed by mass sequencing methodology (NGS) including
between 60-70 genes. Modifiers, related to sickle-cell formation, vascular adhesion to endothelium, tolerance
to hemolysis and acute severe events, and the analysis of the RBC glycolytic enzymes are included. The
variants obtained were studied by mapping in the GRCh38/hg38 version of the human reference genome. For
the prioritization of variants, filters related to pathogenicity and population frequency according to the SnpEff
v4.1 and Mutation Taster programs generally used for analysis. Some are previously used clinical databases of
hemolytic anemia are Human Gene Mutation Database version Professional, ClinVar, Red Cell Membrane
Database Mutations Database, Leiden Open Variation Database – PKLR. The allele frequency is generally
assessed in the population (1000G and ExAC) and in the local database. Finally, most of the researchers
generally used the ‘in silico’ predictions of pathogenicity and Sorting Intolerant from Tolerant (SIFT),
PolyPhen-2, Mutation Taster, and Mutation Assessor

facebook.com/HonorsGroup
READING SUB-TEST

Questions 1-7

For each question, 1-7, decide which text (A, B, C or D) the information comes from. You may use any letter
more than once.

In which text can you find information about;

1 Modern technologies of DNA sequencing

Answer :

2 Determination of the precise sequence of nucleotides in a sample of DNA.

Answer :

3 A source of inspiration

Answer :

4 Early DNA sequencing technologies

Answer :

5 Cost-effective solutions in sequencing

Answer :

6 Clinical databases

Answer :

7 Detection of the disease-causing genes

Answer :

facebook.com/HonorsGroup
READING SUB-TEST

Questions 8-14

Answer each of the questions, 8-14, with a word or short phrase from one of the texts. Each answer may
include words, numbers or both.

8 What is generally analyzed in a large population?

Answer :

9 What is the term which defines a method in which an electric field pulls molecules across a gel substrate or
hairlike capillary fiber?

Answer :

10 Which term may mean "performed on the computer or via computer simulation?

Answer :

11 What is known to make up the backbone of the strands of nucleic acid?

Answer :

12 What is characterized by failure of the bone marrow to produce blood?

Answer :

13 Which method of sequencing usually comprises use of polymerase enzyme for the purpose of building
different types of chains of varied lengths?

Answer :

14 Which sequencing technology is regarded to be more result-oriented?

Answer :

facebook.com/HonorsGroup
READING SUB-TEST

Questions 15-20

Complete each of the sentences, 15-20, with a word or short phrase from one of the texts. Each answer may
include words, numbers or both.

15 Red blood cell affect genes encoding red blood cell enzymes..

16 Today, researchers are busy developed targeted NGS panels which can effectively be used for the purpose

of diagnosis of .

17 The variants obtained were studied by mapping in the of the human reference genome.

18 The key feature of the Sanger method reaction mixture is the inclusion of .

19 analysis is effective and can solve challenges which researchers grappling with

20 NGS is considered the for detection of the disease.

facebook.com/HonorsGroup
READING SUB-TEST

Part B

In this part of the test, there are six short extracts relating to the work of health professionals. For questions 1-
6, choose the answer (A, B or C) which you think fits best according to the text.

Primary infection of muscle

Primary infection of muscle is usually regarded as a tropical disease, and is rare although becoming more
common in temperate climates. A review of the 230 cases of primary obturator myositis which have been
reported, shows that 82% were in children under 18 years of age. The median age was 9.5 years (3 to 46),
with a male:female ratio of approximately 3:2. The median duration of symptoms prior to presentation was
three days, and the most common symptoms were fever, hip or thigh pain, and inability to fully bear
weight. In 41% there was a history of recent trauma to the hip, such as a fall or strenuous exercise. Local
trauma is a recognised initiating factor for pyomyositis and is documented in between 21% and 66% of
cases.

Food and Drug Administration (FDA) Notice

In the year 2017, Food and Drug Administration (FDA) announced the first approval of a CAR-T cell
therapy for kids and young adults with B-cell ALL. This approval was much celebrated and brought new
hope for a more specific and efficient therapy for ALL. This result has come through many years of
research and is expected to improve the quality of treatment of patients. It is worth noting that in addition
to side effects, the high cost of treatment is still an obstacle and the side effects. Although there are
challenges to be overcome as in any innovative research, CAR-T cell therapy seems to be the most
promising therapeutic tool against cancer, including ALL, since chemotherapy introduction in the 1940s.

facebook.com/HonorsGroup
READING SUB-TEST

Huntington’s disease (HD)

Huntington’s disease (HD) is a genetic neurodegenerative disorder that results in chorea, balance and gait
impairments, changes in behavior and declines in cognition. Cognitive decline occurs early in the disease
course, occasionally preceding chorea and motor impairment, and continues to progress throughout the
disease process. Cognitive deficits include difficulty with executive function including trouble with
planning and organizing, problems with working, visual and verbal memory, and impaired concentration.
While gait dysfunction is typically thought to arise primarily from damage to the motor circuitry of the
basal ganglia, studies in elderly populations and other neurologic populations indicate that gait dysfunction
may also be related to changes in cognitive function.

Hypertension - Prevention or Treatment

Adequate changes in lifestyle are the cornerstone for the prevention and treatment of hypertension.
Although rapid medical initiation is necessary for the patients in a high level of risk, lifestyle changes are
fundamental for the therapy. According to the previous report, lowering effects for stable blood pressure
can be equivalent to monotherapy of medicine. Contrarily, the weak point would be the low level of
compliance or adherence associated with necessary time for adequate action. Adequate changes in lifestyle
would be effective for some group of subjects. For grade 1 hypertensive patients, it can prevent or delay
medical therapy. Moreover, for hypertensive patients continuing on medical therapy, it can contribute to
BP reduction of blood pressure and allow reduction of the number and doses of antihypertensive agents.
Appropriate changes in lifestyle would decrease other cardiovascular risk factors and improve several
clinical conditions.

facebook.com/HonorsGroup
READING SUB-TEST

Comparison of 24-h blood pressure monitoring between conventional and custom-made pillows:

Conventional pillow Custom-made pillow P value

24-h SBP, mmHg 111 ± 8 111 ± 7 ns

24-h DBP, mmHg 70 ± 5 69 ± 6 ns

Daytime SBP, mmHg 115 ± 9 117 ± 7 ns

Daytime DBP, mmHg 74 ± 6 73 ± 6 ns

Nighttime SBP, mmHg 101 ± 6 100 ± 7 ns

Nighttime DBP, mmHg 59 ± 5 58 ± 7 ns

Nocturnal BP fall, mmHg 11 ± 5 14 ± 4 <0.05

Events

 3-h chest pain


 Respiratory failure and the need for advanced airway support
 Electrocardiogram with broad R waves, 4 mm ST-segment
downsloping in right precordial leads, right bundle branch block
(RBBB), and ST-segment elevation in posterior leads
Initial presentation at  Coronary angiography with total thrombotic occlusion in the
the emergency room proximal segment of the circumflex artery
 Bare metal stent was placed
 The patient developed cardiogenic shock and intra-aortic balloon
pump was placed; norepinephrine, vasopressin, and dobutamine
were administered with clinical improvement

facebook.com/HonorsGroup
READING SUB-TEST

 Electrocardiogram with disturbance of repolarization only


attributable to RBBB
 The patient developed acute renal failure and haemodialysis was
48 h post-arrival initiated

 During a haemodialysis session, he developed sustained ventricular


tachycardia with degeneration in asystole
After 5 days.  CPR was initiated, there wasn’t a return of spontaneous circulation
 Patient decease

Questions 1-6

1 What does the report indicate?

A Jugular venous engorgement and diffuse pulmonary rales were not found.

B The electrocardiogram (ECG) showed broad R waves.

C The patient was dyspnoeic with SaO2 74%.

2 What does the table indicate?

A Major significant differences were observed in 24-h, daytime and nighttime SBP or DBP when using
the conventional or custom-made pillow.

B No significant differences were observed in 24-h, daytime and nighttime SBP or DBP when using the
conventional or custom-made pillow.

C There are significant changes in Systolic Blood Pressure and Diastolic Blood Pressure.

facebook.com/HonorsGroup
READING SUB-TEST

3 What is correct about Hypertension?

A For subjects with normal or subnormal hypertension, it can prevent or delay hypertension.

B BP can be well tackled with the use of BP specific medicine.

C Right changes in lifestyle can curtail down BP values more effectively than various other medical
therapies.

4 According to the notice given, what is correct?

A Treatment cost has not gone down yet.

B CAR-T is considered more effective.

C After chemotherapy, CAR-T is more reliable.

5 Huntington`s disease;

A may result in the death of brain cells.

B may result in loss of intelligence.

C may result in involuntary movements.

6 What does the report indicate?

A There has been a recent increase in the incidence of myositis worldwide.

B Infection of the skeletal muscles.

C Most common in tropical areas but can also occur in temperate zones.

facebook.com/HonorsGroup
READING SUB-TEST

Part C

In this part of the test, there are two texts about different aspects of healthcare. For questions 7-22, choose the
answer (A, B, C or D) which you think fits best according to the text.

Text 1: Renal Artery Stenosis

Renal artery stenosis (narrowing) is a decrease in the diameter of the renal arteries. The resulting restriction of
blood flow to the kidneys may lead to impaired kidney function (renal failure) and high blood pressure
(hypertension), referred to as renovascular hypertension, or RVHT ("reno" for kidney and "vascular" for blood
vessel). Renovascular hypertension is as likely to occur with bilateral stenosis (when arteries to both kidneys
are narrowed) as with unilateral stenosis (when the artery to one kidney is narrowed). The decreased blood
flow to the kidneys impairs renal function. Renal artery stenosis may cause renal failure in some patients.
There is no predictable relationship between renal failure and renal artery stenosis. Some patients have very
severe bilateral stenosis and normal renal function. Most cases of renal failure are related to diabetes,
hypertension, glomerular sclerosis, contrast nephropathy, drug toxicity and other causes.

The majority of renal artery stenosis is caused by atherosclerosis (hardening and narrowing of blood vessel
wall from the inside) similar to the process that occurs in blood vessels in the heart and other parts of the body.
Risk factors for atherosclerosis include high cholesterol levels, high blood pressure, age, cigarette smoking,
diabetes etc. Less common causes of renal artery stenosis are fibromuscular dysplasia of the vessels
(narrowing of the vessel due to internal thickening of the blood vessel wall), arteritis (inflammation of the
blood vessel), or dissection (tearing and division of the blood vessel wall).

Narrowing of the kidney arteries is more common in individuals 50 years of age and older. It is estimated that
some degree of narrowing (greater than 50%) is found in about 18% of adults between 65-75 years of age and
42% of those older than 75 years of age. This may be due to the fact that atherosclerosis is more common in
this age group. In younger patients, the narrowing of the renal artery is usually due to the thickening of the
artery (fibromuscular dysplasia) and it is more common in women than in men. It is estimated that renal artery
stenosis accounts for approximately 1% of mild to moderate cases of high blood pressure. It may be

facebook.com/HonorsGroup
READING SUB-TEST

responsible for more than 10% of cases of severely elevated or difficult to treat high blood pressure
(hypertension).

In general, renal artery stenosis is not associated with any obvious or specific symptoms. Suspicious signs for
renal artery stenosis include high blood pressure that responds poorly to treatment; severe high blood pressure
that develops prior to age 30 or greater than age 50; an incidental finding (discovered through routine tests or
tests performed for another condition) of one small kidney compared to a normal sized one on the other side.
Typically, unilateral (one-sided) renal artery stenosis may be related to high blood pressure whereas bilateral
(two-sided) renal artery stenosis is more often related to diminished kidney function.

Several tests exist to detect any evidence of renal artery stenosis, which can be divided into imaging tests and
functional tests. The imaging tests provide a picture of the blood vessel and its anatomy and reveal the degree
of narrowing. The functional tests provide information about whether the narrowing is significant enough to
cause the high blood pressure or kidney dysfunction. Each of these tests has advantages and disadvantages.

In bilateral (both-sided) and unilateral (one-sided) renal artery stenosis associated with high blood pressure,
controlling the blood pressure with usual blood pressure medications is the first and the safest treatment. ACE
inhibitors or ARB medications with or without a diuretic (water pill) may be tried first. In some patients, this
approach may be associated with worsening of their kidney function. Therefore, kidney function needs to be
followed closely and if worsening of kidney function is evident, these medications may need to be stopped. It
is worth noting that if renal artery stenosis is found incidentally when performing a test for another disease and
there is no evidence of kidney dysfunction or high blood pressure then no treatment may be necessary.
Sometimes even significant stenosis may not be associated with high blood pressure or kidney dysfunction. In
these situations, periodic monitoring of blood pressure and kidney function may be advised.

Text 2: Hematochezia

Rectal bleeding (hematochezia) is used to describe the presence of blood with a bowel movement. That blood,
whether it fills the toilet bowl, or is a streak on the toilet paper when wiping, or just a few drops in the toilet
bowl, is not a normal finding and should not be ignored. The source of bleeding can be anywhere in the
digestive tract, from the nose and mouth to the rectum and anus. The color can range from bright red to

facebook.com/HonorsGroup
READING SUB-TEST

maroon to black or any shade in between, depending on how much the blood has been exposed to the digestive
juices. Anytime there is blood within the gastrointestinal system, it will eventually be excreted in stool (feces,
bowel movement, BM). The color of the stool will depend upon the amount of blood, the source of the
bleeding and how quickly the stool moves through the digestive tract. Sometimes, the bleeding is too little to
be seen by the naked eye but can be tested for by a healthcare professional.

Depending upon where and why the bleeding has taken place in the digestive tract, the stool consistency and
color may vary greatly: the stool color may be bright red, maroon, dark red or black. The bleeding might be
hidden, unseen to the naked eye, but able to be detected by a fecal occult blood test. There may be blood just in
the bowel movement or there may be associated feces. If the feces are formed, the blood may be mixed in with
the stool or it may just coat the surface. The stool may be well-formed or it may lose and diarrhea like. It may
be normal in shape in size or it may become pencil thin. There may be associated with abdominal pain or the
bleeding may be painless.

Hemorrhoids are the most common cause of blood in the stool. Blood vessels located in the walls of the rectum
can swell, become inflamed and bleed. Hemorrhoids can be caused by straining at stool, diarrhea, pregnancy,
obesity and prolonged sitting on the commode. All these factors increase the pressure within the hemorrhoidal
vessels, causing them to swell. The bleeding is often associated with anal burning or itching. Bleeding can also
occur because of an anal fissure, or a split in the skin of the anus. Hard constipated stool may cause the skin to
split; other causes include pregnancy and anal intercourse. Anal fissures are also associated with other diseases
including inflammatory bowel disease (Crohn`s disease, ulcerative colitis), cancer and infections. Anal fissures
tend to be very painful, even when sitting. The blood in the stool can also be due to swallowed blood from a
nosebleed, dental work, or other mouth injuries that cause bleeding.

Rectal bleeding is often diagnosed by history. The health care professional may ask questions about the
circumstances surrounding the rectal bleeding including the color, the amount of bleeding, any associated
symptoms and past medical history. A variety of medications and food can mimic blood in the stool. Iron
supplements and bismuth (Pepto-Bismol, Kaopectate) can turn stool black, as can beets and licorice. Red food
coloring and beets can turn stool into a reddish hue. Patients who take blood thinners (anticoagulation
medications) are more prone to rectal bleeding. Examples of blood thinners include warfarin (Coumadin),
enoxaparin (Lovenox), aspirin and other antiplatelet drugs including clopidogrel (Plavix), prasugrel and
rivaroxaban (Xarelto).

facebook.com/HonorsGroup
READING SUB-TEST

Physical examination is important to assess the patient`s stability. Vital signs are important and may include
orthostatic vital signs, where the blood pressure and pulse rate are taken both lying and standing. In a patient
with reduced blood volume, the blood pressure may fall, the pulse rate may rise, and the patient may become
lightheaded and weak when standing. Palpation of the abdomen is performed to look for tender areas, masses
or enlarged organs, especially the liver and spleen. Rectal examination is performed by inserting a finger into
the rectum, with the purpose of feeling for a mass or other abnormality. The stool color and consistency may
be examined when the finger is withdrawn. The anus also may be examined. Blood tests may be considered if
there is concern about the amount of bleeding or other associated diseases. A complete blood count (CBC)
measures the number of red blood cells, white blood cells and platelets. Blood clotting tests include PT
(protime), INR (international normalized ratio) and PTT (partial thromboplastin time). Depending upon the
situation, other tests may be ordered to measure electrolytes, and kidney and liver functions.

Most diseases which cause rectal bleeding are likely preventable, but it is not often possible. Hemorrhoids can
be avoided with proper diet and hydration to prevent constipation and straining to pass stool, but normal
pregnancy increases the risk of hemorrhoid formation as does the patient with an acute diarrheal illness.
Avoiding constipation also decreases the risk of diverticulosis, outpouchings in the lining of the colon, and the
risk of a diverticular bleed but this may be a consequence of a Western diet. Alcohol abuse increases the risk of
rectal bleeding in a variety of ways, from directly irritating the lining of the GI tract, to decreasing clotting
capabilities of blood.

facebook.com/HonorsGroup
READING SUB-TEST

Text 1: Questions 7-14

7 Renovascular hypertension is likely to occur with;

A Bilateral stenosis.

B Unilateral stenosis.

C Both bilateral and unilateral stenosis.

D When arteries to one or both kidneys are narrowed.

8 Which one of these statements is true, according to paragraph 1?

A The increase or decrease in blood flow leads to improper functioning of the kidneys

B Renal failure and renal artery stenosis are closely connected to each other

C A patient may have normal renal function even if there is higher bilateral stenosis

D In some cases, bilateral stenosis may affect renal functions too.

9 Renal Artery Stenosis is caused by;

A Hardening of the blood vessel wall from inside.

B Hardening of the blood vessels similar to that of the blood vessels in the heart.

C Narrowing and hardening of the walls like that of blood vessels in the heart.

D Hardening and narrowing of the blood vessels from inside.

facebook.com/HonorsGroup
READING SUB-TEST

10 Thickening of the arteries is more common among;

A Men

B Women

C Children

D All of the above

11 According to paragraph 4, which one of the following statements is true?

A There are no specific symptoms of the renal artery stenosis.

B Higher BP which develops before the age of 30 or after the age of 50 can become the cause of the renal
artery stenosis.

C Untreatable high BP can be the cause of the renal artery stenosis.

D Differences in sizes of the kidneys can be a major cause of the renal artery stenosis.

12 Functional tests provide;

A A clear picture of the blood vessels and functions.

B A clear idea of whether narrowing is significant to cause high BP or kidney dysfunction.

C Anatomy of blood vessels.

D A clear idea of the thickening of the blood vessels.

facebook.com/HonorsGroup
READING SUB-TEST

13 The best possible treatment for renal artery stenosis is;

A Controlling the blood pressure.

B Using ARB medications for the quality functioning of the kidneys.

C Improving the function of the kidney through proper medications.

D None

14 Renal artery stenosis is closely associated with;

A Kidney dysfunction.

B High blood pressure.

C Low blood pressure.

DA&B

Text 2: Questions 15-22

15 Rectal bleeding describes;

A Blood in the bowels.

B Blood in the digestive tract.

C Blood in the stools.

facebook.com/HonorsGroup
READING SUB-TEST

D Blood in the rectum.

16 Paragraph 2 talks more about;

A Which symptoms are associated with rectal bleeding.

B Causes of the rectal bleeding.

C Variations in the color of the stools.

D None

17 “Blood in the stool can originate anywhere in the gastrointestinal tract.”

A False

B True

C Not given

D Sometimes true and sometimes false

18 Hemorrhoids can be well defined by which one of the following?

A Swelling of the rectal walls.

B Inflammation and bleeding of the rectal walls.

C Straining

D All of the above

19 Causes of anal fissure may include;

A Formation of the constipated stool

B Inflammatory bowel disease

facebook.com/HonorsGroup
READING SUB-TEST

C Cancer and infections

D All of the above

20 According to paragraph 4, which of the following statements is true?

A Blood thinners can cause rectal bleeding.

B Blood thinners may sometimes cause rectal bleeding.

C It is not fully established that rectal bleeding is the direct result of the use of blood thinners.

D None

21 Which one of the following statements is not included in paragraph 5?

A Physical examiners will always look for orthostatic vital signs.

B A patient may have low blood pressure with a higher pulse rate.

C Palpitations of the abdomen is performed to look for tender areas.

D Rectal bleeding is identified through taking a stool sample.

22 Which one of these is the preferred blood clotting test?

A PT

B INR

C PTT

D Not given

facebook.com/HonorsGroup
READING SUB-TEST

ANSWERS

PART A

1. Correct Answer Is: B


2. Correct Answer Is: A
3. Correct Answer Is: C
4. Correct Answer Is: A
5. Correct Answer Is: B
6. Correct Answer Is: D
7. Correct Answer Is: C
8. Correct Answer Is: allele frequency
9. Correct Answer Is: Electrophoresis
10. Correct Answer Is: In silico
11. Correct Answer Is: Phosphodiester bonds
12. Correct Answer Is: Aplastic anaemia
13. Correct Answer Is: Sanger sequencing
14. Correct Answer Is: NGS
15. Correct Answer Is: enzymopathies
16. Correct Answer Is: hematologic malignancies
17. Correct Answer Is: GRCh38/hg38 version
18. Correct Answer Is: dideoxynucleotide triphosphates
19. Correct Answer Is: Genome Wide sequence
20. Correct Answer Is: first-line approach

PART B & C

1. Correct Answer Is: B The electrocardiogram (ECG) showed broad R waves.


2. Correct Answer Is: A Major significant differences were observed in 24-h, daytime and nighttime SBP or
DBP when using the conventional or custom-made pillow.
3. Correct Answer Is: A For subjects with normal or subnormal hypertension, it can prevent or delay
hypertension.
4. Correct Answer Is: B CAR-T is considered more effective.
5. Correct Answer Is: C may result in involuntary movements.
6. Correct Answer Is: C Most common in tropical areas but can also occur in temperate zones.
7. Correct Answer Is: A Bilateral stenosis.
8. Correct Answer Is: C A patient may have normal renal function even if there is higher bilateral stenosis
9. Correct Answer Is: D Hardening and narrowing of the blood vessels from inside.
10. Correct Answer Is: B Women
11. Correct Answer Is: C Untreatable high BP can be the cause of the renal artery stenosis.
12. Correct Answer Is: B A clear idea of whether narrowing is significant to cause high BP or kidney
dysfunction.
13. Correct Answer Is: A Controlling the blood pressure.
14. Correct Answer Is: D A & B
15. Correct Answer Is: C Blood in the stools.
16. Correct Answer Is: A Which symptoms are associated with rectal bleeding.
17. Correct Answer Is: B True
18. Correct Answer Is: D All of the above
19. Correct Answer Is: D All of the above
20. Correct Answer Is: A Blood thinners can cause rectal bleeding.

facebook.com/HonorsGroup
READING SUB-TEST

21. Correct Answer Is: D Rectal bleeding is identified through taking a stool sample.
22. Correct Answer Is: D Not given

OET 2.0 READING TASK 3

Part A

TIME: 15 minutes

 Look at the four texts, A-D, in the separate Text Booklet.


 For each question, 1-20, look through the texts, A-D, to find the relevant information.
 Write your answers on the spaces provided in this Question Paper.
 Answer all the questions within the 15-minute time limit.
 Your answers should be correctly spelt.

Text A

As demonstrated by elegant analyses of cancer in various patients, the continued deletion of cancer cells
expressing T cell targets may enable cancers to evolve to avoid the attack. Despite these findings, recent
results from human cancer have demonstrated that overcoming negative regulators to T cell responses in
lymphoid organs and in the tumor bed is likely to explain the failure of immune protection in many patients.
Factors in the tumor microenvironment can act to modulate the existing activated antitumor T cell immune
response, acting as an immune rheostat or “immunostat.” This class of molecules, including PD-L1:PD-1
emphasizes that the immune response in cancer reflects a series of carefully regulated events that may be
optimally addressed not singly but as a group. The most challenging part is now is to use this new
understanding to develop new drugs and implement clinical strategies.

Text B

For an anticancer immune response to lead to the effective killing of cancer cells, a series of stepwise events
must be initiated and allowed to proceed and expand iteratively. We refer to these steps as the Cancer-
Immunity Cycle. In the first step, neoantigens created by oncogenesis are released and captured by dendritic

facebook.com/HonorsGroup
READING SUB-TEST

cells (DCs) for processing. In order for this step to yield an anticancer T cell response, it must be accompanied
by signals that specify immunity lest peripheral tolerance to the tumor antigens is induced. Such immunogenic
signals might include proinflammatory cytokines and factors released by dying tumor cells or by the gut
microbiota. Next, DCs present the captured antigens on MHCI and MHCII molecules to T cells, resulting in
the priming and activation of effector T cell responses against the cancer-specific antigens that are viewed as
foreign or against which central tolerance has been incomplete. The nature of the immune response is
determined at this stage, with a critical balance representing the ratio of T effector cells versus T regulatory
cells being key to the final outcome.

Text C

Finally, the activated effector T cells traffic to and infiltrate the tumor bed, specifically recognize and bind to
cancer cells through the interaction between its T cell receptor (TCR) and its cognate antigen bound to MHCI,
and kill their target cancer cell. The killing of the cancer cell releases additional tumor-associated antigens
(step 1 again) to increase the breadth and depth of the response in subsequent revolutions of the cycle. In
cancer patients, the Cancer-Immunity Cycle does not perform optimally. Tumor antigens may not be detected,
DCs and T cells may treat antigens as self rather than foreign thereby creating T regulatory cell responses
rather than effector responses, T cells may not properly go well with tumors, may be inhibited from infiltrating
the tumor, or (most importantly) factors in the tumor microenvironment might suppress those effector cells
that are produced.

The goal of cancer immunotherapy is to initiate or reinitiate a self-sustaining cycle of cancer immunity,
enabling it to amplify and propagate, but not so much as to generate unrestrained autoimmune inflammatory
responses. Cancer immunotherapies must, therefore, be carefully configured to overcome the negative
feedback mechanisms. Although checkpoints and inhibitors are built into each step that opposes continued
amplification and can dampen or arrest the antitumor immune response, the most effective approaches will
involve selectively targeting the rate-limiting step in any given patient. Amplifying the entire cycle may
provide anticancer activity but at the potential cost of unwanted damage to normal cells and tissues. Many
recent clinical results suggest that a common rate-limiting step is an immunostat function, immunosuppression
that occurs in the tumor microenvironment

facebook.com/HonorsGroup
READING SUB-TEST

Text D

Attempts to activate or introduce cancer antigen-specific T cells, as well as stimulate the proliferation
of these cells over the last 20 years, have led to mostly no, minimal or modest appreciable anticancer
immune responses. The majority of these efforts involved the use of therapeutic vaccines because
vaccines can be easy to deploy and have historically represented an approach that has brought
enormous medical benefit. Yet, cancer vaccines were limited to two accounts. First, until recently,
there was a general lack of understanding of how to immunize the patients for effective cytotoxic T cell
responses. This limitation reflects continued uncertainties concerning the identities of antigens to use,
their mode of delivery, the types of adjuvants required, and the proximal characteristics of the desired
T cell response. Second, the presence of the immunostat in the tumor microenvironment may dampen
or disable antitumor immune responses before clinically relevant tumor kill can occur. Thus, as long
as these negative signals are in place, the prospects for vaccine-based approaches used alone are likely
to be limited. - Mark Stephens, Cardiologist, Care CRM, London.

facebook.com/HonorsGroup
READING SUB-TEST

Questions 1-7

For each question, 1-7, decide which text (A, B, C or D) the information comes from. You may use any letter
more than once.

In which text can you find information about;

1 Seven crucial progression acts.

Answer :

2 Initiating Anticancer Immunity.

Answer :

3 Checkpoints and (immunostat function)

Answer :

4 Didn`t achieve any proper result.

Answer :

5 Talking about the process of removal of cancer cells.

Answer :

6 Killing of target cancer cell.

Answer :

facebook.com/HonorsGroup
READING SUB-TEST

7 Immune editing.

Answer :

Questions 8-14

Answer each of the questions, 8-14, with a word or short phrase from one of the texts. Each answer may
include words, numbers or both

8 Did experiments or endeavors with respect to T-cells lead to any significant results or immune responses?

Answer :

9 At which stage the nature of the immune response is determined?

Answer :

10 Which cells may find it difficult to home to tumors?

Answer :

11 What is the term referred to describe the series of events involved in anticancer immune response?

Answer :

12 Presence of what will dampen or disable antitumor immune responses in the tumor microenvironment?

Answer :

13 What are the most effective approaches in an immunotherapy?

Answer :

facebook.com/HonorsGroup
READING SUB-TEST

14 What target was not achieved so far in immunizing cancer patients?

Answer :

Questions 15-20

Complete each of the sentences, 15-20, with a word or short phrase from one of the texts. Each answer may
include words, numbers or both.

15 Destroying the cancer cell releases additional .

16 is referred to as partial or complete suppression of the immune response of an individual

17 Majority of the processes with respect to activation of the T-cells comprises use of .

18 Presently, the challenge is how to effectively use this understanding and develop .

19 In , activated effector T cells may entirely infiltrate the tumor bed.

20 The initial step can provide a response only when it is along with , which can clearly show
immunity.

facebook.com/HonorsGroup
READING SUB-TEST

Part B

In this part of the test, there are six short extracts relating to the work of health professionals. For
questions 1-6, choose the answer (A, B or C) which you think fits best according to the text.

GCA - Giant Cell Arteritis

The onset of giant cell arteritis (GCA) may be either abrupt or insidious. GCA may begin with constitutional
manifestations such as anorexia, fever, malaise, myalgia, night sweat, and weight loss. These prodromal
symptoms may occur for a few days and may even stretch out to weeks.

The most commonly reported symptoms in patients with GCA are as follows:

 Headache (initial symptom in 33%, present in 72%)


 Neck, torso, shoulder, and pelvic girdle pain that is consistent with polymyalgia
rheumatica (PMR; initial in 25%, present in 58%)
 Fatigue and malaise (initial in 20%, present in 56%)
 Jaw claudication (initial in 4%, present in 40%)
 Fever (initial in 11%, present in 35%)

Neurosurgeon Sergio Canavero proposed the HEAVEN procedure – i.e. head anastomosis venture – several
years ago, and has recently received approval from the relevant regulatory bodies to perform this body-head
transplant (BHT) in China. The BHT procedure involves attaching the donor body (D) to the head of the
recipient (R), and discarding the body of R and head of D. Canavero’s proposed procedure will be incredibly
difficult from a medical standpoint. Aside from medical doubt, the BHT has been met with great resistance
from many, if not most bio- and neuro ethicists.

facebook.com/HonorsGroup
READING SUB-TEST

LS surveillance support initiatives or ideas have included earlier and more frequent colonoscopy, consideration
for the transvaginal ultrasound and endometrial sampling with the possible recommendation for a total
hysterectomy, and clinical neurologic examination. Upper endoscopy, annual urinalysis, and pancreatic cancer
screening may also be considered based on family history. Screening guidelines continue to be updated as new
research clarifies what recommendations are beneficial, but few participants receive repeat genetic counseling
to update surveillance and surgical recommendations based on a patient’s specific LS gene mutation and
family history.

Hereditary breast cancers account for approximately 10% of all breast cancers, and approximately 23% of all
ovarian cancers are considered hereditary. According to Plakhins et al., BRCA1pathogenic founder mutations
contribute to 3.77% of all consecutive primary breast cancers and 9.9% of all consecutive primary ovarian
cancers. BRCA1 and BRCA2 pathogenic founder mutation analysis is a relatively straightforward and cost-
effective screening strategy to identify mutation carriers.In Latvia, all consecutive breast and ovarian cancer
cases are eligible for BRCA1 pathogenic founder mutations (c.181 T > G, c.4035delA, c.5266dupC) screening,
and the costs of the test are covered by the public health care system.

The average age of PMR patients

Total (female & male)

Female

Male

PMR versus Ath

0,4494

facebook.com/HonorsGroup
READING SUB-TEST

0,2972

0,4217

PMR with Ath versus without Ath

0,4222

0,2703

0,0004

PMR with TA versus without TA

0,0164

0,0159

0,7951

PMR with amyloidosis versus without amyloidosis

0,0000

0,0000

0,0037

TA with amyloidosis versus without amyloidosis

0,2995

0,3727

0,7228

facebook.com/HonorsGroup
READING SUB-TEST

Patient clinical data.

Features Description Ranges

Age Age (in years) 30–86

Gender 1: male; 0: female 0–1

HTN Hypertension, 0: no; 1: yes 0–1

RBS Random blood sugar 57–180

Chest pain type 0: nonspecific chest pain 0–2

  1: atypical chest pain  

  2: typical angina  

HT Height (cm) 133–188

WT Weight (kg) 33–110

DBP Diastolic blood pressure (mmHg) 46–110

SBP Systolic blood pressure (mmHg) 100–170

CAD Coronary artery disease 0: no; 1: yes

facebook.com/HonorsGroup
READING SUB-TEST

Questions 1-6

1 The GCA symptoms can;

A Begin and grow suddenly or gradually.

B Begin all of sudden.

C Occur more gradually.

2 Head anastomosis venture may mean;

A human head transplantation.

B head transplantation in China.

C adding head of one patient to the body of the other.

3 What this notice talk about?

A Patients with LS should undergo a range of ongoing surveillance activities.

B Patients with LS are recommended to undergo a range of ongoing surveillance activities.

C There are a wide range of ongoing surveillance activities which are undertaken by caretakers.

4 As per the given notice, hereditary breast cancers;

A occur commonly due to pathogenic mutations.

facebook.com/HonorsGroup
READING SUB-TEST

B can easily be detected with advanced strategy.

C can be treated more cost-effectively.

5 What is correct?

A The average age of PMR patients with amyloidosis is higher than the average age of PMR patients
without TA.

B the average age of PMR patients without TA (p < 0.0164) is significantly lower than the average age of
PMR patients without amyloidosis.

C The average age of PMR patients with TA is significantly higher.

6 Patient clinical data shows that the patient;

A is suffering from BP.

B is suffering from heart disease.

C is suffering from chest pain.

6 Patient clinical data shows that the patient;

A is suffering from BP.

B is suffering from heart disease.

C is suffering from chest pain.

facebook.com/HonorsGroup
READING SUB-TEST

Part C

In this part of the test, there are two texts about different aspects of healthcare. For questions 7-
22, choose the answer (A, B, C or D) which you think fits best according to the text.

Text 1: A note on Eye Infections – Conjunctivitis

Conjunctivitis is a common and often very contagious condition more commonly referred to as a pink eye.
Conjunctivitis often involves inflammation and swelling of the conjunctiva or the clear membrane covering the
eye and lining the inner eyelids. It is very common among young children, particularly those in school or
daycare. Elderly people can also be affected. Conjunctivitis typically poses no real threat to the patient`s vision
or wellness. There are multiple causes and risk factors for conjunctivitis. Some of the more common causes of
conjunctivitis include Allergic reactions, Viral contamination, Bacterial contamination, Exposure to irritants or
chemical pollutants. Infectious conjunctivitis typically results from bacterial or viral contamination. While
bacterial conjunctivitis may be treated with certain antibiotics, there is no traditional treatment for viral
conjunctivitis. Multiple strains of bacteria are responsible for bacterial conjunctivitis; these include
haemophilus influenzae and Staphylococcus aureus. Viral conjunctivitis is a common condition affecting
children from the time of birth through to their adult years. Viruses can enter the eye in many ways, resulting
from the common cold or contagion with the flu. Some patients may develop a more serious form of viral
conjunctivitis associated with a herpes infection; in this case, prompt medical attention is necessary to prevent
permanent damage to the eye or vision.

One of the more commonly reported signs and symptoms of conjunctivitis include discomfort and pain in the
eye, accompanied by redness or swelling of the eyelid, hence the name "pinkeye." Patients may experience
reddening of the inner and outer eyelid or may experience swollen eyelids. Some patients may report increased
sensitivity to light and other irritants including the wind. Many patients with conjunctivitis will have discharge
from the eyes that may be clear, white, green or yellow in color. Yellow or green discharge from the eye may
suggest an infection and may require antibiotics or other forms of aggressive treatment. Conjunctivitis can
spread from person to person or from eye to eye. Simply touching one eye then the other can spread viral
conjunctivitis. Allergic conjunctivitis is a non-contagious condition. Typically, children exposed to bacterial
conjunctivitis will have a two to four day window before symptoms appear.

facebook.com/HonorsGroup
READING SUB-TEST

Bacterial infections mostly come from staphylococci and streptococci organisms that can come from your own
skin or upper respiratory tract. The indicating symptoms of bacterial infections are thick, ropy mucus discharge
accompanied with red, irritated and inflamed eyes. Luckily, bacterial eye infections are easily treated with
antibiotic eye drops and, in most cases, will clear up within a few short days. Viral infections are commonly
caused by an enterovirus and often associated with an upper respiratory infection or common cold. Eyes are
red and inflamed and become watery and runny. One of the most common viral infections is epidemic
keratoconjunctivitis, also known as EKC; it is highly contagious and can last up to 2 weeks or more. This viral
conjunctivitis is caused by an adenovirus and does not have a specific treatment to cure the infection. The
doctor may prescribe steroid eye drops and artificial tears to help decrease inflammation, but mostly the virus
simply needs to run its course.

Conjunctivitis may also result from chlamydia and gonococcal infections or STD`s. Usually, the inner eyelid
becomes infected. This condition is more commonly noted in teens and young adults who are sexually active.
When left untreated, this condition may affect newborn infants born to mothers infected with an STD. Signs
may include a history of pelvic pain or vaginitis as well. Patients with Gonococcal infections may feel like a
foreign object is chronically present within their eye, and are more likely to experience burning and
inflammation. It is possible to transfer these conditions to the eye from hand contact so it is important, to help
prevent the spread of infection, that frequent hand washing is adopted by patients and family members.
Treatment usually involves use of antibiotics taken topically or orally and concomitant treatment may be
necessary to treat genital and eye infections.

Text 2: Osteomyelitis is an infection of the bone.

Osteomyelitis is an infection of the bone. Osteomyelitis can occur in infants, children, and adults. Different
types of bacteria typically affect the different age groups: in children, osteomyelitis most commonly occurs at
the ends of the long bones of the arms and legs, affecting the hips, knees, shoulders, and wrists, whereas in
adults, it is more common in the bones of the spine (vertebrae), feet, or in the pelvis. There are several
different ways to develop the bone infection of osteomyelitis. The first is for bacteria to travel through the
bloodstream (bacteremia) and spread to the bone, causing an infection. This mostly occurs when the patient
has an infection elsewhere in the body, such as pneumonia or a urinary tract infection that spreads through the
blood to the bone. An open wound over a bone can lead to osteomyelitis. A recent surgery or injection around

facebook.com/HonorsGroup
READING SUB-TEST

a bone can also expose the bone to bacteria and lead to osteomyelitis. Patients with conditions or taking
medications that weaken their immune system are at a higher risk of developing osteomyelitis. Risk factors
include cancer, chronic steroid use, sickle cell disease, human immunodeficiency virus (HIV), diabetes,
hemodialysis, intravenous drug users, and the elderly.

Symptoms of osteomyelitis can vary greatly. In children, osteomyelitis usually occurs more quickly. They
develop pain or tenderness over the affected bone, and they may have difficulty or inability to use the affected
limb or to bear weight or walk due to severe pain. They may also have fever, chills, and redness at the site of
infection. In adults, the symptoms often develop more gradually and include fever, chills, irritability, swelling
or redness over the affected bone, stiffness, and nausea. In people with diabetes, peripheral neuropathy, or
peripheral vascular disease, there may be no pain or fever. The only symptom may be an area of skin
breakdown that is worsening or not healing. Acute osteomyelitis occurs with a rapid onset and is usually
accompanied by the symptoms of pain, fever, and stiffness. It generally occurs after a break in the skin from
injury, trauma, surgery, or skin ulceration from wounds. Chronic osteomyelitis is insidious in onset; it may be
the result of a previous infection of osteomyelitis and, despite multiple courses of antibiotics, it may reoccur.
Symptoms of chronic osteomyelitis are subtle but may include fever, pain, redness, or discharge at the site of
infection.

The diagnosis of osteomyelitis begins with a complete medical history and physical examination. During the
discussion about medical history, the doctor may ask questions about recent infections elsewhere in the body,
past medical history, medication usage, and family medical history. The physical examination will look for
areas of tenderness, redness, swelling, decreased or painful range of motion, and open sores. The doctor may
then order tests to help diagnose osteomyelitis. Several blood tests can be used to help determine if there is an
infection present; these include a complete blood count (CBC), the erythrocyte sedimentation rate (ESR), C-
reactive protein (CRP), and blood cultures. None of these is specific for osteomyelitis but they can suggest that
there may be some infection in the body. Imaging studies of the involved bones may be obtained; these can
include plain radiographs (X-rays), bone scans, computed tomography (CT) scans, magnetic resonance
imaging (MRIs), and ultrasounds. These imaging studies can help identify changes in the bones that occur with
osteomyelitis.

In many cases, osteomyelitis can be effectively treated with antibiotics and pain medications. If a biopsy is
obtained, this can help guide the choice of the best antibiotic. The duration of treatment of osteomyelitis with

facebook.com/HonorsGroup
READING SUB-TEST

antibiotics is usually four to eight weeks but varies depending on the type of infection and the response to the
treatments. In some cases, the affected area will be immobilized with a brace to reduce the pain and speed up
the treatment. Sometimes, surgery may be necessary. If there is an area of localized bacteria (abscess), this
may need to be opened, washed out, and drained. If there is damaged soft tissue or bone, this may need to be
removed. If bone needs to be removed, it may need to be replaced with bone graft or stabilized during surgery.

With early diagnosis and appropriate treatment, the prognosis for osteomyelitis is good. Antibiotics regimes
are used for four to eight weeks and sometimes longer in the treatment of osteomyelitis depending on the
bacteria that caused it and the response of the patient. Usually, patients can make a full recovery without
longstanding complications.

However, if there is a long delay in diagnosis or treatment, there can be severe damage to the bone or
surrounding soft tissues that can lead to permanent deficits or make the patient more prone to recurrence. If
surgery or bone grafting is needed, this will prolong the time it takes to recover.

facebook.com/HonorsGroup
READING SUB-TEST

Text 1: Questions 7-14

7 Which part of the eye is affected by conjunctivitis?

A Conjunctiva

B White membrane

C Eyelids

D Only A and B

8 Conjunctivitis is common among;

A Children

B Teens

C School goers

D Daycare children

9 What is linked with the herpes infection?

A Bacterial infections.

B Viral infections.

C Allergic reactions.

D Exposure to chemical pollutants.

facebook.com/HonorsGroup
READING SUB-TEST

10 What is the most common symptom of conjunctivitis?

A Mild pain in the eye.

B Watery eyes.

C Reddening of the inner and outer eyelid.

D All of the above.

11 The color of discharge from eyes may be;

A Yellow and green.

B White, yellow and green.

C Yellow or green.

D White or green.

12 Paragraph 3 is focused more on;

A Types of conjunctivitis.

B Types of bacterial conjunctivitis.

C Types of viral conjunctivitis.

D Types of bacterial and viral conjunctivitis.

facebook.com/HonorsGroup
READING SUB-TEST

13 viral conjunctivitis is caused by;

A Androvirus

B Adenovirus

C Enterovirus

D Adenonvirus

14 Which is more common in gonococcal infections?

A Pain around pelvis.

B Discharge or pus.

C Inflammation of the eyes.

D Reddening of the eyes.

Text 2: Questions 15-22

15 Which of the following statements is not true, according to the information provided in
paragraph 1?

A An osteomyelitis is common among people of all ages.

B An osteomyelitis can be caused by bacteria.

C In most of the cases, people who have pneumonia may develop osteomyelitis.

D An osteomyelitis is commonly observed in people who are prone to bacterial infections.

facebook.com/HonorsGroup
READING SUB-TEST

16 Which of the following can lead to osteomyelitis?

A Bacterial infection.

B An open wound.

C Underrated bone operation.

D None

17 Symptoms of osteomyelitis in children may include;

A Tenderness in the bone;

B High fever;

C Inability to walk;

D All of the above;

18 Which type of osteomyelitis is slow in the beginning?

A Chronic

B Acute

C Both chronic and acute

D None

facebook.com/HonorsGroup
READING SUB-TEST

19 Which group is more specific for diagnosis?

A CBC and ESR only

B Only ESR and CRP

C CRP, ESR and CBC

D None

20 According to paragraph 3, involved bone suggests

A Wounded bone

B Operated bone area

C Bacterial infected bone

D Bone with tenderness

21 According to paragraph 4, which one of the following statements is not correct?

A Treatment may be completed within 4 to 8 weeks.

B The duration of the treatment varies depending on the type of osteomyelitis.

C Working of the affected area is stopped in order to speed up the treatment.

D Bone is often replaced with bone graft for perfect treatment.

facebook.com/HonorsGroup
READING SUB-TEST

22 According to paragraph 5, delay in treatment of diagnosis may result in;

A Inability to walk properly.

B Permanent damage to bone.

C Removal of affected bone.

D None

facebook.com/HonorsGroup
READING SUB-TEST

ANSWERS

PART A

1. Correct Answer Is: B


2. Correct Answer Is: D
3. Correct Answer Is: A
4. Correct Answer Is: D
5. Correct Answer Is: B
6. Correct Answer Is: C
7. Correct Answer Is: A
8. Correct Answer Is: No
9. Correct Answer Is: Stage 3
10. Correct Answer Is: T cells
11. Correct Answer Is: Cancer-Immunity Cycle
12. Correct Answer Is: immunostat
13. Correct Answer Is: selectively targeting the rate-limiting step
14. Correct Answer Is: effective cytotoxic T cell responses
15. Correct Answer Is: tumor-associated antigens
16. Correct Answer Is: immunosuppression
17. Correct Answer Is: therapeutic vaccines
18. Correct Answer Is: new drugs
19. Correct Answer Is: step 4
20. Correct Answer Is: signals

PART B & C

1. Correct Answer Is: A Begin and grow suddenly or gradually.


2. Correct Answer Is: A human head transplantation.
3. Correct Answer Is: B Patients with LS are recommended to undergo a range of ongoing surveillance
activities.
4. Correct Answer Is: B can easily be detected with advanced strategy.
5. Correct Answer Is: C The average age of PMR patients with TA is significantly higher.
6. Correct Answer Is: A is suffering from BP.
7. Correct Answer Is: D Only A and B
8. Correct Answer Is: D Daycare children
9. Correct Answer Is: B Viral infections.
10. Correct Answer Is: C Reddening of the inner and outer eyelid.
11. Correct Answer Is: B White, yellow and green.
12. Correct Answer Is: A Types of conjunctivitis.
13. Correct Answer Is: B Adenovirus
14. Correct Answer Is: C Inflammation of the eyes.
15. Correct Answer Is: C In most of the cases, people who have pneumonia may develop osteomyelitis.
16. Correct Answer Is: B An open wound.
17. Correct Answer Is: D All of the above;
18. Correct Answer Is: A Chronic
19. Correct Answer Is: D None
20. Correct Answer Is: C Bacterial infected bone
21. Correct Answer Is: D Bone is often replaced with bone graft for perfect treatment.
22. Correct Answer Is: B Permanent damage to bone.

facebook.com/HonorsGroup
READING SUB-TEST

OET 2.0 READING TASK 4

Part A

TIME: 15 minutes

Look at the four texts, A-D, in the separate Text Booklet.

For each question, 1-20, look through the texts, A-D, to find the relevant information.

Write your answers on the spaces provided in this Question Paper.

Answer all the questions within the 15-minute time limit.

Your answers should be correctly spelt.

Text A

Chronic fatigue syndrome (CFS

Chronic fatigue syndrome (CFS) is a highly complex illness that results in significant disability and a
considerably diminished quality of life. Due to continued questions regarding etiology, the period of onset for
the illness is of considerable interest to researchers in the field. There is fairly little strife as to whether the
illness labels CFS, ME, ME/CFS, myalgic encephalopathy, and SEID represent one distinct condition, whether
they are part of an illness spectrum, or whether they are simply different terms used to describe the same
condition. CFS, ME, ME/CFS, and the recently named SEID, are often associated with different case criteria.
Early case criteria developed by Holmes. specify that the illness must have a “new onset of persistent or
relapsing, debilitating fatigue” without any previous history of similar problems. Additionally, Holmes
stipulate that the main symptoms of CFS must occur over a few hours or days, indicating a sudden or acute
onset. According to Holmes, symptoms are only met if they begin at the time of the fatigue onset or following
onset. Another case criteria for CFS is referred to as the Oxford Criteria, which stipulate that CFS involves a
“definite” onset as well as clear evidence of infection at the time of onset or first symptoms. Similar to the
Oxford Criteria, the Fukuda criteria describe the onset of the fatiguing illness as “new” and “definite”.

facebook.com/HonorsGroup
READING SUB-TEST

Text B

The terms Myalgic Encephalomyelitis (ME) and Myalgic Encephalomyelitis/chronic fatigue syndrome
(ME/CFS) have corresponding case criteria, which are different from the Fukuda criteria in that they require
what are considered by many to be key symptoms of the illness (e.g. post-exertional malaise and cognitive
dysfunction). The criteria for ME/CFS specify that an individual must have a “significant degree of new onset”
fatigue. Similar to the Holmes criteria for CFS, the case criteria for ME/CFS stipulate that symptoms can only
be counted as meeting criteria if they occur or become significantly worse after the onset of the illness.
Carruthers et al. describe onset as “distinct” and assert that most individuals experience an acute onset;
however, they also assert that some individuals are unhealthy prior to their ME/CFS onset and may not be able
to identify a specific trigger for the development of ME/CFS, or they may experience a more “gradual” or
“insidious” onset.

Text C

Hyde’s Nightingale Definition of ME stipulates that ME is both chronic and disabling and is characterized by
an acute onset. Additionally, Hyde describes ME as an epidemic or an endemic occurring in two phases.
Additionally, Hyde indicates that ME often follows multiple, minor infections in individuals with susceptible
immune systems or immune systems that are weakened by severe stressors. Hyde describes the initial phase of
ME as the Primary Infection Phase, which is characterized as an epidemic or endemic infectious disease with
an incubation period of between four and seven days. He describes the second phase as the Secondary Chronic
Phase, occurring with two to seven days of the Primary Infection Phase. In this phase, Hyde asserts that there
are measurable changes in the central nervous system (CNS) of an affected individual and that this phase is the
chronic form of the disease that is most commonly depictive of ME. Understanding the cause of ME can be
like going through labyrinthine routes in scientific discovery, as Hyde asserts that all cases of epidemic and
primary ME result from an infectious or autoimmune agent, but he also suggests that there are often other
potential causes that may go unnoticed prior to the onset of illness or as part of the illness.

facebook.com/HonorsGroup
READING SUB-TEST

Text D

There is not one universally used or empirically derived definition of onset duration for ME and CFS.
Researchers interested in assessing the mode of illness onset have used various definitions. Often, the
distinction is made between sudden and acute onset and insidious and gradual onset, but the duration length
prescribed to each category differs. For instance, DeLuca et al. defined sudden onset for CFS as an
unrecoverable “viral-like illness” that could be traced to a definite date. Gradual onset was described as a
“slow progression of symptoms over a period of weeks to several months” or longer. In his study of CFS in an
adolescent sample, Bell defined sudden/acute onset as an “abrupt onset of constant and debilitating fatigue that
could be dated to a specific event or illness” . He described all other onset patterns as gradual.

Questions 1-7

For each question, 1-7, decide which text (A, B, C or D) the information comes from. You may use any letter
more than once.

In which text can you find information about;

1 Occurs in two different phases.

Answer

2 All patterns are gradual.

Answer

facebook.com/HonorsGroup
READING SUB-TEST

3 He believed that ME is more complicated.

Answer

4 Controversy over the disease conditions.

Answer

5 No clear definition.

Answer

6 Theory which suggests beginning of body weakening.

Answer

7 Causes a spike in symptoms and a massive energy crash.

Answer

Questions 8-14

Answer each of the questions, 8-14, with a word or short phrase from one of the texts. Each answer may
include words, numbers or both.

8 Who thinks that ME is long lasting and ant it can be acquired extreme onset?

Answer

facebook.com/HonorsGroup
READING SUB-TEST

9 Who defined onset as incurable?

Answer

10 According to which principle, CFS involves a definite onset & clear evidence of infection?

Answer

11 Who specified that the CFS symptoms must occur over a few hours or days, indicating an acute onset?

Answer

12 Who described that sudden onset for CFS as an unrecoverable viral-like illness?

Answer

13 Whose statement signifies that the affected patient may go through a highly secretive and detrimental
beginning?

Answer

14 Who described onset as more debilitating fatigue?

Answer

Similar to the Oxford Criteria

Questions 15-20

facebook.com/HonorsGroup
READING SUB-TEST

Complete each of the sentences, 15-20, with a word or short phrase from one of the texts. Each answer may
include words, numbers or both.

15 described the onset phase of ME as the Primary Infection

16 ME may follow infections in patients with not-so-strong immune system

17 CFS criteria are said to have much more similarity with respect to

18 CFS may comprise a onset and strong evidence of infection when there are first signs of
symptoms.

19 The distinction is made between and insidious/gradual onset, but the duration length
prescribed to each category differs.

20 may occur due to an infectious or autoimmune agent.

facebook.com/HonorsGroup
READING SUB-TEST

Part B

In this part of the test, there are six short extracts relating to the work of health professionals. For questions 1-
6, choose the answer (A, B or C) which you think fits best according to the text.

Gait disturbance

Gait disturbance is the most common problem after stroke. This problem is related to poor ADL and mobility,
and increases the risk of fall in severe cases. The body alignment of stroke patients becomes asymmetric if
they have a hemi-paralysis, muscle weakness, motor and sensory function decrease. These problems produce a
hemiplegic gait in stroke patients. It may include poor equilibrium reaction, and impaired selective motor
control. Good body alignment is very important clinically because asymmetry leads to inefficient energy
during walking, the risk of musculoskeletal injury in the unaffected side, and loss of bone density. Excessive
pelvic elevation and the pelvic tilt angle is directly connected to hemiplegic gait and poor motor function in
stroke patient causes an excessive pelvic tilt during gait

facebook.com/HonorsGroup
READING SUB-TEST

IV Insulin Infusions

Patients that have required IV insulin infusions can be transitioned to subcutaneous insulin once infusion rates
are stable and glucose controlled, particularly if a diet has been initiated. Because IV insulin has a very short
half-life, the subcutaneous insulin should be administered prior to discontinuation of IV insulin. The basal
infusion rate during fasting is a good predictor of basal subcutaneous insulin requirements, however, providers
often reduce the amount by 20% upon transition. For instance, if a patient required 1.5 unit/hour of IV insulin
overnight, this would suggest a basal need of approximately 36 units of insulin daily. However, reducing this
by 20% would result in a starting basal dose of 30 units of insulin daily.

Molecule-altering technologies

The advent of molecule-altering technologies and improved synthetic methods has led to the finding of newer
proteins and peptides that resemble human proteins and peptides. Although, capable of producing potential
therapeutic benefits, protein molecules have serious biopharmaceutical concerns such as, poor shelf- life, rapid
degradation in the physiological environment, poor solubility, immunogenicity and antigenicity. These
concerns can be overcome by utilizing the beneficial properties of polyethylene glycols and PEGylation.
‘PEGylation’ is the process of chemical attachment of PEG to bioactive proteins and peptides, to modify their
pharmacokinetic and pharmacodynamic properties.

facebook.com/HonorsGroup
READING SUB-TEST

Moderate mid-postprandial exercise

Although an uneasy consensus favoring moderate mid-postprandial exercise as better suited for glycaemia
benefits exists among researchers, pre-meal exercise has its advantages: little risk for hypoglycemia, enhanced
insulin sensitivity, and improved muscle glycogen content and GLUT-4 protein levels. The downside of pre-
meal exercise is elevated postprandial glucose (PPG), A recent systematic review concluded that >45 min of
aerobic exercise (AE) post-meal offered consistent glycaemia benefits. Also, the review identified resistance
training as an effective modality in this regard. The patient found a 30-min pre-meal walk followed by another
30 minutes of post-meal walk blunted the post-meal glucose surge better than a 60 min pre-meal or post-meal
walk. The post-meal walk might have cleared some of the extra blood glucose arrived from the liver. Split
exercise at lunchtime was comparable to mid-postprandial exercise in improving glycaemia and oxidative
stress, although there was less hyperglycemia after the meal.

Zidovudine

Nausea is another common side effect that may be present in early zidovudine use. Other side effects of
zidovudine may include granulocytopenia, myopathy, lactic acidosis, hepatomegaly with steatosis, headache.
Myopathy may occur within 6–12 months of initiating zidovudine, and has an insidious onset that involves
proximal muscle weakness and exercise-induced myalgias. The mechanism of myopathy is believed to be
mitochondrial toxicity within myocytes. Zidovudine should be used with caution in patients who have anemia
(hemoglobin less than 9.5 grams/deciliter). Reduction of hemoglobin may occur as early as 2 to 4 weeks.
Severe anemia may require dose adjustment, discontinuation, and/or blood transfusions. Doses should be

facebook.com/HonorsGroup
READING SUB-TEST

reduced until bone marrow recovers if the anemia is significant (hemoglobin less than 7.5 grams/deciliter or
reduction of greater than 25% of baseline).

Amikacin and gentamicin dosing in critically and noncritically ill patients

Critically ill (%) Non Critically ill (%) P

Dose of amikacin

500 mg every 8 h 9/11 (81.8) 2/5 (40) 0.087*

500 mg every 12 h 1/11 (9.1) 3/5 (60)

500 mg every 24 h 1/11 (9.1) 0/0

Dose of gentamicin

(mg/kg)

5 3/9 (33.3) 0/22 0.003*

4 1/9 (11.1) 0/22

3 3/9 (33.3) 3/22 (13.6)

2.5 1/9 (11.1) 2/22 (9.1)

2 1/9 (11.1) 17/22 (77.3)

facebook.com/HonorsGroup
READING SUB-TEST

1 What is correct about gait disturbance?

Pelvic tilt is common

Fast gait speed and improved gait pattern is the most important goal of stroke rehabilitation

A hemiplegic gait may include body asymmetry, decreased weight bearing on the affected side.

2 The notice is talking about;

Postoperative care.

functions of IV insulin

insulin dosage

3 The use of molecule-altering technologies;

has flared up more concerns.

is not suitable.

is fast advancing.

4 The notice is talking about;

when to perform the exercise

best exercise timing

facebook.com/HonorsGroup
READING SUB-TEST

benefits of exercise

5 Zidovudine can;

Increase decrease of lactate.

Lead to fatty change

Be used for treating anemia

6 The table

doesnt show a much significant difference in gentamicin doses between two groups of patients

Use of gentamicin dose is significantly lower

Dose of 2 mg/kg is higher in noncritically ill patients.

facebook.com/HonorsGroup
READING SUB-TEST

Part C

In this part of the test, there are two texts about different aspects of healthcare. For questions 7-22, choose the
answer (A, B, C or D) which you think fits best according to the text.

Text 1: What is Yersiniosis?

Yersiniosis is an infectious disease caused by a bacterium of the genus Yersinia. In the United States, most
human illness is caused by one species, Y enterocolitica (not more than one species often). Infection with Y.
enterocolitica can cause a variety of symptoms depending on the age of the person infected. Infection with Y.
enterocolitica occurs most often in young children; adults may be on a safer side. Common symptoms in
children are fever, abdominal pain, and diarrhea, which is often bloody. Symptoms typically develop 4 to 7
days after exposure and may last 1 to 3 weeks or longer. In older children and adults, right-sided abdominal
pain and fever may be the predominant symptoms, and may be confused with appendicitis. In a small
proportion of cases, complications such as skin rashes, joint pains, or the spread of bacteria to the bloodstream
can occur.

Y. enterocolitica belongs to a family of rod-shaped bacteria. Other species of bacteria in this family include Y.
pseudotuberculosis, which causes an illness similar to Y. enterocolitica, and Y. pestis, which cause plague.
Only a few strains of Y. enterocolitica cause illness in humans. The major animal reservoir for Y.
enterocolitica strains that causes human illness is pigs, but other strains are also found in many other animals
including rodents, rabbits, sheep, cattle, horses, dogs, and cats. In pigs, the bacteria are most likely to be found
on the tonsils.

Infection is most often acquired by eating contaminated food, especially raw or undercooked pork products.
The preparation of raw pork intestines (chitterlings) may be particularly risky. Infants can be infected if their
caretakers handle raw chitterlings and then do not adequately clean their hands before handling the infant or
the infant’s toys, bottles, or pacifiers. Drinking contaminated unpasteurized milk or untreated water can also
transmit the infection. Occasionally Y. enterocolitica infection occurs after contact with infected animals. On
rare occasions, it can be transmitted as a result of the bacterium passing from the stools or soiled fingers of one
person to the mouth of another person. This may happen when basic hygiene and handwashing habits are
inadequate. Rarely, is the organism transmitted through contaminated blood during a transfusion.

facebook.com/HonorsGroup
READING SUB-TEST

Y. enterocolitica is a relatively infrequent cause of diarrhea and abdominal pain. Based on data from the Food-
borne Diseases Active Surveillance Network which measures the burden and sources of specific diseases over
time, approximately one culture-confirmed Y. enterocolitica infection per 100,000 people occurs each year.
Children are infected more often than adults, and the infection is more common in the winter. Y. enterocolitica
infections are generally diagnosed by detecting the organism in stools. Many laboratories do not routinely test
for Y. enterocolitica, so it is important to notify laboratory personnel when infection with this bacterium is
suspected so that special tests can be done. The organism can also be recovered from other sites, including the
throat, lymph nodes, joint fluid, urine, bile, and blood. Uncomplicated cases of diarrhea due to Y.
enterocolitica usually resolve on their own without antibiotic treatment. However, in more severe or
complicated infections, antibiotics such as aminoglycosides, doxycycline, trimethoprim-sulfamethoxazole, or
fluoroquinolones may be useful.

There are many things which can be done to prevent the infection or the spread of the infection:

Avoid eating raw or undercooked pork. Consume only pasteurized milk or milk products. Wash hands with
soap and water before eating and preparing food, after contact with animals, and after handling raw meat. After
handling raw chitterlings, clean hands and fingernails scrupulously with soap and water before touching infants
or their toys, bottles, or pacifiers. Someone other than the food handler should care for children while
chitterlings are being prepared. Prevent cross-contamination in the kitchen - use separate cutting boards for
meat and other foods, carefully clean all cutting boards, counter-tops, and utensils with soap and hot water
after preparing raw meat. Dispose of animal feces in a sanitary manner.

Text 2 : What is an MRI scan?

An MRI (or magnetic resonance imaging) scan is a radiology technique that uses magnetism, radio waves, and
a computer to produce images of body structures. The MRI scanner is a tube surrounded by a giant circular
magnet. The patient is placed on a moveable bed that is inserted into the magnet. The magnet creates a strong
magnetic field that aligns the protons of hydrogen atoms, which are then exposed to a beam of radio waves.
This spins the various protons of the body, and they produce a faint signal that is detected by the receiver
portion of the MRI scanner. The receiver information is processed by a computer, and an image is produced.
The image and resolution produced by MRI are quite detailed and can detect tiny changes of structures within
the body. For some procedures, contrast agents, such as gadolinium, are used to increase the accuracy of the
images.

facebook.com/HonorsGroup
READING SUB-TEST

An MRI scan can be used as an extremely accurate method of disease detection throughout the body. In the
head, trauma to the brain can be seen as bleeding or swelling. Other abnormalities often found include brain
aneurysms, strokes, tumors of the brain, as well as tumors or inflammation of the spine. Neurosurgeons use an
MRI scan not only in defining brain anatomy but in evaluating the integrity of the spinal cord after trauma. It is
also used when considering problems associated with the vertebrae or intervertebral discs of the spine. An MRI
scan can evaluate the structure of the heart and aorta, where it can detect aneurysms or tears. It provides
valuable information on glands and organs within the abdomen, and accurate information about the structure of
the joints, soft tissues, and bones of the body. Often, surgery can be deferred or more accurately directed after
knowing the results of an MRI scan.

An MRI scan is a painless radiology technique that has the advantage of avoiding x-ray radiation exposure.
There are no known side effects of an MRI scan. The benefits of an MRI scan relate to its precise accuracy in
detecting structural abnormalities of the body. Patients who have any metallic materials within the body must
notify their physician prior to the examination or inform the MRI staff; metallic chips, materials, surgical clips,
or foreign material can significantly distort the images obtained by the MRI scanner. Patients who have heart
pacemakers, metal implants, or metal chips or clips in or around the eyeballs cannot be scanned with an MRI
because of the risk that the magnet may move the metal in these areas. Similarly, patients with artificial heart
valves, metallic ear implants, bullet fragments, and chemotherapy or insulin pumps should not undergo MRI
scanning. During the MRI scan, the patient lies in a closed area inside the magnetic tube; some patients can
experience a claustrophobic sensation during the procedure. Therefore, patients with any history of
claustrophobia should relate this to the practitioner who is requesting the test, as well as the radiology staff. A
mild sedative can be given prior to the MRI scan to help alleviate this feeling.

All metallic objects on the body are removed prior to obtaining an MRI scan. Occasionally, patients will be
given a sedative medication to decrease anxiety and relax the patient during the MRI scan. MRI scanning
requires that the patient lies still for best accuracy; patients lie within a closed environment inside the magnetic
machine. Relaxation is important during the procedure and patients are asked to breathe normally. Interaction
with the MRI technologist is maintained throughout the test and there are loud, repetitive clicking noises which
occur during the test as the scanning proceeds. Occasionally, patients require injections of liquid intravenously
to enhance the images which are obtained. The MRI scanning time depends on the exact area of the body
studied, but ranges from half an hour to an hour and a half.

facebook.com/HonorsGroup
READING SUB-TEST

After the MRI scanning is completed, the computer generates visual images of the area of the body that was
scanned. These images can be transferred to film (hard copy). A radiologist is a physician who is specially
trained to interpret images of the body. The interpretation is transmitted in the form of a report to the
practitioner who requested the MRI scan. The practitioner can then discuss the results with the patient and/or
family.

Text 1: Questions 7-14

7 Yersiniosis occurs more commonly in;

Children

Americans

Adults

Teens

8 Symptoms such as fever and diarrhea may persist for about;

4 days

7 days

1-3 weeks

More than 3 weeks

facebook.com/HonorsGroup
READING SUB-TEST

9 Plague is caused by;

Y. pestis

Y. enterocolitica

Y. pseudotuberculosis

None

10 One of the following is not a common form of transmission of the parasite;

Contaminated unpasteurized milk

Blood transfusion

Raw or undercooked pork products

After contact with infected animals

11 Paragraph 4 talks about;

Prevention of the disease

Spread of the infection

How common the infection of Y. enterocolitica is

Reports by Active Surveillance Network.

12 Paragraph 5 talks about;

Diagnosis

Treatment

Spread of the infection and its control.

facebook.com/HonorsGroup
READING SUB-TEST

A&B

13 Which of these can be derived from Paragraph 5?

Treatment is not required for the infection caused by Y. enterocolitica.

Diarrhea requires no treatment

Complications which are mild can be resolved easily.

None

14 What is said about chitterlings?

Handling raw chitterlings with care is necessary

Contamination occurs more due to the careless handling chitterlings

Washing hands with soap before touching infants or their toys is vital

There should be separate caretakers to look after infants, while chitterlings are prepared

Text 2: Questions 15-22

15 According to paragraph 1, an image is produced;

When a signal is detected by the receiver portion of the scanner.

When the signal passes through the receiver, after coupling of the protons of the body.

When the signal is identified by the receiver which is accurately processed by a computer.

Only when the information, detected by the receiver is processed by a computer.

facebook.com/HonorsGroup
READING SUB-TEST

16 According to paragraph 1, what is the image like?

Crystal clear with no patches.

Self explanatory

Dense

Well enough to give a clear idea of the structures within the body.

17 According to paragraph 2, an MRI gives a clear idea about;

Brain aneurysms.

Strokes of the brain.

Brain tumours and spinal cord injury.

All

18 An MRI can;

Detect brain tumors.

Give a clear picture of spinal cord injury and other soft tissues related to it.

Give clear pictures to evaluate the structure of the heart and aorta.

All

facebook.com/HonorsGroup
READING SUB-TEST

19 Paragraph 3 talks about;

Detection of the diseases through an MRI scan.

Importance of taking an MRI scan.

Risks of an MRI scan.

None

20 Metallic materials mentioned in paragraph 3 include;

Surgical clips.

Artificial joints, metallic bone plates.

Only metal clips in or around the eyeballs.

A&B

21 Paragraph 4 talks about;

How an MRI is performed.

How a patient prepares for an MRI scan.

How a patient prepares for an MRI scan and how it is performed.

How an MRI scan is different from others.

facebook.com/HonorsGroup
READING SUB-TEST

22 According to paragraph 5, interpreting the images implies;

Identifying the disease through scanning.

Analysis of the disease.

Decoding the report.

All of the above.

facebook.com/HonorsGroup
READING SUB-TEST

ANSWERS

PART A

1. Correct Answer Is: C


2. Correct Answer Is: D
3. Correct Answer Is: C
4. Correct Answer Is: A
5. Correct Answer Is: D
6. Correct Answer Is: A
7. Correct Answer Is: B
8. Correct Answer Is: Hyde
9. Correct Answer Is: DeLuca
10. Correct Answer Is: Oxford Criteria
11. Correct Answer Is: Holmes
12. Correct Answer Is: DeLuca et al.
13. Correct Answer Is: Carruthers
14. Correct Answer Is: Bell
15. Correct Answer Is: Hyde
16. Correct Answer Is: Multiple Minor
17. Correct Answer Is: Fukuda Criteria
18. Correct Answer Is: Sudden Correct Answer Is: Acute Onset
19. Correct Answer Is: Hydes
20. Correct Answer Is: Primary Me

PART B & C

1. Correct Answer Is: A hemiplegic gait may include body asymmetry, decreased weight bearing on the
affected side.
2. Correct Answer Is: Postoperative care.
3. Correct Answer Is: has flared up more concerns.
4. Correct Answer Is: best exercise timing
5. Correct Answer Is: Lead to fatty change
6. Correct Answer Is: Use of gentamicin dose is significantly lower
7. Correct Answer Is: Children
8. Correct Answer Is: 1-3 weeks
9. Correct Answer Is: Y. pestis
10. Correct Answer Is: Blood transfusion
11. Correct Answer Is: How common the infection of Y. enterocolitica is
12. Correct Answer Is: A & B
13. Correct Answer Is: None
14. Correct Answer Is: Washing hands with soap before touching infants or their toys is vital
15. Correct Answer Is: Only when the information, detected by the receiver is processed by a computer.
16. Correct Answer Is: Dense
17. Correct Answer Is: All
18. Correct Answer Is: All
19. Correct Answer Is: Risks of an MRI scan.
20. Correct Answer Is: A & B
21. Correct Answer Is: How a patient prepares for an MRI scan and how it is performed.
22. Correct Answer Is: All of the above.

facebook.com/HonorsGroup
READING SUB-TEST

OET 2.0 READING TASK 5

Part A

TIME: 15 minutes

Look at the four texts, A-D, in the separate Text Booklet.

For each question, 1-20, look through the texts, A-D, to find the relevant information.

Write your answers on the spaces provided in this Question Paper.

Answer all the questions within the 15-minute time limit.

Your answers should be correctly spelt.

Text A

Systemic sclerosis (SSc)

Systemic sclerosis (SSc) is a disorder of the connective tissue characterized by fibrosis of the skin, vascular
abnormalities, and presence of autoantibodies. It is characterized by excessive deposition of extracellular
matrix. Therefore, there is significant heterogeneity in organ progression and prognosis. Interstitial lung
disease (ILD) is a heterogeneous group of parenchymal lung disorders that share common radiologic,
pathologic, and clinical manifestations. It is characterized by lung parenchyma damage, accompanied by
inflammation and fibrosis, and fibrosis is often incurable. The fibrosing forms of ILD are often incurable, and
are associated with significant morbidity and mortality. SSc is often accompanied by ILD. The incidence of
SSc-ILD in the relevant literature ranges from 45% to 90%. A recent European League Against Rheumatism
Scleroderma Trials and Research analysis revealed in a cohort of 3,656 SSc patients that ILD was present in
53% of cases with diffuse cutaneous SSc and in 35% of cases with limited cutaneous SSc.

facebook.com/HonorsGroup
READING SUB-TEST

Text B

NSIP is the more common subtype of inflammation in ILD. 77% of SSc-ILD is NSIP. A large number of
clinical and pathological studies have confirmed that a high-resolution CT (HRCT) pattern in patients is
correlated with pathologic NSIP and pathologic UIP. NSIP pattern is associated with better patient outcome
than UIP pattern. It includes reticular, frosted glass shadows, hollow, thickened honeycomb lung nodules,
emphysema, bronchial vascular bundles, bullae, traction bronchiectasis, cobblestone-like appearance,
bronchial tree, bronchiectasis and so on. The most common manifestation of NSIP is lobular reticular
formation in the pleural and basal regions. UIP is mainly represented by grid or honeycomb shadow. Different
patterns in HRCT can reflect NSIP and UIP. The extent of ILD lesions can be graded according to HRCT. At
present, the commonly used methods for clinical detection of ILD are HRCT, pulmonary function tests (PFTs)
(react as per sensitivity), bronchoalveolar lavage fluid (BALF), lung biopsy. HRCT has now become the most
common and sensitive imaging method for diagnosing ILD as it offers the most detailed images of the lungs.

Text C

KL-6

Krebs von den Lungen-6 (KL-6) is an important serum marker for ILD. It is a high molecular weight, mucin-
like glycoprotein secreted by type-II alveolar pneumocytes and bronchial epithelial cells in response to cellular
damage and regeneration in patients with ILD. KL-6 is a mucin-associated glycoprotein, which may be a
trigger for TGF-β signaling and fibrosis. The level of KL-6 as a predictive factor could be used to identify the
clinical development of ILD. Hideaki et al retrospectively analyzed the medical records of 29 patients with
SSc-ILD. They found serum KL-6 correlated positively with diffusing capacity of the lung for carbon
monoxide (DLCO)(% predicted) and disease extent on HRCT, and the changes in serum levels of KL-6 were
significantly related to the changes in forced vital capacity (FVC) in SSc -associated ILD. Their study suggests
KL-6 can be a useful monitoring tool of SSc-ILD activity.

facebook.com/HonorsGroup
READING SUB-TEST

Text D

SP-D

Surfactant, a lipoprotein complex, was originally described for its essential role in reducing surface tension at
the air-liquid interface of the lung. However, it is now recognized as being a critical component in lung
immune host defense. They include SP-B and SP-C and hydrophilic proteins SP-A and SP-D . SP-D levels are
more sensitive than SP-A in detecting ILD as defined by CT. The sensitivities and specificities for detecting
CT-positive ILD in 42 patients with SSc were 33% and 100% for SP-A and 77% and 83% for SP-D,
respectively. In a small but prospective study of 35 patients with SSc-ILD followed over 1-10 years, SP-D
levels were seen to definitely increase over time in 9 out of the 10 patients with worsening ILD, as defined by
changes in symptoms, lung function, and imaging, compared to mild increases in only 3 out of 25 patients with
stable or improving SSc-ILD. Therefore, SP-D is closely related to SSc-ILD. In addition, Takahashi H et al.
found a less-invasive and lung-specific clinical biomarker. They found the levels of SP-D in sera were
significantly higher in the CT-positive ILD group than in the CT-negative ILD group.

facebook.com/HonorsGroup
READING SUB-TEST

Questions 1-7

For each question, 1-7, decide which text (A, B, C or D) the information comes from. You may use any letter
more than once.

In which text can you find information about;

1 One of the major aspects of assessment of the how ILD may develop.

Answer

2 Detection or assessment of the ILD lesions.

Answer

3 Talk of naturally occurring molecule, gene, or characteristic by which a particular pathological or


physiological process, disease, etc. can be identified.

Answer

4 Common features of the disease.

Answer

5 Production of the protein in response to damage to cell.

Answer

facebook.com/HonorsGroup
READING SUB-TEST

6 SSc more commonly occurs with the ILD.

Answer

7 A more common form of interstitial lung disease.

Answer

Questions 8-14

Answer each of the questions, 8-14, with a word or short phrase from one of the texts. Each answer may
include words, numbers or both.

8 What are known to be more sensitive to change?

Answer

9 What is known to be closely related to SSc-ILD?

Answer

10 Which is an important serum marker for interstitial lung disease?

Answer

11 What are the major characteristics of UIP?

Answer

facebook.com/HonorsGroup
READING SUB-TEST

12 What is known to play a major role with respect to lung immune host defense?

Answer

13 What can be considered a perfect monitoring tool of SSc-ILD activity?

Answer

14 Which lipoprotein complex plays an essential role in reducing surface tension at the air-liquid interface of
the lung?

Answer

Questions 15-20

Complete each of the sentences, 15-20, with a word or short phrase from one of the texts. Each answer may
include words, numbers or both.

15 is a form of lung disease characterized by progressive scarring of both lungs

16 are recorded to have shown a remarkable increase with more deteriorating ILD conditions.

17 As per HRCT, it can be easy to analyze the extent of the

18 The sensitivities in detecting CT-positive ILD was SP-D.

19 The surfactant may include and hydrophilic proteins.

20 There are four major methods for clinical detection of

facebook.com/HonorsGroup
READING SUB-TEST

Part B

In this part of the test, there are six short extracts relating to the work of health professionals. For questions 1-
6, choose the answer (A, B or C) which you think fits best according to the text.

Obesity - Weight Loss

The relationship between obesity, weight loss, and disease control has been investigated among patients
already diagnosed with chronic diseases. Health benefits of weight loss in chronic diseases include
improvements in cholesterol levels, decreased risk of cardiovascular events, and improved quality of life.
Among patients with hypertension, patients that lost ≥5 kg had improved blood pressure. Weight loss of ≥5%
was associated with improved glycemic control in patients with diabetes mellitus. In patients with
osteoarthritis, weight loss of ≥5% was associated with improvement in joint symptoms. Weight loss of ≥5%
was also associated with low/remission disease activity in patients with psoriatic arthritis.

Hemophagocytic lymphohistiocytosis

Hemophagocytic lymphohistiocytosis (HLH) is the clinical manifestation of a wide array of different entities,
which include primary or familial hemophagocytic lymphohistiocytosis (FHLH) and secondary forms and can
lead to deteriorating conditions and eventually loss of proper body mechanism. The hallmark is
hemophagocytosis, appearance of activated macrophages that have engulfed other haematopoietic elements.
FHLH, mainly documented in early infancy, is related to familiar inheritance or genetic causes. Secondary
forms (SHLH), also called reactive HLH, are frequently diagnosed in adults and refer to cases with underlying

facebook.com/HonorsGroup
READING SUB-TEST

infection, malignancy or autoimmune disease. Over the last decade immunosuppression, immunodeficiency,
autoinflammatory diseases and inborn errors of metabolism have been also described as triggering diseases.
Macrophage activation syndrome (MAS) is a secondary form, recently reported in patients with autoimmune
or autoinflammatory diseases, especially patients with systemic juvenile idiopathic arthritis (SJIA),

Granulomatous– Lymphocytic Interstitial Lung Disease (GLILD)

Granulomatous– Lymphocytic Interstitial Lung Disease (GLILD is an inflammatory pulmonary complication


of common variable immunodeficiency (CVID) with distinctive patterns in the biopsy; granulomatous disease,
lymphocytic interstitial disease, follicular bronchiolitis and areas of organizing pneumonia. The
immunological data of the pathophysiology suggests that it is initiated by an infiltration of T as well as B
lymphocytes and macrophages, that will further lead the progression of the inflammatory process to fibrosis.
Regarding the treatment of this disease there are many immunomodulatory treatments with few standardized
protocols, but recent studies suggest that the combination of Rituximab and Azathioprine could be effective for
preserving the pulmonary function.

CCL2 is known as monocyte chemoattractant protein-1 (MCP-1). MCP-1 is a member of the C-C chemokines.
In vivo studies suggest that MCP-1 recruits monocytes/ macrophages to sites of inflammation in a wide variety
of pathological conditions, including ILD. The plasma level of CCL2 is correlated with FVC value in SSc.
However, there was no correlation between ILD severity and primary fibrotic genes such as collagen. That
might be because skin fibrosis peaks early during the course of SSc and improves later, while fibrosis in
pulmonary tissue continues to progress even at later stages of disease. A study examined serum levels,
spontaneous production by peripheral blood mononuclear cells (PBMC), and histological distribution in the
affected skin, of MCP-1. Elevated serum levels of MCP-1 significantly correlated with the presence of

facebook.com/HonorsGroup
READING SUB-TEST

pulmonary fibrosis. MCP-1 was expressed in mononuclear cells or vascular endothelial cells in 41% (9/22) of
SSc patients. The frequency of infiltrating mononuclear cells and endothelial cells that produced MCP-1 was
significantly higher in SSc patients with early onset than in SSc patients with late onset.

Accuracy of clinical decision making utilising FRAX without DXA

Clinical Decision

Correct Over treated OP missed

FRAX Hip 154 (84.6%) 25 (13.7%) 3 (1.6%)


70-74
FRAX MOF 154 (84.6%) 24 (13.2%) 4 (2.2%)

FRAX Hip 138 (90.2%) 11 (7.2%) 4 (2.6%)


75-79
FRAX MOF 136 (88.9%) 12 (7.8%) 5 (3.3%)

FRAX Hip 108 (87.8%) 10 (8.1%) 5 (4.1%)


80-84
FRAX MOF 108 (87.8%) 9 (7.3%) 6 (4.9%)

FRAX Hip 46 (63%) 14 (19.2%) 13 (17.8%)


85 & over
FRAX MOF 45 (61.6%) 13 (17.8%) 15 (20.5%)

FRAX Hip 446 (84%) 60 (11%) 25 (5%)


Total
FRAX MOF 443 (83%) 58 (11%) 30 (6%)

facebook.com/HonorsGroup
READING SUB-TEST

CXCL10 testing, 31 were RF positive and 11 were RF negative.

Antibody Positive Antibody Negative P-value

SSA 247.07 ± 146.30 (n=14) 221.91 ± 135.50 (n=28) 0.78

ANA 247.56 ± 139.76 (n=12) 223.49 ± 139.22 (n=30) 0.56

RF 253.14 ± 134.46 (n=31) 153.46 ± 128.09 (n=11) 0.04

CCP 244.02 ± 130.55 (n=26) 206.01 ± 152.86 (n=16) 0.22

facebook.com/HonorsGroup
READING SUB-TEST

1 What does this notice explain?

Effects of obesity.

Effects of weight loss.

How weight loss is correlated to other diseases.

2 Hemophagocytic lymphohistiocytosis.

Is a rare disease.

Is potentially fatal.

Occurs in all ages.

3 Granulomatous-“ Lymphocytic Interstitial Lung Disease;

Complication of common variable immunodeficiency disorders.

Can effectively be cured by using drugs that can bring in great change in immune response.

Appears majorly due to activity of T and B lymphocytes.

4 What is correct about MCP?

MCP-1 may play an important role in the development of pulmonary fibrosis in SSc.

The abnormal accumulation of macrophages will lead to the production of MCP-1

There is not much evidence to show that ILD is directly interlinked to collagen.

facebook.com/HonorsGroup
READING SUB-TEST

5 What information does this table provide?

Shows the clinical decision outcomes of the FRAX 10-year Hip and Major Osteoporotic fracture risk
score thresholds.

The thresholds based on the FRAX MOF risk score with DXA.

The thresholds based on the FRAX HF risk score with or without DXA.

6 The table clearly shows that;

there are high differences in serum CXCL10 concentration between SSA positive and SSA negative
subjects.

The RF-positive group had significantly elevated score.

The RF-positive group had an average CXCL10 concentration for the RF-negative group.

facebook.com/HonorsGroup
READING SUB-TEST

Part C

In this part of the test, there are two texts about different aspects of healthcare. For questions 7-22, choose the
answer (A, B, C or D) which you think fits best according to the text.

Text 1: Classification of Seizures

In order to communicate about types of seizures, epilepsy specialists have developed a classification system
for seizures. This system is not based on any fundamental property of seizures, but rather on committee-
generated conventions of terminology. Classification is as follows: partial seizures and generalized seizures.
Partial seizures are further divided into simple partial seizures with no alteration of consciousness or memory,
or complex partial seizures with alteration of consciousness or memory. Simple partial seizures can be motor
seizures with twitching, abnormal sensations, abnormal visions, sounds or smells, and distortions of
perception. Seizure activity can spread to the autonomic nervous system, resulting in flushing, tingling, or
nausea. If the patient becomes confused or cannot remember what is happening during the seizure, then the
seizure is classified as a complex partial seizure. Previously, they were called “psychomotor seizures”,
“temporal lobe seizures” or “limbic seizures”. During the complex partial seizure, patients may fumble or
perform automatic fragments of activity such as lip smacking, picking at their clothes, walking around
aimlessly, or saying nonsense phrases over and over again; these purposeless activities are called automatisms.
About 75% of people with complex partial seizures have automatisms; those who do not simply stop stare and
blank out for a few seconds or minutes.

Generalized seizures are divided into absence seizures and tonic-clonic seizures. Absence seizures were
previously called petit mal seizures and usually have onset in childhood, but they can persist into adulthood.
Absence seizures present with staring spells lasting several seconds, sometimes in conjunction with eyelid
fluttering or head nodding. These seizures can be difficult to distinguish from complex partial seizures that
may also result in staring. Usually, absence seizures are briefer and permit quicker recovery. Generalized
tonic-clonic seizures were previously called grand mal seizures; these seizures start with sudden loss of

facebook.com/HonorsGroup
READING SUB-TEST

consciousness and tonic activity (stiffening) followed by clonic activity (rhythmic jerking) of the limbs. The
patient’s eyes will roll up at the beginning of the seizure and the patient will typically emit a cry, not because
of pain, but because of contraction of the respiratory muscles against a closed throat. Generalized tonic-clonic
seizures usually last one to three minutes.

Seizures that begin focally can spread to the entire brain, in which case a tonic-clonic seizure ensues. It is
important, however, to distinguish those that are true grand mal, generalized from the start, from those that
start focally and secondarily generalize. Secondarily generalized seizures arise from a part of the brain that is
focally abnormal. Drugs used to treat primary and secondary generalized tonic-clonic seizures are different:
patients with secondarily generalized tonic-clonic seizures may be candidates for curative epilepsy surgery,
whereas primarily generalized tonic-clonic seizures are not surgical candidates, because there is no seizure
origin site (focus) to remove.

Atonic seizures are epileptic drop attacks. Atonic seizures typically occur in children or adults with widespread
brain injuries. People with atonic seizures suddenly become limp and may fall to the ground and football
helmets are sometimes required to protect against serious injuries. A myoclonic seizure is a brief un-sustained
jerk or series of jerks, less organized than the rhythmic jerks seen during a generalized tonic-clonic seizure.
Other specialized seizure types are occasionally encountered. Tonic seizures involve stiffening of muscles as
the primary seizure manifestation: arms or legs may extend forward or up into the air; consciousness may or
may not be lost. By definition, the clonic (jerking) phase is absent. Classification can be difficult, because
stiffening is a feature of many complex partial seizures. Tonic seizures, however, are much less common than
complex partial or tonic-clonic seizures. Patients can have more than one seizure type. One seizure type may
progress into another as the electrical activity spreads throughout the brain. A typical progression is from a
simple partial seizure, to a complex partial seizure (when the patient becomes confused), to a secondarily
generalized tonic-clonic seizure (when the electrical activity has spread throughout the entire brain). The brain
has control mechanisms to keep seizures localized. Antiepileptic medications enhance the ability of the brain
to limit the spread of a seizure.

facebook.com/HonorsGroup
READING SUB-TEST

Text 2: Fascioliasis Infection

Fascioliasis is a parasitic infection typically caused by Fasciola hepatica, which is also known as "the common
liver fluke" or "the sheep liver fluke." A related parasite, Fasciola gigantica, can also infect people. Fascioliasis
is found in all 5 continents, in over 50 countries, especially where sheep or cattle are reared. People usually
become infected by eating raw watercress or other water-based plants contaminated with immature parasite
larvae. The immature larval flukes migrate through the intestinal wall, the abdominal cavity, and the liver
tissue, into the bile ducts, where they develop into mature adult flukes, which produce eggs. Typically, the
pathology is most pronounced in the bile ducts and liver. A Fasciola infection is both treatable and
preventable.

The standard way to be sure a person is infected with Fasciola is by seeing the parasite - this is usually done by
finding Fasciola eggs in stool (fecal) specimens examined under a microscope. More than one specimen may
need to be examined to find the parasite. Sometimes eggs are found by examining duodenal contents or bile.
Infected people don't start passing eggs until they have been infected for several months; people don't pass
eggs during the acute phase of the infection. Therefore, early on, the infection has to be diagnosed in other
ways than by examining stool. Even during the chronic phase of infection, it can be difficult to find eggs in
stool specimens from people who have light infections.

Fasciola parasites develop into adult flukes in the bile ducts of infected mammals, which pass immature
Fasciola eggs in their feces. The next part of the life cycle occurs in freshwater. After several weeks, the eggs
hatch, producing a parasite form known as the miracidium, which then infects a snail host. Under optimal
conditions, the development process in the snail may be completed in 5 to 7 weeks; cercariae are then shed in
the water around the snail. The cercariae lose their tails when they encyst as metacercariae (infective larvae) on
water plants. In contrast to cercariae, metacercariae have a hard outer cyst wall and can survive for prolonged
periods in wet environments.

Immature Fasciola eggs are discharged in the biliary ducts and in the stool. Eggs become embryonated in
water; eggs release miracidia, which invade a suitable snail intermediate host, including the genera Galba,
fossaria and pseudosuccinea. In the snail the parasites undergo several developmental stages: sporocysts,
rediae, and cercariae. The cercariae are released from the snail and encyst as metacercariae on aquatic

facebook.com/HonorsGroup
READING SUB-TEST

vegetation or other surfaces. Mammals acquire the infection by eating vegetation containing metacercariae
whereas humans can become infected by ingesting metacercariae-containing freshwater plants, especially
watercress. After ingestion, the metacercariae excyst in the duodenum and migrate through the intestinal wall,
the peritoneal cavity, and the liver parenchyma into the biliary ducts, where they develop into adult flukes.

No vaccine is available to protect people against Fasciola infection. In some areas of the world where
Fascioliasis is found (endemic), special control programs are in place or are planned. The types of control
measures depend on the setting (such as epidemiologic, ecologic, and cultural factors). Strict control of the
growth and sale of watercress and other edible water plants is important. Individual people can protect
themselves by not eating raw watercress and other water plants, especially from endemic grazing areas. As
always, travelers to areas with poor sanitation should avoid food and water that might be contaminated
(tainted). Vegetables grown in fields that might have been irrigated with polluted water should be thoroughly
cooked, as should viscera from potentially infected animals.

In the early (acute) phase, symptoms can occur as a result of the parasite's migration from the intestine to and
through the liver. Symptoms can include gastrointestinal problems such as nausea, vomiting, and abdominal
pain/tenderness. In addition, fever, rashes and difficulty breathing may occur. During the chronic phase (after
the parasite settles in the bile ducts), the clinical manifestations may be similar or more discrete, reflecting
inflammation and blockage of bile ducts, which can be intermittent. Inflammation of the liver, gallbladder and
pancreas can also occur.

facebook.com/HonorsGroup
READING SUB-TEST

Text 1: Questions 7-14

7 Motor seizures are;

Simple partial seizures

Partial seizures

Complex seizures

Complex partial seizures

8 In which type of seizure does the patient generally not remember what is happening around
them?

Simple partial

Complex seizures

Complex partial seizures

Partial temporal lobe seizures

9 Which one of these activities are related to automatism?

Fumbling

Lip smacking

Speaking leisurely

None

facebook.com/HonorsGroup
READING SUB-TEST

10 Which seizures last for one to three minutes?

Simple partial seizures

Tonic-clonic seizures

Absence seizures

None

11 Which type of seizure occurs in childhood and may persist into adulthood?

Grand mal seizures

Petit mal seizures

Both A and B

None

12 Which seizures arise from a focally abnormal part of the brain?

Petit mal seizures

Grand mal seizures

Secondarily generalized seizures

Both B and C

facebook.com/HonorsGroup
READING SUB-TEST

13 As per the given information, who may undergo surgery?

Patients with grand mal seizures

Patients with secondarily generalized seizures

Patients with primarily generalized tonic-clonic seizures

Both B and C

14 Which one of the following statements correctly describes tonic seizures?

Rhythmic jerking

Stiffening of muscles

Loss of consciousness

None

Text 2: Questions 15-22

15 Which one of the following statements is correct?

Infection caused by Fasciola spreads faster than any other types of infections

Infection by Fasciola is deadly

Infection by Fasciola is treatable

Infection by Fasciola is very common

facebook.com/HonorsGroup
READING SUB-TEST

16 In which phase is it not easy to find the eggs in the stool?

Chronic phase

Infective phase

Acute phase

A and B

17 Paragraph 3 talks about which of the following;

Biology of Fasciola hepatica

Time period in a snail

Initial stages of the development of the parasite

Complete life cycle

18 Which of these forms survives for a longer period of time?

Cercariae

Metacercariae

Miracidia

Fasciola eggs

facebook.com/HonorsGroup
READING SUB-TEST

19 Which of these topics does paragraph 4 talk about?

How infection occurs in humans

How animals get infected

How plants get infected

All of the above

20 Excystation occurs in which of these?

Intestinal wall

Duodenum

Peritoneal cavity

Liver

21 Paragraph 5 talks about which of these topics?

Prevention and control

Availability of the treatment for the infection

Drugs to be used

A and C

facebook.com/HonorsGroup
READING SUB-TEST

22 Which of these topics does paragraph 6 talk about?

How infection is controlled

How infection spreads through bile ducts and liver

How infection is prevented from spreading to different parts

Possibility of infection spreading to other parts of the body

facebook.com/HonorsGroup
READING SUB-TEST

ANSWERS

PART A

1. Correct Answer Is: C


2. Correct Answer Is: B
3. Correct Answer Is: D
4. Correct Answer Is: A
5. Correct Answer Is: C
6. Correct Answer Is: A
7. Correct Answer Is: B
8. Correct Answer Is: HRCT and PFTs
9. Correct Answer Is: SP-D
10. Correct Answer Is: KL-6
11. Correct Answer Is: grid or honeycomb shadow
12. Correct Answer Is: Surfactant
13. Correct Answer Is: KL-6
14. Correct Answer Is: Surfactant
15. Correct Answer Is: Usual Interstitial Pneumonia (uip)
16. Correct Answer Is: Sp-d Levels
17. Correct Answer Is: Ild Lesions
18. Correct Answer Is: 83%
19. Correct Answer Is: Sp-b And Sp-c
20. Correct Answer Is: Ild

PART B & C

1. Correct Answer Is: How weight loss is correlated to other diseases.


2. Correct Answer Is: Is potentially fatal.
3. Correct Answer Is: Can effectively be cured by using drugs that can bring in great change in immune
response.
4. Correct Answer Is: MCP-1 may play an important role in the development of pulmonary fibrosis in SSc.
5. Correct Answer Is: Shows the clinical decision outcomes of the FRAX 10-year Hip and Major Osteoporotic
fracture risk score thresholds.
6. Correct Answer Is: The RF-positive group had an average CXCL10 concentration for the RF-negative
group.
7. Correct Answer Is: Simple partial seizures
8. Correct Answer Is: Complex partial seizures
9. Correct Answer Is: Lip smacking
10. Correct Answer Is: Tonic-clonic seizures
11. Correct Answer Is: Petit mal seizures
12. Correct Answer Is: Secondarily generalized seizures
13. Correct Answer Is: Patients with secondarily generalized seizures
14. Correct Answer Is: Loss of consciousness
15. Correct Answer Is: Infection by Fasciola is treatable
16. Correct Answer Is: Chronic phase
17. Correct Answer Is: Initial stages of the development of the parasite
18. Correct Answer Is: Metacercariae
19. Correct Answer Is: How animals get infected
20. Correct Answer Is: Duodenum
21. Correct Answer Is: Prevention and control
22. Correct Answer Is: Possibility of infection spreading to other parts of the body

facebook.com/HonorsGroup
READING SUB-TEST

OET 2.0 READING TASK 6

Part A

TIME: 15 minutes

Look at the four texts, A-D, in the separate Text Booklet.

For each question, 1-20, look through the texts, A-D, to find the relevant information.

Write your answers on the spaces provided in this Question Paper.

Answer all the questions within the 15-minute time limit.

Your answers should be correctly spelt.

Text A

Rheumatoid arthritis (RA) has wide variability in both its clinical presentation and its autoantibody profile.
Two well-known autoantibodies that are found in between 60-90% of RA patients are rheumatoid factor (RF)
and cyclic citrullinated peptide (CCP) antibody. Seropositivity for these antibodies is associated with more
destructive joint pathology and radiographic progression of RA. Anti-Sjogren’s Syndrome related Antigen A
(SSA) is associated with numerous autoimmune conditions, including most notably Sjogren’s Syndrome. Anti-
SSA is also found in between 3-16% of RA patients and it is believed to be a clinical indicator of poor
prognosis in RA. Several studies have shown that RA patients with this antibody have a lesser clinical
response to infliximab. Anti-SSA seropositivity is also associated with secondary Sjogren’s Syndrome. RA
with secondary Sjogren’s Syndrome is associated with worse clinical manifestations and increased antinuclear

facebook.com/HonorsGroup
READING SUB-TEST

antibody (ANA) positivity. The prevalence of ANA and anti-SSA has been shown to be higher in African
American (AA) RA patients compared to Caucasian (CAU) RA patients in two established RA cohorts.

Text B

In RA, inflammatory cytokines such as tumor necrosis factor-alpha (TNF-α) and interferon-gamma (IFN-γ)
are the primary inducers of chemokine production. Chemokines then lead to increased numbers of
inflammatory cells, such as macrophages, lymphocytes, and fibroblast-like synoviocytes, in inflamed synovial
tissue. Chemokines also contribute to cartilage degradation and pannus formation by stimulating the release of
various inflammatory cytokines. Several studies have shown that serum chemokines including CX3CL1,
CCL5, CXCL9, and CXCL10 are increased in active RA patients compared to healthy controls.. Particularly,
several studies have found that CXCL10 could serve as a disease activity marker in RA. Elevated CXCL10
and CXCL13 levels have been shown to be predictive of a favorable response to TNF inhibitor therapy.
Studies have also shown that serum chemokine levels, including CXCL9, CXCL10 and CXCL16 decrease
after treatment with disease-modifying antirheumatic drugs or biologic agents.

Text C

Previous studies have shown a wide variation in anti-SSA prevalence across different RA populations. It is
possible that the increased frequency of anti- SSA in AA subjects may be due to an increased frequency of
secondary Sjogren’s Syndrome. Co-existent RA and SS may then partially explain the increased disease
activity and worse clinical outcomes seen in AA RA patients. However, it was not possible to determine the
prevalence of Sjogren’s Syndrome in our cohort with the available data. The AA group also had a higher
prevalence of anti-SSB than the CAU group (4.26% vs. 1.08%). This was not a statistically significant
difference; however, there were only 5 total patients that were anti-SSB positive. The biological and clinical
implications of the increased prevalence of anti-SSA and ANA in AA RA patients are currently unknown.
However, several studies have suggested that autoantibody profiles may be clinically significant. Specifically,

facebook.com/HonorsGroup
READING SUB-TEST

anti-SSA has been shown to be associated with more severe disease in multiple connective tissue disease and it
is also involved in the molecular pathogenesis of immune dysregulation in Sjogren’s Syndrome.

Text D

In RA, a predominance of Th17 cytokines, including IFN-γ and TNF have been suggested to be of
pathological importance. IFN-γ induces several chemokines including CXCL9, CXCL10, and CXCL11.
Increased CXCL10 has been detected in the serum and synovial fluid of RA patients and in the saliva of
Sjogren’s Syndrome patients compared to healthy controls. Additionally, this chemokine may have clinical
significance as a human phase II clinical trial using an anti-CXCL10 monoclonal antibody (MDX-1100)
showed a significantly increased response rate in RA patients who had an inadequate response to methotrexate
therapy. The study revealed an association between RF seropositivity and increased CXCL10 levels but it
found no association between anti-SSA positivity and CXCL10. Therefore, while the increased clinical
severity seen in AA RA patients may be associated with a higher prevalence of anti-SSA, the presence of this
autoantibody does not appear to directly affect the expression of CXCL10.

facebook.com/HonorsGroup
READING SUB-TEST

Questions 1-7

For each question, 1-7, decide which text (A, B, C or D) the information comes from. You may use any letter
more than once.

In which text can you find information about;

1 Not much difference in data found.

Answer

2 Enhanced clinical response observed in AA RA patients.

Answer

3 doesn’t create a more effective response with respect to drug.

Answer

4 Their name is derived from their ability to induce directed chemotaxis in nearby responsive cells.

Answer

5 Despite differences in autoantibody prevalence, it is currently unknown if there is a higher. occurrence of


clinically diagnosed Sjogren’s Syndrome in AA RA patients.

Answer

facebook.com/HonorsGroup
READING SUB-TEST

6 The AA group is reported to have a large number of anti-SSB.

Answer

7 signaling protein involved in systemic inflammation.

Answer

Questions 8-14

Answer each of the questions, 8-14, with a word or short phrase from one of the texts. Each answer may
include words, numbers or both.

8 What is referred to as chemoattractant, which is induced by IFN-γ?

Answer

9 What is recommended to be of more importance pathologically?

Answer

10 What do previous studies indicate?

Answer

11 What is known to be more connected with the severity of multiple connective tissue disease?

Answer

facebook.com/HonorsGroup
READING SUB-TEST

12 What can work as a disease activity marker in RA?

Answer

13 What usually increases in RA patients?

Answer

14 What does the study suggest with respect to CXCL10?

Answer

Questions 15-20

Complete each of the sentences, 15-20, with a word or short phrase from one of the texts. Each answer may
include words, numbers or both.

15 levels often go down when target-specific drugs are used.

16 Anti-SSA is found in between of RA patients

17 can be defined as having or being a positive serum reaction especially in a test for the
presence of an antibody

18 Increased levels are known to be predictive of a favorable response to TNF inhibitor


therapy.

19 The reason for the increased frequency of can be directly linked to enhanced frequency of
secondary Sjogren’s Syndrome.

20 The research performed clearly indicate that there is no association between anti-SSA positivity

and

facebook.com/HonorsGroup
READING SUB-TEST

Part B

In this part of the test, there are six short extracts relating to the work of health professionals. For questions 1-
6, choose the answer (A, B or C) which you think fits best according to the text.

Important Steps - Dealing With Patients

1. Slow down. Communication can be improved by speaking slowly, and by spending just a small amount of
additional time with each patient. This will help foster a patient-centered approach to the clinician-patient
interaction. 2. Use plain, nonmedical language. Explain things to patients like you would explain them to your
grandmother. 3. Show or draw pictures. Visual images can improve the patient’s recall of ideas. 4. Limit the
amount of information provided— and repeat it. Information is best remembered when it is given in small
pieces that are pertinent to the tasks at hand. Repetition further enhances recall. 5. Use the “teach-back”
technique. Confirm that patients understand by asking them to repeat back your instructions. 6. Create a
shame-free environment: Encourage questions. Make patients feel comfortable asking questions. Consider
using the Ask-Me-3 program. Enlist the aid of others (patient’s family or friends) to promote understanding.

Duodenal duplication

Duodenal duplication is an extremely rare pathology. It represents 4 % of all digestive tract congenital
malformations. It is often connected with intestinal malrotation, scalloped vertebras). In one of the cases, the
degeneration arose on mucous membrane of gastric type within the cyst of duplication and in the second it was
duodenal mucous membrane. The diagnosis of cancer was made in both cases on the surgical pieces at
anatomopathology. There had been no biopsy within the cyst. This malformation appears in 70% of the cases

facebook.com/HonorsGroup
READING SUB-TEST

before one year of the age but the late revelation is possible. The digestive obstruction is the most frequent
mode of revelation. Acute pancreatitis was reported and sometimes the diagnosis can be delayed many years.
The duodenal duplication, the anomaly of the embryogenesis, is diagnosed most of the time in the childhood,
even by prenatal diagnosis. The average age at the time of the diagnosis is from four months to nine years.
Prevalence is lightly in favour of the male.

Anti-reabsorption medications

Presently, anti-reabsorption medications are most widely used for treating osteoporosis. Zoledronic acid
(Aclasta) is a common clinical anti-reabsorption medication. As the third generation of bisphosphonates (BPs),
it outperforms the previous nitrogen-containing BPs in improving patients’ balance and quality of life by
inhibiting bone resorption and increasing bone mineral density (BMD). It is administered via intravenous drip
infusion once a year, which, therefore, brings good compliance with treatment. However, those who have been
treated with zoledronic acid intravenously are likely to suffer from acute side effects, such as fever, bone and
joint pain and flu-like symptoms, especially after the first administration. Although the common adverse
reactions generally disappear within 72 h, or in rare cases, last 7 to 10 days without recurrence, they have
serious impacts on the aging population, especially those with underlying diseases. Thus, the patients who are
afraid of any adverse reactions or have suffered from any side effects during the first administration may show
poor compliance with the second dose and refuse the clinical application of zoledronic acid.

Cancer - Brief Overview

Historically, individuals with cancer have rarely survived past the latency periods inherent to the development
of treatment-induced malignancies, but improvements in systemic and radiation therapy have increased the
time for these second primaries to emerge. In the past 15 years, advances in cancer diagnostics, therapy and

facebook.com/HonorsGroup
READING SUB-TEST

supportive care have significantly increased disease-free survival and overall survival rates in cancer patients.
A proper care plan is reported to have increased 10 or more years beyond their initial diagnosis and introduced
the potential for secondary cancers induced by therapy. It is possible that the incidence of treatment-induced
cancers may increase in the future. Long-term toxicity of cancer treatments both from systemic chemotherapy
and radiation therapy become an important survivorship concern for patients and their physicians.

The table shows the distribution of the characteristics of the surgery performed.

Evaluated Factor n % p-value¹

Length of hospital stay

4 days 16 30.8

5 days 18 34.6 0.926

6 or more 18 34.6

Minimum-maximum 4 - 51 -

Average ± Standard
7.2 ± 8.5 -
Deviation

Type of surgery

TKA-•right knee 38 47.5


0.655
TKA-•left knee 42 52.5

Duration of surgery (in


hours)

facebook.com/HonorsGroup
READING SUB-TEST

Up to 2 hours 15 18.5

Between 2 and 3 hours 60 74.1 <0.001

Above 3 hours 6 7.4

Minimum-maximum 1.6 - 3.3 -

Average ± Standard
2.4 ± 0.4 -
Deviation

Distribution of the personal profile of evaluated patients.

Factor evaluated n % p-value¹

Gender

Male 17 21.0
<0.001
Female 64 79.0

Age

Less than 60 years 19 23.5

60 to 70 years 38 46.9 0.028

Over 70 years 24 29.6

facebook.com/HonorsGroup
READING SUB-TEST

1 The manual talks about;

Steps to improving interpersonal communication with patients.

How to effectively deal with the patients.

Steps with regard to clinician-patient communication.

2 Duodenal duplication;

is associated with various anomalies.

can occur in two different ways.

Is a benign congenital defect, acquired during the embryonic development of the digestive tract

3 Anti-reabsorption medications;

Comprised of agents which limit the rate of bone loss.

Decrease the rate at which osteoclasts resorb bone.

Can have a detrimental effect on elderly people.

4 The given notice talks about;

How cancer has successfully been dealt with in today’s advanced scientific world.

Effective cancer treatment has resulted in many cancer patients.

facebook.com/HonorsGroup
READING SUB-TEST

Future course of action.

5 What is correct about the surgery performed?

On average, hospitalization was 7 days and surgery time was 2 hours.

The majority of patients remained hospitalized for 5 days or more

performed right knee arthroplasty (TRKA) more on males than females.

6 The table clearly shows that;

The majority of the patients were male.

People who are aged 60-70 are more in number.

The majority of the patients were females.

facebook.com/HonorsGroup
READING SUB-TEST

Part C

In this part of the test, there are two texts about different aspects of healthcare. For questions 7-22, choose the
answer (A, B, C or D) which you think fits best according to the text.

Text 1: Paget's Disease

Paget's disease of the bone is an unusual, chronic bone condition that occurs in only about 1% of people in the
United States and slightly more often in men than in women (3 to 2). Individuals with Paget's disease
experience rapid bone repair, which causes a variety of symptoms from softer bones to enlarged bone growth,
typically in the pelvis, lower back (spine), hips, thighs, head (skull) and arms. Medical therapies have proven
effective in reducing the frequency of pain, fractures and arthritis that may be caused by this condition. Paget's
disease typically occurs in the older population and usually only in a few of their bones. Bones become large
and soft, leading to problems such as bending, breaking, pinched nerves, arthritis and reduced hearing.
Effective and safe treatment methods can help most people with Paget's disease.

Normally, as people age, their bones rebuild at a slower rate. For those with Paget's disease, however, this
process of rebuilding bones takes place at a faster rate. As a result, the rebuilt bone has an abnormal structure.
The involved bone can be soft, leading to weakness and bending of the pelvis, lower back (spine), hips, thighs,
head and arms. Or, the rebuilt bone can enlarge, making it more susceptible to arthritis, hearing loss, fractures
and discomfort. Given that this takes place in those over the age of 40, the symptoms are often mistaken for
changes associated with aging.

The cause of Paget's disease is unknown. It does appear to be, at least partially, hereditary, perhaps when
activated by exposure to a virus. Indicative of the hereditary consideration: Paget's disease occurs more
commonly in European populations and their descendants. In 30% of cases, disease incidents often involve
more than one member of a family. Paget's is rarely discovered in individuals before they reach the age of 40,
and the number of people identified increases in each progressive age group. Typically, it is the appearance of
the bones on an X-ray that prompts the physician to make the diagnosis. Blood tests taken most often will
indicate an increase in serum alkaline phosphatase (SAP), which is reflective of the rapid new bone turnover.
Urine test results will also indicate the speed at which this rebuilding is taking place. Physicians usually obtain

facebook.com/HonorsGroup
READING SUB-TEST

a non-invasive bone scan to determine the extent of bone involvement. Only if cancer is suspected will it be
necessary to do a bone biopsy to examine it under a microscope.

Treatment approaches can focus on providing physical assistance, including the addition of wedges in the shoe,
canes as walking aids and the administration of physical therapy (this is considered to be the best).
Medications that help reduce the pain associated with Paget's include acetaminophen (e.g. Tylenol) and anti-
inflammatory drugs such as ibuprofen and naproxen. In addition, a group of medications called
bisphosphonates reduce the pain and help the body regulate the bone-building process to stimulate more
normal bone growth. Your physician may prescribe an oral medication such as Alendronate (Fosamax) or
etidronate (Didronel) to be taken orally every day for 6 months; Tiludronate (Skelid) to be taken orally every
day for 3 months, Risedronate (Actonel) to be taken orally every day for 2 months. All oral medications should
be taken with a large glass of water (6-8 oz) upon rising in the morning. Patients should remain upright for the
next 30 minutes and not eat until that time has passed. Any of these treatments can be repeated if necessary.
Side effects of these medicines may involve heartburn and sometimes an increase in bone pain for a short
period of time.

There are also injectable medications that can be given to a patient for Paget's which include Pamidronate
(Aredia), which is injected into the vein once a month or once every few months. The injection takes a few
hours. Unusually, there is inflammation of the eye or loss of bone around the teeth (osteonecrosis);
Zoledronate (Reclast), which is injected in the vein once a year. The injection takes less than 30 minutes;
Calcitonin, a hormone that is injected under the skin several times a week. Surgery for arthritis caused by
Paget's disease is effective in reducing pain and improving function. Medical treatment is not expected to
correct some of the changes of the Paget's disease that have already occurred, such as hearing loss, deformity
or osteoarthritis.

Text 2: Pancreatic Cancer

Cancer is a class of diseases characterized by out-of-control cell growth, and pancreatic cancer occurs when
this uncontrolled cell growth begins in the pancreas. Rather than developing into healthy, normal pancreas
tissue, these abnormal cells continue dividing and form lumps or masses of tissue called tumors. Tumors then
interfere with the main functions of the pancreas. If a tumor stays in one spot and demonstrates limited growth,

facebook.com/HonorsGroup
READING SUB-TEST

it is generally considered to be benign. More dangerous, or malignant, tumors form when the cancer cells
migrate to other parts of the body through the blood or lymph systems. When a tumor successfully spreads to
other parts of the body and grows, invading and destroying other healthy tissues, it is said to have
metastasized. This process itself is called metastasis, and the result is a more serious condition that is very
difficult to treat. In the United States each year, over 30,000 people are diagnosed with pancreatic cancer.
Europe sees more than 60,000 diagnosed each year. In Asian countries, numbers can be even higher than that.

Pancreatic cancer is categorized depending on whether it affects the exocrine or endocrine functions of the
pancreas. There is an important distinction between the two broad types of pancreatic cancer because they have
different risk factors, causes, symptoms, diagnostic tests, treatments, and prognosis. Tumors that affect the
exocrine functions are the most common type of pancreatic cancer. Sometimes these tumors or cysts are
benign, called cystadenomas. However, it is more likely to find malignant tumors called adenocarcinomas,
which account for 95% of exocrine pancreatic cancers. Adenocarcinomas typically start in gland cells in the
ducts of the pancreas, but they can also arise from pancreatic enzyme cells (acinar cell carcinoma). Other types
of pancreatic cancers that are associated with exocrine functions include adenosquamous carcinomas,
squamous cell carcinomas, and giant cell carcinomas, named for their appearances underneath a microscope.
There is also a disease called ampullary cancer (carcinoma of the ampulla of Vater) that starts where the bile
duct and pancreatic duct meet the duodenum of the small intestine.

Cancer is ultimately the result of cells that grow uncontrollably and do not die. Normal cells in the body follow
an orderly path of growth, division, and death. Programmed cell death is called apoptosis, and when this
process breaks down, cancer occurs. Pancreatic cancer cells do not experience programmatic death, but
instead, continue to grow and divide. Although scientists do not know exactly what causes these cells to
behave in this way, they have identified several potential risk factors. Cancer can be the result of a genetic
predisposition that is inherited from family members. It is possible to be born with certain genetic mutations or
a fault in a gene that makes one statistically more likely to develop cancer later in life.

About 10% of pancreatic cancers are thought to be caused by inherited gene mutations. Genetic syndromes that
are associated with pancreatic cancer include hereditary breast and ovarian cancer syndrome, melanoma,
pancreatitis, and non-polyposis colorectal cancer (Lynch syndrome).

Carcinogens are a class of substances that are directly responsible for damaging DNA, promoting or aiding
cancer. Certain pesticides (dyes may also be included in this list here), and chemicals used in purification of

facebook.com/HonorsGroup
READING SUB-TEST

the metal are thought to be carcinogenic, increasing the risk of developing pancreatic cancer. When our bodies
are exposed to carcinogens, free radicals have formed that try to steal electrons from other molecules in the
body. These free radicals damage cells, affecting their ability to function normally, and the result can be
cancerous growths. As we age, there is an increase in the number of possible cancer-causing mutations in our
DNA. This makes age an important risk factor for pancreatic cancer, especially for those over the age of 60.
There are several other diseases that have been associated with an increased risk of cancer of the pancreas.
These include cirrhosis or scarring of the liver, Helicobacter pylori infection (infection of the stomach with the
ulcer-causing bacteria H. pylori), diabetes mellitus, chronic pancreatitis (inflammation of the pancreas), and
gingivitis or periodontal disease.

In order to detect pancreatic cancer, physicians will request a complete physical examination as well as
personal and family medical histories. The way in which cancer presents itself will differ depending on
whether the tumor is in the head or the tail of the pancreas. Tail tumors present with pain and weight loss while
head tumors present with steatorrhea, weight loss, and jaundice. Doctors also look for recent onset of atypical
diabetes mellitus, Trousseau's sign, and recent pancreatitis. In general, when making a pancreatic cancer
diagnosis, physicians pay special attention to common symptoms such as abdominal or back pain, weight loss,
poor appetite, tiredness, irritability, digestive problems, gallbladder enlargement, blood clots (deep venous
thrombosis (DVT) or pulmonary embolism), fatty tissue abnormalities, diabetes, swelling of lymph nodes,
diarrhea, steatorrhea, and jaundice.

facebook.com/HonorsGroup
READING SUB-TEST

Text 1: Questions 7-14

7 Pagets disease;

Affects more women than men;

Affects 3 men in 4.

Affects 3 women in 4.

Affects a higher number of men than women.

8 Pagets disease can lead to;

Bone disorder.

Softening of bones.

Arthritis in its final stage.

None of the above.

9 Pagets disease is a condition in which;

Bones become large and soft.

Rebuilding of bones is stopped.

Pelvis bends at a faster rate.

Rebuilding of bone is accelerated.

10 Reformation of the bones can lead to;

facebook.com/HonorsGroup
READING SUB-TEST

Lower back pain, loss of hearing and discomfort.

Arthritis, loss of hearing and softening of bone tissues.

Fractures and discomfort only.

All of the above.

11 Which of the following statements is right as per the given information in the passage?

If one member of the family is affected with Pagets disease then other members will also be affected.

If one member of the family is known to have Pagets disease then others can also be affected.

Pagets disease is highly hereditary.

Pagets disease, is both heritable and inheritable.

12 Pick the correct statement as per the given information in the passage;

Those who reach the age of 40 shall undergo blood tests and urine tests for the identification of Pagets
disease.

Physicians should always advise the patients to go for a non-invasive bone scan.

Increase is indicative of the development of the bone at a rapid speed.

It is necessary to do a biopsy of the bone to understand the nature of the disease.

13 What is right about oral medications?

facebook.com/HonorsGroup
READING SUB-TEST

May increase bone pain.

Should be taken only during the morning.

May disturb physiological function.

All of the above.

14 Which of the following statements is incorrect as per the given information in the passage?

Pagets disease is a condition which can lead to arthritis.

Surgery can get rid of Pagets disease.

Surgery of arthritis which occurred due to Pagets disease can only reduce the pain associated with it.

Medical treatment will not correct a loss in hearing or deformity or osteoarthritis which resulted from it.

Text 2: Questions 15-22

15 Pancreatic tumors can be;

Benign

Malignant

Benign and malignant

None

facebook.com/HonorsGroup
READING SUB-TEST

16 Metastasized is a condition

When pancreatic tumors spread to other parts of the body.

When malignant tumors form in the pancreas.

When harmful tumors invade and destroy other healthy tissues of the body.

When tumors are grown automatically in other parts of the body.

17 Pancreatic cancer is most commonly associated with.

Endocrine gland

Exocrine gland

Cystadenomas

Adenocarcinomas

18 Ampullary cancer is associated with.

Endocrine gland

Exocrine gland

Pancreatic duct

None

19 Failure of apoptosis results in;

Pancreatic cell growth

Enlargement of the pancreatic duct

Cancerous tissues in the pancreas

facebook.com/HonorsGroup
READING SUB-TEST

B&C

20 A cancerous growth in the pancreas is a result of;

Specific carcinogens

Family Genes

Genetic mutations

None

21 The risk of pancreatic cancer is associated with these carcinogens;

Pesticides and dyes

Pesticides, dyes and chemicals used for refining metals

Only dyes

None

22 One of these is not a symptom associated with PC;

Back pain and problems with digestion

Digestive problems and blood clotting

Pulmonary edema and enlargement of the gallbladder

Jaundice and modification in the lymph

facebook.com/HonorsGroup
READING SUB-TEST

ANSWERS

PART A

1. Correct Answer Is: C


2. Correct Answer Is: D
3. Correct Answer Is: A
4. Correct Answer Is: B
5. Correct Answer Is: A
6. Correct Answer Is: C
7. Correct Answer Is: B
8. Correct Answer Is: CXCL9
9. Correct Answer Is: Th17 cytokines
10. Correct Answer Is: variation in anti-SSA prevalence
11. Correct Answer Is: Anti SSA
12. Correct Answer Is: CXCL10
13. Correct Answer Is: serum chemokines
14. Correct Answer Is: relation between RF seropositivity & CXCL10 levels
15. Correct Answer Is: Serum Chemokine
16. Correct Answer Is: 3-16%
17. Correct Answer Is: Seropositivity
18. Correct Answer Is: Cxcl10 And Cxcl13
19. Correct Answer Is: Anti- Ssa In Aa
20. Correct Answer Is: Cxcl10

PART B & C

1. Correct Answer Is: Steps to improving interpersonal communication with patients.


2. Correct Answer Is: is associated with various anomalies.
3. Correct Answer Is: Can have a detrimental effect on elderly people.
4. Correct Answer Is: Future course of action.
5. Correct Answer Is: The majority of patients remained hospitalized for 5 days or more
6. Correct Answer Is: The majority of the patients were females.
7. Correct Answer Is: Affects a higher number of men than women.
8. Correct Answer Is: Arthritis in its final stage.
9. Correct Answer Is: Rebuilding of bone is accelerated.
10. Correct Answer Is: Lower back pain, loss of hearing and discomfort.
11. Correct Answer Is: Pagets disease, is both heritable and inheritable.
12. Correct Answer Is: Increase is indicative of the development of the bone at a rapid speed.
13. Correct Answer Is: Should be taken only during the morning.
14. Correct Answer Is: Surgery can get rid of Pagets disease.
15. Correct Answer Is: Benign and malignant
16. Correct Answer Is: When harmful tumors invade and destroy other healthy tissues of the body.
17. Correct Answer Is: Exocrine gland
18. Correct Answer Is: Exocrine gland
19. Correct Answer Is: Cancerous tissues in the pancreas
20. Correct Answer Is: Genetic mutations
21. Correct Answer Is: Pesticides, dyes and chemicals used for refining metals
22. Correct Answer Is: Pulmonary edema and enlargement of the gallbladder

facebook.com/HonorsGroup
READING SUB-TEST

OET 2.0 READING TASK 7

Part A

TIME: 15 minutes

Look at the four texts, A-D, in the separate Text Booklet.

For each question, 1-20, look through the texts, A-D, to find the relevant information.

Write your answers on the spaces provided in this Question Paper.

Answer all the questions within the 15-minute time limit.

Your answers should be correctly spelt.

Text A

Galectin-3

It is well known that there is a close relation between obesity-induced insulin resistance, immune cells
accumulation in white adipose tissue (WAT) and inflammation. Indeed, in obesity WAT is characterized by an
increased production and secretion of a wide range of inflammatory cytokines including TNF-alpha and
interleukin (IL)- 6, which may have local effects on endothelial, vasculature and target adipose tissues.
Therefore, activated macrophages and other antigen presenting cells that are accumulated in elevated number
in fat tissue in both types of obese actively secrete a broad spectrum of locally produced pro-inflammatory
cytokines including galectin-3 (Gal-3). Gal-3 is a beta-galactoside-binding lectin belonging to a
multifunctional protein family, which enhances chemotaxis of immune and antigen presenting cells, reduces
insulin-stimulated glucose uptake in myocytes and adipocytes and impairs insulin-mediated suppression of
glucose output in hepatocytes. Gal-3 may bind directly to the insulin receptor (IR) and thereby inhibit

facebook.com/HonorsGroup
READING SUB-TEST

downstream insulin resistance signaling via diminishing interleukin-1 beta production. Therefore, Gal- 3 is a
modulator of apoptosis, necrosis and fibrosis associated with extracellular remodeling.

Text B

Gal-3 is increased in obesity and mediates inflammation and fibrosis in the heart and vessels, as well as in the
WAT. The most preclinical and clinical studies suggest that this protein protects from inflammation in obese,
while there is a large body of evidence regarding the ability of Gal-3 to deteriorate glucose homeostasis,
modulate cell adhesion and induce pro-oxidant pathways. Interestingly, the low serum Gal-3 concentrations
are closely associated with insulin resistance in patients with type 2 diabetes mellitus. In contrast, an inverse
correlation between serum Gal-3 and glycosylated hemoglobin in type 2 diabetes mellitus was found. In
clinical settings Gal-3 strongly independently predicts all-cause mortality and CV mortality in the general
population and in patients with known CV disease. In fact, in cross-sectional analyses of 2946 Framingham
Heart Study participants circulating Gal-3 was associated well with abdominal adiposity, dyslipidemia, and
hypertension, but Gal-3 did not predict incident CV and metabolic diseases after adjusting for cardiometabolic
risk factors. Whether Gal-3 could be a predictive marker of the metabolically unhealthy obese is not clear,
although Gal-3 deserves further large clinical trials to understand its role in different obese phenotypes’
development.

Text C

Natriuretic peptides (NPs) are “cardiometabolic” hormones with well-established cardiovascular, renal, and
endocrine abilities affecting sodium reabsorption and blood pressure regulation. Although, NP levels are
markers of biomechanical cardiac stress, their role in the nature of the evolution of obese is not fully
understood. These controversies affect the clearance of NPs in obese and pathophysiological mechanisms
controlling the synthesis of them. While, NP secretion result in a stretch of the cardiac wall and volume
overload of cardiac cavities. On the other hand, recent epidemiological and preclinical/ clinical studies have
shown that the NP system acts as a deficiency in obese patients is due to worse clearance of NP receptors and

facebook.com/HonorsGroup
READING SUB-TEST

neutral endopeptidases. Consequently, NP system in obese is not able to mediate a wide spectrum of
cardiovascular and metabolic protective effects (i.e., vasodilation, natriuresis, diuresis, lipolysis, weight loss,
lusitropy, lipid peroxidation, and also improve mitochondrial respiration and insulin sensitivity).

Text D

Brutsaert et al. have reported that higher levels of brain NP have associated with decreased risk of diabetes in
middle-aged adults and that the interrelation has remained after adjustment for waist circumference, low
physical activity, estimated glomerular filtration rate and high sensitive C-reactive protein level. In contrast, it
is suggested that the low brain NP levels observed in obesity could causally associated with the incidence of
diabetes in obese individuals. The effect of brain NPs might relate to an ability of natriuretic peptides to
activate a thermogenic program in brown and white fat tissues, increase energy expenditure and inhibit food
intake. Thus, NPs might play several metabolic roles in the development of different phenotypes of obesity,
but their predictive role in CV disease development in obese patients is uncertain.

facebook.com/HonorsGroup
READING SUB-TEST

Questions 1-7

For each question, 1-7, decide which text (A, B, C or D) the information comes from. You may use any letter
more than once.

In which text can you find information about;

1 Role of this metabolic hormones is not yet certain.

Answer

2 Perform its role when there is a deficiency.

Answer

3 Higher and lower level of the metabolic hormones.

Answer

4 Known to affect the interior surface of blood vessels and lymphatic vessels.

Answer

5 Worsening relative sugar equilibrium.

Answer

6 Introduction to carbohydrate-binding lectin whose expression is associated with inflammatory cells.

facebook.com/HonorsGroup
READING SUB-TEST

Answer

7 May have a direct impact on hemoglobin to which glucose is bound.

Answer

Questions 8-14

Answer each of the questions, 8-14, with a word or short phrase from one of the texts. Each answer may
include words, numbers or both.

8 What defines the rate of myocardial relaxation?

Answer

9 What are more associated with insulin resistance?

Answer

10 What can have the potential to have direct impact on insulin resistance?

Answer

11 What defines an abnormal amount of lipids?

Answer

12 What are the markers of biomechanical cardiac stress?

facebook.com/HonorsGroup
READING SUB-TEST

Answer

13 What is associated with programmed cell death?

Answer

14 What causes the cardiac wall to stretch and volume overload of cardiac cavities?

Answer

Questions 15-20

Complete each of the sentences, 15-20, with a word or short phrase from one of the texts. Each answer may
include words, numbers or both.

15 can be associated with a significantly increased risk for coronary heart disease.

16 higher levels of brain NP can be connected with the decreased risk of

17 The term is referred to the thickening and scarring of connective tissue, usually as a result
of an injury.

18 It is still not clear that Gal-3 is a of the metabolically unhealthy obese

19 Excretion of sodium in the urine is a common condition and is called

20 NPs may not mediate of lipids.

facebook.com/HonorsGroup
READING SUB-TEST

Part B

In this part of the test, there are six short extracts relating to the work of health professionals. For questions 1-
6, choose the answer (A, B or C) which you think fits best according to the text.

Liver toxicity from sulfasalazine

Liver toxicity from sulfasalazine is a rare but serious side effect. It can range from mild elevation in LFTs to
hepatic failure and cirrhosis. The occurrence of severe liver toxicity such as acute hepatitis as seen in our
patient is <1%. After reaching the gut, sulfasalazine is broken down by the colonic bacteria into its
metabolites, i.e., sulfapyridine and 5-aminosalicylic acid. Sulfapyridine is absorbed in the gut and eliminated
after acetylation by enzyme N-acetyltransferase which can have variable activity based on the patient's
genotype. Patients who have genotypes for slow acetylation are found to be more predisposed to sulfasalazine-
induced liver toxicity. Injury can be hepatocellular which presents with disproportionate elevation in serum
aminotransferases or cholestasis which presents with disproportionate elevation in alkaline phosphatase. Both
patterns of liver injury can have elevation in bilirubin and abnormal tests for liver synthetic function.

Adiponectin

Adiponectin accounts for 0.01% of plasma protein with a half-life of 2.5 h; normal adiponectin plasma level is
5–10 μg/mL with higher levels in females than males due to sexual dimorphism. Adiponectin plasma forms are
of two types, high-molecular-weight and low-molecular-weight. In addition, high-molecular-weight
adiponectin levels are positively associated with CAD and negatively associated with risk of type 2 diabetes
mellitus (DM), but this is not true to the low-molecular-weight adiponectin. Adiponectin serum levels are
inversely correlated with body mass index (BMI), visceral obesity, and insulin resistance (IR); thus, it is
regarded as an indicator and predictor of noninsulin dependent DM, insulin resistant, and overt hyperglycemia.

facebook.com/HonorsGroup
READING SUB-TEST

NF‘induced pulmonary toxicity

NF‘induced pulmonary toxicity can be seen in three different forms, that is, due to acute, subacute or chronic
reaction. The acute form is the most common. Side effects occurring up to the 1st month of receiving the first
treatment are classified as the acute form. Acute form develops secondary to hypersensitivity reaction with
peripheral demonstrating eosinophilia and thoracic CT showing ground‘glass opacity. Chronic NF‘induced
lung disease is seen predominantly in older women who present with respiratory symptoms after a year or
more of NF therapy. Characteristic pathologic finding in chronic NF pulmonary toxicity is diffuse interstitial
fibrosis, vascular sclerosis, fibrosis, and thickening of the alveolar septa, interstitial inflammation, and
bronchiolitis obliterans with organising pneumonia.

Case Study

A 46-year-old male with a past medical history of schizoaffective disorder and chronic lower back pain, was
admitted for management of worsening depression and anxiety. He was started on gabapentin, 300mg twice
daily for his back pain and anxiety symptoms. His only other medication was hydrocodone. Over the next few
days, he started developing worsening bilateral lower extremity edema. He did not have any cardiovascular-
related symptoms. Physical exam was only significant for 3+ pitting edema with all laboratory values and
imaging being unremarkable. Gabapentin was discontinued and his lower extremity swelling improved over
subsequent days. The incidence of pedal edema with gabapentin use is approximately 7 to 7.5% with all
studies being in elderly patients receiving doses above 1200 mg/day. This case illustrates that lower doses of
gabapentin can also cause this adverse effect. It is important to recognize this adverse effect because
gabapentin is used in conditions like diabetic neuropathy, which is associated with multiple comorbidities that

facebook.com/HonorsGroup
READING SUB-TEST

can give rise to bilateral leg swelling. Presence of gabapentin-induced leg swelling can thus confound the
clinical picture.

Two hundred and six of the 250 pharmacists completed the questionnaires, giving a response rate of
82.4%. The mean age of pharmacists was approximately 32 years with a majority in the young adult age group
(20-50 years, 92.22%).

Sociodemographic parameter Respondents, N=206(%)

Age (yr) Mean (SD) 36

Gender
128
Male
78
Female

Education qualification
164
Graduate(B. Pharm, D. Pharm)
42
Postgraduate(M. Pharm)

Professional status
142
Practicing Pharmacists
45
Working professional
10
Academicians

facebook.com/HonorsGroup
READING SUB-TEST

Working professional (involved in the manufacture, marketing, distributors, research of drugs)

Demographic data of patients

Number (%)

Age

0-28 days 7 (2.4)

29 days to 12 months 82 (27.9)

13-24 months 85 (28.9)

25-36 months 40 (13.6)

37-48 months 26 (8.8)

49-60 months 54 (18.4)

Sex

Male 172 (58.5)

Female 122 (41.5)

facebook.com/HonorsGroup
READING SUB-TEST

1 What is correct about liver toxicity from sulfasalazine?

Hepatotoxicity can arise either from direct toxicity of the drug or its metabolites.

In rare cases, it can be immune mediated.

Immune-mediated toxicity is believed to be due to an idiosyncratic delayed-type hypersensitivity


reaction.

2 Adiponectin;

Is a protein hormone which is involved in regulating glucose levels.

Can have positive impact on CAD.

Plays a role in the development of insulin resistance.

3 NF‘induced pulmonary toxicity;

Is more common among women.

Is rare, but a serious toxic side effect may occur.

Can have the potential to negatively affect lungs.

4 Case Study gives information about;

A rare case of bilateral lower extremity edema in a young patient.

Talks of the effects of the low dose gabapentin therapy.

Worsening condition and management of the disease.

facebook.com/HonorsGroup
READING SUB-TEST

5 What is correct about the survey?

Males were accounted for 53% of total participants

43% participants had a bachelor degree of pharmacy

71% were practicing as pharmacists

6 The data clearly explains that

The majority of the patients were between the age bracket of 29 days to 24 months and

Over half of the patients were female

Male patients outnumbered female patients by 10%

facebook.com/HonorsGroup
READING SUB-TEST

Part C

In this part of the test, there are two texts about different aspects of healthcare. For questions 7-22, choose the
answer (A, B, C or D) which you think fits best according to the text.

Text 1: Occupational Lung Diseases

Occupational lung diseases are a group of illnesses that are caused by either repeated, extended exposure or a
single, severe exposure to irritating or toxic substances that leads to acute or chronic respiratory ailments. The
rate of occupational lung conditions was highest for education and health service workers in the private sector
and local government workers at 3 .8 and 5 .9 per 10,000 full-time workers, respectively. There are two broad
categories of occupational lung diseases: (i) Diseases that are not occupation-specific, but are aggravated at
work, such as occupational asthma (ii) Diseases related to a specific occupation, such as asbestosis, coal
worker’s pneumoconiosis (black lung), berylliosis (brown lung), and farmer’s lung. Common occupational
lung diseases include mesothelioma, occupational asthma, silicosis, asbestosis, and sick building syndrome.
Adult-onset asthma can be triggered by occupational exposures.

The estimated yearly cost of occupational injuries and illnesses is between $128 and $150 billion. Although,
occupational lung diseases are often incurable, they are always preventable. Improving ventilation, wearing
protective equipment, changing work procedures, and educating workers are key factors for prevention.
Occupational Asthma (OA) is the most common form of occupational lung disease. Occupational asthma (also
known as work-related asthma) is asthma that is caused by or made worse by exposures in the workplace.
Estimates suggest that 15 to 23 percent of new asthma cases in adults are work related. Four states (California,
New Jersey, Massachusetts, and Michigan) tracked cases of occupational asthma over a seven-year period.
During this time, the occupations with the highest percentage of asthma cases were operators, fabricators, and
laborers (32.9%); managerial and professional specialty (20.2%), and technical, sales, and administrative
support jobs (19.2%). The four most common agents associated with occupational asthma were miscellaneous
chemicals (19.7%), cleaning materials (11.6%), mineral and inorganic dust (11.1%), and indoor air pollutants
(9.9%).

facebook.com/HonorsGroup
READING SUB-TEST

Malignant mesothelioma is a fatal type of cancer caused by exposure to asbestos. Millions of construction and
general industry workers have been exposed to asbestos while on the job. Occupations associated with
significantly higher mesothelioma deaths include plumbers, pipefitters, and steamfitters; mechanical engineers;
electricians; and elementary school teachers. In the U.S., asbestos use peaked in 1973 but had declined by 99.8
percent in 2007. Because mesothelioma usually does not show up until 20 to 40 years after exposure, most of
the deaths from the disease are the result of exposures that occurred decades ago. This long lag time means that
mesothelioma deaths are expected to peak around 2010, despite the much lower current use of asbestos. From
1999 to 2005, 18,068 malignant mesothelioma deaths were reported in the U.S. Men (81%) and Caucasians
(95%) accounted for the majority of these cases.

Silicosis is a disabling, dust-related disease and is one of the oldest occupational lung diseases in the world.
Silicosis is caused by exposure to and inhalation of airborne crystalline silica. Dust particles from silica can
penetrate the respiratory system and land on alveoli (air sacs). This causes scar tissue to develop in the lungs
and impair the exchange of oxygen and carbon dioxide in the blood. Though symptoms of silicosis rarely
develop in less than five years, progression of the disease can lead to extreme shortness of breath, loss of
appetite, chest pains, and respiratory failure, which can cause death. Silicosis also makes a person more
susceptible to infectious diseases of the lungs, such as tuberculosis. The death rate is generally low, but still
too high considering that every one of these deaths could have been prevented. Because of the low number of
overall deaths due to silicosis, multiple years of data are combined to provide a more accurate estimate of the
burden of this disease.

Text 2: Immune System – Notes

Since inflammation in the body can lead to inflammation in the brain, we first need to understand what
inflammation is. Inflammation is part of the immune system’s response to defend you against microbial
infections. It is the body’s first line of defense against invasion by microorganisms such as bacteria and
viruses, and it is activated rapidly after infection. The microbes are detected as foreign to the body by immune
cells such as macrophages (literally “big eater”). When macrophages encounter and recognize a foreign
microorganism they engulf the microorganism and, in addition, release a variety of cellular products into the
space around them that start and regulate further defenses that include inflammation. Two classes of these
products, known as cytokines and chemokines, leading to inflammation. Cytokines are chemical messengers

facebook.com/HonorsGroup
READING SUB-TEST

that travel away from the cells that release them which causes alterations to the function of other cells.
Chemokines also leave the cell and attract other cells into the region. Together, they alter the blood vessels
near the site of infection, causing increased blood flow to the area and the entry of immune system cells.

Inflammation—swelling, redness and heat—is part of the immune system’s first response to microbial
infections, but this defensive response is not limited to the bodily site of infection. Soon after infection, a
pattern develops that includes what is called the “acute phase response (APR)” and “sickness behavior.” Fever
is the most prominent feature of the APR and for good reason: many microorganisms reproduce best at
humans’ normal core body temperature, and many of the immune system’s agents for killing them are
bolstered by elevated temperature.

Sickness behaviors are well known to anyone who has had the flu. They include reductions in activity, food
intake, social interaction, mood sags; difficulty in forming new memories; sleep changes; and sensitivity to
pain increases (just think of how even a light touch hurts when you have the flu). These changes also reduce
the energetic costs of behavior to free available energy stores to fight the infection. Fever, for example, is quite
energy intensive, requiring an extra 10 to 12 percent in energy for each degree rise. It is obvious how all the
sickness behaviors, with the exception of memory disruption, fit the scheme of keeping us away from our
usual activities. Memory disruption serves a different purpose.

We now understand that all of the changes described above are accomplished through the CNS. Fever, for
example, occurs because the set point of temperature-sensitive cells in the hypothalamus is increased. Of
course, behavior, mood, and pain are all products of the CNS. This raises two issues: a) How does the CNS
“know” what is going on in the peripheral immune system, and b) What kinds of changes are produced in the
CNS that mediate fever and sickness behaviors? The same cytokines that participate in producing the
inflammatory response in the body also initiate the communication process to the CNS. They accumulate in
the bloodstream and thereby travel to the brain, where, although they are large proteins and cannot readily
cross the blood-brain barrier, these chemical signals are carried across the barrier by active transport. They
cross into the brain in regions where the barrier is weak, and they bind to receptors on the insides of the
cerebral vascular blood vessels, thereby inducing the production of soluble mediators within the epithelial cells
that can cross into the brain.

The cytokine interleukin-1 beta is released in response to pathogen recognition, and a) activates vagal fibers, b)
diffuses into the brain where the barrier is weak, and c) are actively transported across. In the brain, they act on

facebook.com/HonorsGroup
READING SUB-TEST

microglia, which then produce and secrete further cytokines that can act on neuronal cells, thereby producing
sickness behaviors.

Often, a set of mechanisms that evolve to handle acute emergencies lead to outcomes that nature did not intend
if they are engaged too long. During a normal infection, neuroinflammation and the resulting adaptive sickness
behaviors persist only for several days. However, if these responses become exaggerated or prolonged, the
outcomes may well become established, leading to cognitive impairment instead of brief memory disruption,
depression instead of reduced mood, fatigue instead of inactivity, and chronic pain instead of acute pain. That
is, physiology can become pathology when a set of processes designed to be relatively brief becomes
prolonged.

facebook.com/HonorsGroup
READING SUB-TEST

Text 1: Questions 7-14

7 The rate of OLC is reported to be higher in;

Healthcare service providers (private sectors)

Professionals in the field of education.

Government officials.

Local government workers and healthcare professionals in private industries.

8 One of these groups of diseases doesn’t come under OLD;

Mesothelioma and occupational asthma.

Occupational asthma and silicosis, asbestosis.

Asbestosis and mesothelioma.

Asbestosis and silicosis.

9 According to paragraph 2, OLDs are;

Incurable

Curable

Preventable but not curable

Curable and preventable

facebook.com/HonorsGroup
READING SUB-TEST

10 According to paragraph 2, ‘work-related asthma’ means;

A disease which occurs due to more work

A disease which occurs due to less work

A disease which occurs due to exposure to work

None of the above

11 Common agents which are associated with OA in the lowest percentage are;

Air pollutants

Mineral and inorganic dust

Cleaning materials

Miscellaneous chemicals

12 The root cause of malignant mesothelioma is associated with the;

Use of the asbestos in the construction field

Exposure to asbestos on a regular basis

Low-quality asbestos

None of the above

facebook.com/HonorsGroup
READING SUB-TEST

13 The use of the asbestos was almost next to naught in the year;

1997

1973

2007

2010

14 Silicosis is;

More dangerous than occupational asthma.

More dangerous than mesothelioma.

Not as fatal as occupational asthma and mesothelioma.

Not very fatal.

Text 2: Questions 15-22

15 The first line of defense implies;

Macrophages

Immune system

Inflammation

All of the above

facebook.com/HonorsGroup
READING SUB-TEST

16 Chemical messengers have;

The ability to change the functioning of other cells.

A greater level of mobility.

No capacity to move further away from cells.

None

17 APR develops;

At the time of infection.

After the infection.

After curing of the infection.

Before or after infection.

18 Fever occurs due to;

The increase in the growth of the microbes.

A powerless immune system.

The multiplication of the microbes, supported by the temperature of the human body.

none

facebook.com/HonorsGroup
READING SUB-TEST

19 One of the following is not associated with sickness behaviors;

Loss of appetite and reduction in social interaction.

Reduction in social interaction and sleeping hours.

Change in mood and body language.

Body language and sleeping disorder.

20 Cytokines are described as;

Messenger cells.

Protein bodies and messenger cells.

Immune response bodies.

None

21 Cytokines cross the barrier;

Through active transport.

By bridging the blood-brain barrier.

By penetrating into the blood vessels.

Through the bloodstream and lymph vessels.

facebook.com/HonorsGroup
READING SUB-TEST

22 What happens if the responses become exaggerated?

Loss of memory may take place.

Reduction in normal mood may be noted.

Cognitive impairment occurs.

Acute pain may begin.

facebook.com/HonorsGroup
READING SUB-TEST

ANSWERS

PART A

1. Correct Answer Is: D


2. Correct Answer Is: C
3. Correct Answer Is: D
4. Correct Answer Is: A
5. Correct Answer Is: B
6. Correct Answer Is: A
7. Correct Answer Is: B
8. Correct Answer Is: Lusitropy
9. Correct Answer Is: Low serum Gal-3 concentrations
10. Correct Answer Is: Gal-3
11. Correct Answer Is: Dyslipidemia
12. Correct Answer Is: NP levels
13. Correct Answer Is: Gal- 3
14. Correct Answer Is: NP secretion
15. Correct Answer Is: Interleukin 1 Beta
16. Correct Answer Is: Diabetes
17. Correct Answer Is: Fibrosis
18. Correct Answer Is: Predictive Marker
19. Correct Answer Is: Natriuresis
20. Correct Answer Is: Oxidative Degradation

PART B & C

1. Correct Answer Is: Hepatotoxicity can arise either from direct toxicity of the drug or its metabolites.
2. Correct Answer Is: Plays a role in the development of insulin resistance.
3. Correct Answer Is: Can have the potential to negatively affect lungs.
4. Correct Answer Is: Talks of the effects of the low dose gabapentin therapy.
5. Correct Answer Is: 71% were practicing as pharmacists
6. Correct Answer Is: The majority of the patients were between the age bracket of 29 days to 24 months and
7. Correct Answer Is: Local government workers and healthcare professionals in private industries.
8. Correct Answer Is: Asbestosis and mesothelioma.
9. Correct Answer Is: Preventable but not curable
10. Correct Answer Is: None of the above
11. Correct Answer Is: Air pollutants
12. Correct Answer Is: Exposure to asbestos on a regular basis
13. Correct Answer Is: 2007
14. Correct Answer Is: Not very fatal.
15. Correct Answer Is: Inflammation
16. Correct Answer Is: The ability to change the functioning of other cells.
17. Correct Answer Is: After the infection.
18. Correct Answer Is: The multiplication of the microbes, supported by the temperature of the human body.
19. Your Answer Is: Body language and sleeping disorder.
20. Correct Answer Is: None
21. Correct Answer Is: Through active transport.
22. Correct Answer Is: Cognitive impairment occurs.

facebook.com/HonorsGroup
READING SUB-TEST

OET 2.0 READING TASK 8

Part A

TIME: 15 minutes

Look at the four texts, A-D, in the separate Text Booklet.

For each question, 1-20, look through the texts, A-D, to find the relevant information.

Write your answers on the spaces provided in this Question Paper.

Answer all the questions within the 15-minute time limit.

Your answers should be correctly spelt.

Text A

Galectin-3

It is well known that there is a close relation between obesity-induced insulin resistance, immune cells
accumulation in white adipose tissue (WAT) and inflammation. Indeed, in obesity WAT is characterized by an
increased production and secretion of a wide range of inflammatory cytokines including TNF-alpha and
interleukin (IL)- 6, which may have local effects on endothelial, vasculature and target adipose tissues.
Therefore, activated macrophages and other antigen presenting cells that are accumulated in elevated number
in fat tissue in both types of obese actively secrete a broad spectrum of locally produced pro-inflammatory
cytokines including galectin-3 (Gal-3). Gal-3 is a beta-galactoside-binding lectin belonging to a
multifunctional protein family, which enhances chemotaxis of immune and antigen presenting cells, reduces

facebook.com/HonorsGroup
READING SUB-TEST

insulin-stimulated glucose uptake in myocytes and adipocytes and impairs insulin-mediated suppression of
glucose output in hepatocytes. Gal-3 may bind directly to the insulin receptor (IR) and thereby inhibit
downstream insulin resistance signaling via diminishing interleukin-1 beta production. Therefore, Gal- 3 is a
modulator of apoptosis, necrosis and fibrosis associated with extracellular remodeling.

Text B

Gal-3 is increased in obesity and mediates inflammation and fibrosis in the heart and vessels, as well as in the
WAT. The most preclinical and clinical studies suggest that this protein protects from inflammation in obese,
while there is a large body of evidence regarding the ability of Gal-3 to deteriorate glucose homeostasis,
modulate cell adhesion and induce pro-oxidant pathways. Interestingly, the low serum Gal-3 concentrations
are closely associated with insulin resistance in patients with type 2 diabetes mellitus. In contrast, an inverse
correlation between serum Gal-3 and glycosylated hemoglobin in type 2 diabetes mellitus was found. In
clinical settings Gal-3 strongly independently predicts all-cause mortality and CV mortality in the general
population and in patients with known CV disease. In fact, in cross-sectional analyses of 2946 Framingham
Heart Study participants circulating Gal-3 was associated well with abdominal adiposity, dyslipidemia, and
hypertension, but Gal-3 did not predict incident CV and metabolic diseases after adjusting for cardiometabolic
risk factors. Whether Gal-3 could be a predictive marker of the metabolically unhealthy obese is not clear,
although Gal-3 deserves further large clinical trials to understand its role in different obese phenotypes’
development.

Text C

Natriuretic peptides (NPs) are “cardiometabolic” hormones with well-established cardiovascular, renal, and
endocrine abilities affecting sodium reabsorption and blood pressure regulation. Although, NP levels are
markers of biomechanical cardiac stress, their role in the nature of the evolution of obese is not fully
understood. These controversies affect the clearance of NPs in obese and pathophysiological mechanisms
controlling the synthesis of them. While, NP secretion result in a stretch of the cardiac wall and volume

facebook.com/HonorsGroup
READING SUB-TEST

overload of cardiac cavities. On the other hand, recent epidemiological and preclinical/ clinical studies have
shown that the NP system acts as a deficiency in obese patients is due to worse clearance of NP receptors and
neutral endopeptidases. Consequently, NP system in obese is not able to mediate a wide spectrum of
cardiovascular and metabolic protective effects (i.e., vasodilation, natriuresis, diuresis, lipolysis, weight loss,
lusitropy, lipid peroxidation, and also improve mitochondrial respiration and insulin sensitivity).

Text D

Brutsaert et al. have reported that higher levels of brain NP have associated with decreased risk of diabetes in
middle-aged adults and that the interrelation has remained after adjustment for waist circumference, low
physical activity, estimated glomerular filtration rate and high sensitive C-reactive protein level. In contrast, it
is suggested that the low brain NP levels observed in obesity could causally associated with the incidence of
diabetes in obese individuals. The effect of brain NPs might relate to an ability of natriuretic peptides to
activate a thermogenic program in brown and white fat tissues, increase energy expenditure and inhibit food
intake. Thus, NPs might play several metabolic roles in the development of different phenotypes of obesity,
but their predictive role in CV disease development in obese patients is uncertain.

facebook.com/HonorsGroup
READING SUB-TEST

Questions 1-7

For each question, 1-7, decide which text (A, B, C or D) the information comes from. You may use any letter
more than once.

In which text can you find information about;

1 Role of this metabolic hormones is not yet certain.

Answer

2 Perform its role when there is a deficiency.

Answer

3 Higher and lower level of the metabolic hormones.

Answer

4 Known to affect the interior surface of blood vessels and lymphatic vessels.

Answer

5 Worsening relative sugar equilibrium.

Answer

facebook.com/HonorsGroup
READING SUB-TEST

6 Introduction to carbohydrate-binding lectin whose expression is associated with inflammatory cells.

Answer

7 May have a direct impact on hemoglobin to which glucose is bound.

Answer

Questions 8-14

Answer each of the questions, 8-14, with a word or short phrase from one of the texts. Each answer may
include words, numbers or both.

8 What defines the rate of myocardial relaxation?

Answer

9 What are more associated with insulin resistance?

Answer

10 What can have the potential to have direct impact on insulin resistance?

Answer

11 What defines an abnormal amount of lipids?

Answer

facebook.com/HonorsGroup
READING SUB-TEST

12 What are the markers of biomechanical cardiac stress?

Answer

13 What is associated with programmed cell death?

Answer

14 What causes the cardiac wall to stretch and volume overload of cardiac cavities?

Answer

Questions 15-20

Complete each of the sentences, 15-20, with a word or short phrase from one of the texts. Each answer may
include words, numbers or both.

15 can be associated with a significantly increased risk for coronary heart disease.

16 higher levels of brain NP can be connected with the decreased risk of

17 The term is referred to the thickening and scarring of connective tissue, usually as a result
of an injury.

18 It is still not clear that Gal-3 is a of the metabolically unhealthy obese

19 Excretion of sodium in the urine is a common condition and is called

20 NPs may not mediate of lipids.

facebook.com/HonorsGroup
READING SUB-TEST

Part B

In this part of the test, there are six short extracts relating to the work of health professionals. For questions 1-
6, choose the answer (A, B or C) which you think fits best according to the text.

Liver toxicity from sulfasalazine

Liver toxicity from sulfasalazine is a rare but serious side effect. It can range from mild elevation in LFTs to
hepatic failure and cirrhosis. The occurrence of severe liver toxicity such as acute hepatitis as seen in our
patient is <1%. After reaching the gut, sulfasalazine is broken down by the colonic bacteria into its
metabolites, i.e., sulfapyridine and 5-aminosalicylic acid. Sulfapyridine is absorbed in the gut and eliminated
after acetylation by enzyme N-acetyltransferase which can have variable activity based on the patient's
genotype. Patients who have genotypes for slow acetylation are found to be more predisposed to sulfasalazine-
induced liver toxicity. Injury can be hepatocellular which presents with disproportionate elevation in serum
aminotransferases or cholestasis which presents with disproportionate elevation in alkaline phosphatase. Both
patterns of liver injury can have elevation in bilirubin and abnormal tests for liver synthetic function.

Adiponectin

Adiponectin accounts for 0.01% of plasma protein with a half-life of 2.5 h; normal adiponectin plasma level is
5–10 μg/mL with higher levels in females than males due to sexual dimorphism. Adiponectin plasma forms are
of two types, high-molecular-weight and low-molecular-weight. In addition, high-molecular-weight
adiponectin levels are positively associated with CAD and negatively associated with risk of type 2 diabetes
mellitus (DM), but this is not true to the low-molecular-weight adiponectin. Adiponectin serum levels are
inversely correlated with body mass index (BMI), visceral obesity, and insulin resistance (IR); thus, it is
regarded as an indicator and predictor of noninsulin dependent DM, insulin resistant, and overt hyperglycemia.

facebook.com/HonorsGroup
READING SUB-TEST

NF‘induced pulmonary toxicity

NF‘induced pulmonary toxicity can be seen in three different forms, that is, due to acute, subacute or chronic
reaction. The acute form is the most common. Side effects occurring up to the 1st month of receiving the first
treatment are classified as the acute form. Acute form develops secondary to hypersensitivity reaction with
peripheral demonstrating eosinophilia and thoracic CT showing ground‘glass opacity. Chronic NF‘induced
lung disease is seen predominantly in older women who present with respiratory symptoms after a year or
more of NF therapy. Characteristic pathologic finding in chronic NF pulmonary toxicity is diffuse interstitial
fibrosis, vascular sclerosis, fibrosis, and thickening of the alveolar septa, interstitial inflammation, and
bronchiolitis obliterans with organising pneumonia.

Case Study

A 46-year-old male with a past medical history of schizoaffective disorder and chronic lower back pain, was
admitted for management of worsening depression and anxiety. He was started on gabapentin, 300mg twice
daily for his back pain and anxiety symptoms. His only other medication was hydrocodone. Over the next few
days, he started developing worsening bilateral lower extremity edema. He did not have any cardiovascular-
related symptoms. Physical exam was only significant for 3+ pitting edema with all laboratory values and
imaging being unremarkable. Gabapentin was discontinued and his lower extremity swelling improved over
subsequent days. The incidence of pedal edema with gabapentin use is approximately 7 to 7.5% with all
studies being in elderly patients receiving doses above 1200 mg/day. This case illustrates that lower doses of
gabapentin can also cause this adverse effect. It is important to recognize this adverse effect because
gabapentin is used in conditions like diabetic neuropathy, which is associated with multiple comorbidities that

facebook.com/HonorsGroup
READING SUB-TEST

can give rise to bilateral leg swelling. Presence of gabapentin-induced leg swelling can thus confound the
clinical picture.

Two hundred and six of the 250 pharmacists completed the questionnaires, giving a response rate of
82.4%. The mean age of pharmacists was approximately 32 years with a majority in the young adult age group
(20-50 years, 92.22%).

Sociodemographic parameter Respondents, N=206(%)

Age (yr) Mean (SD) 36

Gender
128
Male
78
Female

Education qualification
164
Graduate(B. Pharm, D. Pharm)
42
Postgraduate(M. Pharm)

Professional status
142
Practicing Pharmacists
45
Working professional
10
Academicians

facebook.com/HonorsGroup
READING SUB-TEST

Working professional (involved in the manufacture, marketing, distributors, research of drugs)

Demographic data of patients

Number (%)

Age

0-28 days 7 (2.4)

29 days to 12 months 82 (27.9)

13-24 months 85 (28.9)

25-36 months 40 (13.6)

37-48 months 26 (8.8)

49-60 months 54 (18.4)

Sex

Male 172 (58.5)

Female 122 (41.5)

facebook.com/HonorsGroup
READING SUB-TEST

1 What is correct about liver toxicity from sulfasalazine?

Hepatotoxicity can arise either from direct toxicity of the drug or its metabolites.

In rare cases, it can be immune mediated.

Immune-mediated toxicity is believed to be due to an idiosyncratic delayed-type hypersensitivity


reaction.

2 Adiponectin;

Is a protein hormone which is involved in regulating glucose levels.

Can have positive impact on CAD.

Plays a role in the development of insulin resistance.

3 NF‘induced pulmonary toxicity;

Is more common among women.

Is rare, but a serious toxic side effect may occur.

Can have the potential to negatively affect lungs.

4 Case Study gives information about;

A rare case of bilateral lower extremity edema in a young patient.

Talks of the effects of the low dose gabapentin therapy.

Worsening condition and management of the disease.

facebook.com/HonorsGroup
READING SUB-TEST

5 What is correct about the survey?

Males were accounted for 53% of total participants

43% participants had a bachelor degree of pharmacy

71% were practicing as pharmacists

6 The data clearly explains that

The majority of the patients were between the age bracket of 29 days to 24 months and

Over half of the patients were female

Male patients outnumbered female patients by 10%

facebook.com/HonorsGroup
READING SUB-TEST

Part C

In this part of the test, there are two texts about different aspects of healthcare. For questions 7-22, choose the
answer (A, B, C or D) which you think fits best according to the text.

Text 1: Occupational Lung Diseases

Occupational lung diseases are a group of illnesses that are caused by either repeated, extended exposure or a
single, severe exposure to irritating or toxic substances that leads to acute or chronic respiratory ailments. The
rate of occupational lung conditions was highest for education and health service workers in the private sector
and local government workers at 3 .8 and 5 .9 per 10,000 full-time workers, respectively. There are two broad
categories of occupational lung diseases: (i) Diseases that are not occupation-specific, but are aggravated at
work, such as occupational asthma (ii) Diseases related to a specific occupation, such as asbestosis, coal
worker’s pneumoconiosis (black lung), berylliosis (brown lung), and farmer’s lung. Common occupational
lung diseases include mesothelioma, occupational asthma, silicosis, asbestosis, and sick building syndrome.
Adult-onset asthma can be triggered by occupational exposures.

The estimated yearly cost of occupational injuries and illnesses is between $128 and $150 billion. Although,
occupational lung diseases are often incurable, they are always preventable. Improving ventilation, wearing
protective equipment, changing work procedures, and educating workers are key factors for prevention.
Occupational Asthma (OA) is the most common form of occupational lung disease. Occupational asthma (also
known as work-related asthma) is asthma that is caused by or made worse by exposures in the workplace.
Estimates suggest that 15 to 23 percent of new asthma cases in adults are work related. Four states (California,
New Jersey, Massachusetts, and Michigan) tracked cases of occupational asthma over a seven-year period.
During this time, the occupations with the highest percentage of asthma cases were operators, fabricators, and
laborers (32.9%); managerial and professional specialty (20.2%), and technical, sales, and administrative
support jobs (19.2%). The four most common agents associated with occupational asthma were miscellaneous
chemicals (19.7%), cleaning materials (11.6%), mineral and inorganic dust (11.1%), and indoor air pollutants
(9.9%).

facebook.com/HonorsGroup
READING SUB-TEST

Malignant mesothelioma is a fatal type of cancer caused by exposure to asbestos. Millions of construction and
general industry workers have been exposed to asbestos while on the job. Occupations associated with
significantly higher mesothelioma deaths include plumbers, pipefitters, and steamfitters; mechanical engineers;
electricians; and elementary school teachers. In the U.S., asbestos use peaked in 1973 but had declined by 99.8
percent in 2007. Because mesothelioma usually does not show up until 20 to 40 years after exposure, most of
the deaths from the disease are the result of exposures that occurred decades ago. This long lag time means that
mesothelioma deaths are expected to peak around 2010, despite the much lower current use of asbestos. From
1999 to 2005, 18,068 malignant mesothelioma deaths were reported in the U.S. Men (81%) and Caucasians
(95%) accounted for the majority of these cases.

Silicosis is a disabling, dust-related disease and is one of the oldest occupational lung diseases in the world.
Silicosis is caused by exposure to and inhalation of airborne crystalline silica. Dust particles from silica can
penetrate the respiratory system and land on alveoli (air sacs). This causes scar tissue to develop in the lungs
and impair the exchange of oxygen and carbon dioxide in the blood. Though symptoms of silicosis rarely
develop in less than five years, progression of the disease can lead to extreme shortness of breath, loss of
appetite, chest pains, and respiratory failure, which can cause death. Silicosis also makes a person more
susceptible to infectious diseases of the lungs, such as tuberculosis. The death rate is generally low, but still
too high considering that every one of these deaths could have been prevented. Because of the low number of
overall deaths due to silicosis, multiple years of data are combined to provide a more accurate estimate of the
burden of this disease.

Text 2: Immune System – Notes

Since inflammation in the body can lead to inflammation in the brain, we first need to understand what
inflammation is. Inflammation is part of the immune system’s response to defend you against microbial
infections. It is the body’s first line of defense against invasion by microorganisms such as bacteria and
viruses, and it is activated rapidly after infection. The microbes are detected as foreign to the body by immune
cells such as macrophages (literally “big eater”). When macrophages encounter and recognize a foreign
microorganism they engulf the microorganism and, in addition, release a variety of cellular products into the
space around them that start and regulate further defenses that include inflammation. Two classes of these
products, known as cytokines and chemokines, leading to inflammation. Cytokines are chemical messengers

facebook.com/HonorsGroup
READING SUB-TEST

that travel away from the cells that release them which causes alterations to the function of other cells.
Chemokines also leave the cell and attract other cells into the region. Together, they alter the blood vessels
near the site of infection, causing increased blood flow to the area and the entry of immune system cells.

Inflammation—swelling, redness and heat—is part of the immune system’s first response to microbial
infections, but this defensive response is not limited to the bodily site of infection. Soon after infection, a
pattern develops that includes what is called the “acute phase response (APR)” and “sickness behavior.” Fever
is the most prominent feature of the APR and for good reason: many microorganisms reproduce best at
humans’ normal core body temperature, and many of the immune system’s agents for killing them are
bolstered by elevated temperature.

Sickness behaviors are well known to anyone who has had the flu. They include reductions in activity, food
intake, social interaction, mood sags; difficulty in forming new memories; sleep changes; and sensitivity to
pain increases (just think of how even a light touch hurts when you have the flu). These changes also reduce
the energetic costs of behavior to free available energy stores to fight the infection. Fever, for example, is quite
energy intensive, requiring an extra 10 to 12 percent in energy for each degree rise. It is obvious how all the
sickness behaviors, with the exception of memory disruption, fit the scheme of keeping us away from our
usual activities. Memory disruption serves a different purpose.

We now understand that all of the changes described above are accomplished through the CNS. Fever, for
example, occurs because the set point of temperature-sensitive cells in the hypothalamus is increased. Of
course, behavior, mood, and pain are all products of the CNS. This raises two issues: a) How does the CNS
“know” what is going on in the peripheral immune system, and b) What kinds of changes are produced in the
CNS that mediate fever and sickness behaviors? The same cytokines that participate in producing the
inflammatory response in the body also initiate the communication process to the CNS. They accumulate in
the bloodstream and thereby travel to the brain, where, although they are large proteins and cannot readily
cross the blood-brain barrier, these chemical signals are carried across the barrier by active transport. They
cross into the brain in regions where the barrier is weak, and they bind to receptors on the insides of the
cerebral vascular blood vessels, thereby inducing the production of soluble mediators within the epithelial cells
that can cross into the brain.

The cytokine interleukin-1 beta is released in response to pathogen recognition, and a) activates vagal fibers, b)
diffuses into the brain where the barrier is weak, and c) are actively transported across. In the brain, they act on

facebook.com/HonorsGroup
READING SUB-TEST

microglia, which then produce and secrete further cytokines that can act on neuronal cells, thereby producing
sickness behaviors.

Often, a set of mechanisms that evolve to handle acute emergencies lead to outcomes that nature did not intend
if they are engaged too long. During a normal infection, neuroinflammation and the resulting adaptive sickness
behaviors persist only for several days. However, if these responses become exaggerated or prolonged, the
outcomes may well become established, leading to cognitive impairment instead of brief memory disruption,
depression instead of reduced mood, fatigue instead of inactivity, and chronic pain instead of acute pain. That
is, physiology can become pathology when a set of processes designed to be relatively brief becomes
prolonged.

facebook.com/HonorsGroup
READING SUB-TEST

Text 1: Questions 7-14

7 The rate of OLC is reported to be higher in;

Healthcare service providers (private sectors)

Professionals in the field of education.

Government officials.

Local government workers and healthcare professionals in private industries.

8 One of these groups of diseases doesn’t come under OLD;

Mesothelioma and occupational asthma.

Occupational asthma and silicosis, asbestosis.

Asbestosis and mesothelioma.

Asbestosis and silicosis.

9 According to paragraph 2, OLDs are;

Incurable

Curable

Preventable but not curable

Curable and preventable

10 According to paragraph 2, ‘work-related asthma’ means;

A disease which occurs due to more work

A disease which occurs due to less work

facebook.com/HonorsGroup
READING SUB-TEST

A disease which occurs due to exposure to work

None of the above

11 Common agents which are associated with OA in the lowest percentage are;

Air pollutants

Mineral and inorganic dust

Cleaning materials

Miscellaneous chemicals

12 The root cause of malignant mesothelioma is associated with the;

Use of the asbestos in the construction field

Exposure to asbestos on a regular basis

Low-quality asbestos

None of the above

13 The use of the asbestos was almost next to naught in the year;

1997

1973

2007

2010

facebook.com/HonorsGroup
READING SUB-TEST

14 Silicosis is;

More dangerous than occupational asthma.

More dangerous than mesothelioma.

Not as fatal as occupational asthma and mesothelioma.

Not very fatal.

Text 2: Questions 15-22

15 The first line of defense implies;

Macrophages

Immune system

Inflammation

All of the above

16 Chemical messengers have;

The ability to change the functioning of other cells.

A greater level of mobility.

No capacity to move further away from cells.

None

17 APR develops;

facebook.com/HonorsGroup
READING SUB-TEST

At the time of infection.

After the infection.

After curing of the infection.

Before or after infection.

18 Fever occurs due to;

The increase in the growth of the microbes.

A powerless immune system.

The multiplication of the microbes, supported by the temperature of the human body.

none

19 One of the following is not associated with sickness behaviors;

Loss of appetite and reduction in social interaction.

Reduction in social interaction and sleeping hours.

Change in mood and body language.

Body language and sleeping disorder.

20 Cytokines are described as;

Messenger cells.

Protein bodies and messenger cells.

Immune response bodies.

facebook.com/HonorsGroup
READING SUB-TEST

None

21 Cytokines cross the barrier;

Through active transport.

By bridging the blood-brain barrier.

By penetrating into the blood vessels.

Through the bloodstream and lymph vessels.

22 What happens if the responses become exaggerated?

Loss of memory may take place.

Reduction in normal mood may be noted.

Cognitive impairment occurs.

Acute pain may begin.

facebook.com/HonorsGroup
READING SUB-TEST

ANSWERS

PART A

1. Correct Answer Is: D


2. Correct Answer Is: C
3. Correct Answer Is: D
4. Correct Answer Is: A
5. Correct Answer Is: B
6. Correct Answer Is: A
7. Correct Answer Is: B
8. Correct Answer Is: Lusitropy
9. Correct Answer Is: Low serum Gal-3 concentrations
10. Correct Answer Is: Gal-3
11. Correct Answer Is: Dyslipidemia
12. Correct Answer Is: NP levels
13. Correct Answer Is: Gal- 3
14. Correct Answer Is: NP secretion
15. Correct Answer Is: Interleukin 1 Beta
16. Correct Answer Is: Diabetes
17. Correct Answer Is: Fibrosis
18. Correct Answer Is: Predictive Marker
19. Correct Answer Is: Natriuresis
20. Correct Answer Is: Oxidative Degradation

PART B & C

1. Correct Answer Is: Hepatotoxicity can arise either from direct toxicity of the drug or its metabolites.
2. Correct Answer Is: Plays a role in the development of insulin resistance.
3. Correct Answer Is: Can have the potential to negatively affect lungs.
4. Correct Answer Is: Talks of the effects of the low dose gabapentin therapy.
5. Correct Answer Is: 71% were practicing as pharmacists
6. Correct Answer Is: The majority of the patients were between the age bracket of 29 days to 24 months and
7. Correct Answer Is: Local government workers and healthcare professionals in private industries.
8. Correct Answer Is: Asbestosis and mesothelioma.
9. Correct Answer Is: Preventable but not curable
10. Correct Answer Is: None of the above
11. Correct Answer Is: Air pollutants
12. Correct Answer Is: Exposure to asbestos on a regular basis
13. Correct Answer Is: 2007
14. Correct Answer Is: Not very fatal.
15. Correct Answer Is: Inflammation
16. Correct Answer Is: The ability to change the functioning of other cells.
17. Correct Answer Is: After the infection.
18. Correct Answer Is: The multiplication of the microbes, supported by the temperature of the human body.
19. Correct Answer Is: Body language and sleeping disorder.
20. Correct Answer Is: None
21. Correct Answer Is: Through active transport.
22. Correct Answer Is: Cognitive impairment occurs.

facebook.com/HonorsGroup
READING SUB-TEST

OET 2.0 READING TASK 9


Part A

TIME: 15 minutes

Look at the four texts, A-D, in the separate Text Booklet.

For each question, 1-20, look through the texts, A-D, to find the relevant information.

Write your answers on the spaces provided in this Question Paper.

Answer all the questions within the 15-minute time limit.

Your answers should be correctly spelt.

Text A

Battens Disease

The Neuronal Ceroid Lipofuscinoses (NCL's), also known as Battens disease, are a collection of congenital
neurodegenerative conditions that span from prenatal life to late adulthood with an incidence of 1:12,500.
They comprise of at least 8 autosomal recessive disorders defined by having a mutation in a CLN gene, either
coding for an enzyme (CLN1 and CLN2) or a transmembrane protein (CLN3, CLN5, CLN6 and CLN8) with
all disorders having common clinical features, including progressive visual loss to blindness, seizures, speech
disturbances, motor degeneration and intellectual decline, leading to early death LINCL has an incidence of
0.36-0.46 per 100,000 with an age of onset of between 2-4 years, and death commonly anticipated in the early
teenage years. LINCL is caused by a mutation of the CLN2 gene on chromosome 11p15, of which 98
mutations are known, three of which account for the majority of cases.

facebook.com/HonorsGroup
READING SUB-TEST

Text B

The normal product of CLN2 is tripeptidyl peptidase I (TPP-1), which functions within the lysosome to
degrade N-terminal tripeptides from their substrates. Therefore deficiencies lead to an accumulation, in
particular of subunit C of mitochondrial ATP synthase causing subsequent neuronal and retinal cell death. This
accumulation is seen on UV imaging as autofluorescent storage and on electron microscopy as curvilinear
bodies. Gene therapy is an attractive prospect for long-term therapy in LINCL because it is a monogenic
disorder. The process involves introducing CLN2 human complementary DNA (cDNA) into the central
nervous system under the control of a promoter and in a suitable vector. Adeno-associated viruses are
considered the ideal vector as they can transduce postmitotic cells, mediate long-term gene expression and
have an excellent safety record. Therefore, these vectors are considered potential delivery vectors for the
treatment of LINCL. The challenge is whether using a suitable AAV vector allows sufficient activity,
expression and distribution of TPP-1 to destroy existing lysosomal storage protein, prevent its on-going
formation and consequently halt the progression of the disease. One way of maximizing enzyme dispensation
is through cross-correction using mannose-6-phosphate pathway, this means that CLN2 cDNA does not need
not be introduced to the whole of the CNS. The target for gene expression is 1-5% normal expression of the
gene product which is sufficient to reverse pathology in other conditions.

Text C

One human trial evaluated the use of AAV2 vector to transfer human cCLN2 cDNA to the CNS of 10 children
with LINCL aged between 3 and 10 with five different mutation types. The study was an 18-month follow-up
to vector administration with a primary outcome measure being neurological assessment of disease status
using the modified Hamburg LINCL scale. Secondary measures were quantitative CNS Magnetic Resonance
Imaging assessment of the brain including grey matter and ventricular volume. Control comparisons were
made with data from 4 independent untreated LINCL children who had been assessed twice at 1-year intervals.
Participants received an average dose of 2.5 × 1012 particles (1.8-3.2 × 1012) of the AAV2Hcln2 vector. The
dose was shared equally through 6 burr holes (3 in each hemisphere) of the cranial vault, and 12 cortical
locations were targeted. Assessments were made on days 7 and 14 and at 1, 6, 12 and 18 months after therapy.
Adverse effects were assessed at 2 and 3 months.

facebook.com/HonorsGroup
READING SUB-TEST

Text D

While surgery itself recorded no evidence of adverse effects, post-therapy highlighted 60 serious and 94 non-
serious complications, the majority occurring within the first 2 weeks after therapy. Serious events, including
seizures and myoclonus were considered to be consequences of drug administration, whereas vomiting and
thrombocytosis (non-serious examples) were not considered to be caused by the operation or by tolerance to
the therapy. Out of the 10 patients, 2 died, one of unknown reasons and the other during the study period from
status epilepticus, a known complication of late LINCL. Primary assessments demonstrated that gene therapy
had an effect on the progression of disease, with an improved modified Hamburg scale score in treated
subjects monitored for >6 months, and a disease progression rate which was significantly slower than in
controls. This difference was shown by the mean rate of change of the modified Hamburg scale in the treated
and untreated groups (P<0.05). Secondary measures showed that the MRI parameters indicated a decreased
rate of decline, comparable with the primary outcomes, however, there was no statistical significance between
groups including in the grey matter volume as a percentage of total brain volume with -2.6%/year loss of
volume compared to -2.84%/year in controls (p=0.8).

Questions 1-7

For each question, 1-7, decide which text (A, B, C or D) the information comes from. You may use any letter
more than once.

In which text can you find information about;

1 It is not currently known to cause disease, but causes a very mild immune response.

facebook.com/HonorsGroup
READING SUB-TEST

Asnwer

2 A disorder in which nerve cell activity in the brain is disturbed, causing seizures.

Asnwer

3 Not so prevalent disorder that primarily affects the nervous system.

Asnwer

4 Degeneration of the neurons.

Asnwer

5 Recent studies conducted on evaluation of the disease.

Asnwer

6 After effects of the treatment.

Asnwer

7 Affected person may not be in a state to use mental power effectively.

Asnwer

Questions 8-14

facebook.com/HonorsGroup
READING SUB-TEST

Answer each of the questions, 8-14, with a word or short phrase from one of the texts. Each answer may
include words, numbers or both.

8 What is the common reason for Battens Disease?

Answer

9 Which can transduce postmitotic cells?

Answer

10 Which is a known complication of late LINCL?

Answer

11 What are the two conditions which are known to arise not out of therapy ?

Answer

12 What is used to maximize the enzyme dispensation through cross-correction?

Answer

13 Which is the effective long-term therapy in LINCL?

Answer

facebook.com/HonorsGroup
READING SUB-TEST

14 What dosage of the AAV2Hcln2 vector did the participants receive?

Answer

Questions 15-20

Complete each of the sentences, 15-20, with a word or short phrase from one of the texts. Each answer may
include words, numbers or both.

15 can be described as spasmodic jerky contraction of groups of muscles.

16 The gene provides instructions for making an enzyme called tripeptidyl peptidase 1.

17 It is always not required to introduce to CNS.

18 made it all clear that the therapy can have more positive impact on the disease.

19 Out of all the mutations, are considered to be the major causes for the disease to appear in
infants and adults.

20 The gene provides instructions for making a protein whose function is not well understood

facebook.com/HonorsGroup
READING SUB-TEST

Part B

In this part of the test, there are six short extracts relating to the work of health professionals. For questions 1-
6, choose the answer (A, B or C) which you think fits best according to the text.

Angina pectoris

Angina pectoris, whose common type is Qi-blood stagnation type is one of chest congestion in TCM. The
main cause of this type of angina is damage due to an excess of seven emotions. The heart governs the mind
and blood circulation, liver stores of blood and controls catharsis. There is close relationship between the two.
Only if liver function is normal can Qi-blood of the heart can be calm. For elderly individuals, who lack Qi-
blood, yin and yang of the heart, spleen and kidney, are of cold invasion, consume an inappropriate diet, have
emotional disorders and excessive fatigue, among others, can also lead to blood stasis stagnation, which can
consequently cause coronary angina pectoris of Qi-blood stagnation. The ‘Suwen•Yujizhenzang’ theory has
described “blockage of vessel, inability of QI to flow freely”, which illustrates that blood stagnation can lead
to Qi depression. Therefore, accelerating blood circulation, removing blood stasis and promoting the
circulation of Qi are the main therapies.

facebook.com/HonorsGroup
READING SUB-TEST

Spinal cord stimulation

For many years, Spinal cord stimulation has been used as a salvage treatment for intractable CRPS even
though many studies have not proven long-term benefit. In multiple studies published by a European
neurosurgical group, there has been great benefit from this technology in the first year but the vast majority of
patients experience a return of symptoms by year six. Newer dorsal root ganglion (DRG) stimulation
technology may be more promising. In the most recent DRG stimulation trial, patients with CRPS have been
shown to decrease pain by fifty percent or greater in 93% of patients with chronic intractable pain at three-
month follow-up, versus 72% of patients with an SCS implant. Unfortunately, this product is currently only
FDA approved for treatment in the lower extremity.

Stellate ganglion blocks (SGB)

Stellate ganglion blocks (SGB) are the most commonly performed interventional procedure for patients with
upper extremity CRPS. The stellate ganglion is located anterior to the 7th cervical transverse process on the
anterior surface of the longus coli muscle. It lies medial to the vertebral artery and anterolateral to the
ipsilateral common carotid artery. This ganglion can be accessed either through fluoroscopy, CT, or ultrasound
guidance. In a study published in 2006 by Ackerman and Zhang, 25 subjects underwent SGB at weekly
intervals for 3 weeks. At 6 months, 40% of patients had complete symptom relief while 24% of patients had
no pain relief. In a second study, published in 2009, three weekly blockades were completed at weekly
intervals in 22 patients with CRPS type I of the hand. Pain intensity and range of motion were assessed two
weeks after treatment. In this study patients had statistically significant improvement in wrist ROM (P>0.001)
and an overall decrease in VAS values from 8 to 1. While most physicians do not believe that SGB alone is
effective in curing the disease, we do know that stellate ganglion blocks at least offer temporary benefits that
last well beyond the effects of local anesthetic.

facebook.com/HonorsGroup
READING SUB-TEST

Metabonomics

Metabonomics as an advanced technology of systems biology has made considerable contributions to early
diagnosis of serious illness, toxicology and pharmacology. Early, accurate diagnosis of tumours, such as liver
cancer and ovarian cancer, is a distinguishing feature and advantage of metabonomics. Valine, saturated lipids,
glycine, lactate, inositol, nucleotides, polyunsaturated fatty acids, taurine and other tumour-related metabolic
markers can be identified by metabolomics, which makes metabonomics increasingly applicable to diagnosis,
therapy and prognostic evaluation. Acupuncture has a therapeutic effect for tumours: it can improve
symptoms, prolong the lifespan of patients, improve the quality of life, relieve cancer pain syndrome and,
especially, improve the side effects of radiotherapy and chemotherapy, eg, myelosuppression, gastrointestinal
reaction, fever etc. Acupuncture treatment of tumour possesses many features and advantages. Applying
metabonomics to research investigating acupuncture treatment of tumours both gives play to the sensitive
detection advantage of metabonomics for tumour diagnosis, treatment assessment and exploring the
mechanism of acupuncture treatment at deeper, more extensive and more dynamic levels. In addition, there are
some reports investigating Alzheimer diseases at home and abroad, however, few in research of venereal
disease.

facebook.com/HonorsGroup
READING SUB-TEST

Treatment Groups - Recent Analysis

Total effective
Groups Cases excellent effective Ineffective exacerbated
rate, %

Control 40 19 9 10 2 70.0

Treatment 41 28 9 4 0 90.2*

Disease Treatment Results - Before and After

Control group (n=58) Observation group (n=59)

Before After Before After

Index treatment treatment treatment treatment

FBG, mmol/L 9.85 ± 1.25 8.53 ± 1.42 9.92 ± 1.25 7.21 ± 1.33*

PBG, mmol/L 13.54 ± 2.73 11.32 ± 2.89 14.07 ± 2.32 9.25 ± 1.08*

HbA1c, % 11.02 ± 2.34 10.32 ± 2.53 11.31 ± 1.97 8.79 ± 1.17*

Microalbuminuria, mg/L 81.24 ± 11.93 76.42 ± 12.33* 82.31 ± 10.64 45.22 ± 13.17*

facebook.com/HonorsGroup
READING SUB-TEST

1 What does the notice indicates?

Angina pectoris is common among people.

There is a direct relationship between the functioning of liver and Qi blood of the heart.

Various ailments can lead to angina pectoris.

2 The notice gives information about;

two different technologies used in treatment.

ineffectiveness of the SCS.

DRG and SCS comparison.

3 SGB is known;

to be more effective in treatment.

to provide temporary relief from pain.

to be a proven medicine in certain instances.

4 What is correct about metabonomics?

High-throughput metabolomic approach revealed the acupuncture exerting intervention effects.

Metabolomics has seen a surge in popularity in recent scientific research.

The ultimate aim of metabonomics is to detect every small-molecule metabolite.

5 What does this notice indicate?

The curative effect of improving disease condition in treatment group is superior to the control group.

two groups differ only by minor values.

facebook.com/HonorsGroup
READING SUB-TEST

total effectiveness of the control group is lesser than treatment group.

6 What does the result indicate?

The results indicate that subjects in the two groups improved after treatment.

The laboratory indexes of the treatment group significantly declined.

After treatment, the laboratory indexes of the observation group were also more declined than the control
group.

facebook.com/HonorsGroup
READING SUB-TEST

Part C

In this part of the test, there are two texts about different aspects of healthcare. For questions 7-22, choose the
answer (A, B, C or D) which you think fits best according to the text.

Text 1: Hormonal Disorder – Adrenal Insufficiency

Adrenal insufficiency is an endocrine or hormonal disorder that occurs when the adrenal glands do not produce
enough of certain hormones. The adrenal glands are located just above the kidneys. Adrenal insufficiency can
be primary or secondary. Primary adrenal insufficiency, also called Addison’s disease, occurs when the adrenal
glands are damaged and cannot produce enough of the hormone cortisol, and often the hormone aldosterone.
Addison’s disease affects one to four of every 100,000 people, in all age groups and both sexes. Secondary
adrenal insufficiency occurs when the pituitary gland, a bean-sized organ in the brain, fails to produce enough
adrenocorticotropin (ACTH), a hormone that stimulates the adrenal glands to produce cortisol. If ACTH output
is too low, cortisol production drops. Eventually, the adrenal glands can shrink due to lack of ACTH
stimulation. Secondary adrenal insufficiency is much more common than Addison’s disease.

Addison's disease symptoms usually develop slowly, often over several months, and may include muscle
weakness and fatigue, weight loss and decreased appetite, darkening of skin (hyperpigmentation), low blood
pressure (even fainting), salt cravings, low blood sugar (hypoglycemia), nausea, diarrhea or vomiting, muscle
or joint pains etc. Sometimes, however, the signs and symptoms of Addison's disease may appear suddenly. In
acute adrenal failure (Addisonian crisis), the signs and symptoms may also include pain in your lower back,
abdomen or legs, severe vomiting and diarrhea, leading to dehydration, low blood pressure and loss of
consciousness.

Your adrenal glands are composed of two sections: the interior (medulla) produces adrenaline-like hormones;
the outer layer (cortex) produces a group of hormones called corticosteroids, which include glucocorticoids,
mineralocorticoids and male sex hormones (androgens). Glucocorticoids: These hormones, which include
cortisol, influence your body's ability to convert food fuels into energy, play a role in your immune system's
inflammatory response and help your body respond to stress. Mineralocorticoids: These hormones, which
include aldosterone, maintain your body's balance of sodium and potassium to keep your blood pressure

facebook.com/HonorsGroup
READING SUB-TEST

normal. Androgens, male sex hormones, are produced in small amounts by the adrenal glands in both men and
women. They cause sexual development in men and influence muscle mass, libido and a sense of well-being in
men and women.

Addison's disease occurs when the cortex is damaged and doesn't produce its hormones in adequate quantities.
Doctors refer to the condition involving damage to the adrenal glands as primary adrenal insufficiency. The
failure of your adrenal glands to produce adrenocortical hormones is most commonly the result of the body
attacking itself (autoimmune disease). For unknown reasons, your immune system views the adrenal cortex as
foreign, something to attack and destroy. Other causes of adrenal gland failure may include tuberculosis, other
infections of the adrenal glands, spreading of cancer to the adrenal glands and bleeding into the adrenal glands.
Adrenal insufficiency can also occur if your pituitary gland is diseased. The pituitary gland produces a
hormone called adrenocorticotropic hormone (ACTH), which stimulates the adrenal cortex to produce its
hormones. Inadequate production of ACTH can lead to insufficient production of hormones normally produced
by your adrenal glands, even though your adrenal glands aren't damaged. Doctors call this condition secondary
adrenal insufficiency. Another more common cause of secondary adrenal insufficiency occurs when people
who take corticosteroids for treatment of chronic conditions, such as asthma or arthritis, abruptly stop taking
the corticosteroids. If you have untreated Addison's disease, an Addisonian crisis may be provoked by physical
stress (such as an injury), infection or illness. All treatment for Addison's disease involves hormone
replacement therapy to correct the levels of steroid hormones your body isn't producing. Some options for
treatment include oral corticosteroids, corticosteroid injections and androgen replacement therapy.

Text 2: Carpal Tunnel Syndrome

Carpal tunnel syndrome is a condition that may be caused by repeatedly performing stressful motions with
your hand or holding your hand in the same position for long periods of time. CTS is classified as a cumulative
trauma disorder, an ailment that attacks the body’s musculoskeletal system. The musculoskeletal system is
made up of muscles that pull on tendons and move the bones at joints. The joints are held together by
ligaments. Carpal tunnel syndrome specifically affects the sensitive nerves of - and the blood supply that feeds
- the hands and wrists.

facebook.com/HonorsGroup
READING SUB-TEST

Carpal tunnel syndrome has been around for a long time; meatpackers began complaining of pain and loss of
hand function in the 1860s. Back then, these complaints were largely attributed to poor circulation. The nature
of work has changed over the years; today, more jobs are highly specialized and require use of only a small
number of muscles repeatedly. With the growing numbers of people using computers and keyboards, plus the
focus on better healthcare for workers, carpal tunnel syndrome is of real concern to both employers and health-
care professionals. Recent studies have shown that carpal tunnel syndrome, like all other cumulative trauma
disorders, is on the rise while other workplace injuries have leveled off. Many companies are turning to
physical therapists for help with designing and implementing health promotion and injury prevention programs
to protect their employees from CTS.

People with CTS usually experience feelings of numbness, weakness, tingling, and burning in their fingers and
hands. If not treated, the symptoms may escalate into acute, persistent pain. CTS can become so crippling that
people can no longer do their work or even perform simple tasks at home. At its most extreme, carpal tunnel
syndrome forces people to undergo surgery and miss many days of work, or prevents them from working at all
because their hand functions are permanently impaired. Carpal tunnel syndrome occurs in men and women of
all ages, and is often found in workers whose tasks require repeating the same motion in the fingers and hand
for long periods of time. CTS has surfaced among meat packers, assembly line workers, jackhammer
operators, and employees who spend hours working at a computer or typewriter. Carpal tunnel syndrome
shows up in athletes as well as homemakers.

The U.S. Department of Labor has cited carpal tunnel syndrome, as well as other cumulative trauma disorders,
as the cause of 48 percent of all industrial workplace illnesses. The disease affects more than five million
Americans. CTS’s impact on American businesses is devastating. It shows up in the workplace in the form of
fatigue, poor work performance, discomfort and pain, and poor employer/employee relations. The high cost of
treatment for an employee with CTS, plus the lost productivity when that employee is absent for a long period
of time, strains the company’s ability to operate efficiently and can lead to morale problems when other
employees have to take over the absent workers’ responsibilities. Physical therapists with specialized training
in cumulative trauma disorders have been working in industrial and corporate settings for many years to meet
the health-care needs of America’s workforce. They work closely with employers to educate employees about
CTS—what causes it and how to avoid it through proper use of the musculoskeletal system.

facebook.com/HonorsGroup
READING SUB-TEST

Physical therapists can target and correct poor work habits and improper work designs, such as tools, furniture,
equipment, and workspace. They can also assess the potential risks of an individual and determine if that
person is physically unsuited to a particular job. Among their many responsibilities, physical therapists teach
health awareness and job safety. A typical education program includes exercises employees can do at work and
at home, adjustments to the overall work environment and individual workstations, plus early detection of
symptoms to avoid painful and costly surgery. Physical therapists also work with employers and their
engineering departments to design and modify the work environment, helping to remove the causal factors of
CTS. If anyone has symptoms of carpal tunnel syndrome, then consulting a physical therapist or other
qualified healthcare practitioners for an evaluation and individualized treatment is always recommended.

facebook.com/HonorsGroup
READING SUB-TEST

Text 1: Questions 7-14

7 Addison’s disease effects;

Four of every 1,000 people.

One of every 1,000 people, including all age-groups.

One to four of every 1,000 people, including all age groups.

One to four of every 100,000 people.

8 Secondary adrenal insufficiency occurs due to one of these reasons;

More production of adrenocorticotropin (ACTH).

More production of cortisol.

Low production of aldosterone.

Low production of ACTH.

9 According to the passage, symptoms of Addison’s disease occur;

Slowly

Suddenly

After a month

Slowly after many months

10 Cortex produces;

Androgens

Glucocorticoids

facebook.com/HonorsGroup
READING SUB-TEST

Mineralocorticoids

All of the above

11 Aldosterone maintains;

Body balance

Balance of sodium and potassium

High BP

Low BP

12 According to the information given in the passage, the statement that “Androgen influences
muscle mass and physical and mental nature of men and women” is;

True

False

Can’t say

Not given in the passage(s)

13 Doctors refer to the damage to the cortex of the kidney as;

Primary adrenal insufficiency.

Secondary adrenal insufficiency.

Other fatal infections.

Not given

14 Secondary adrenal insufficiency also occurs because of;

facebook.com/HonorsGroup
READING SUB-TEST

Asthma

Arthritis

Both asthma and arthritis.

Stopping the use of corticosteroids.

Text 2: Questions 15-22

15 According to the passage, CTS;

Is a cumulative trauma disorder.

Is caused due to weakness in musculoskeletal system.

Occurs due to weakness in ligaments between joints.

All of the above

16 According to the information given, CTS is;

On the rise.

On the rise without any other cumulative trauma disorders.

One of the common cumulative trauma disorders that are increasing.

On the rise with many other cumulative trauma disorders.

17 “In CTS, _____ may become permanently impaired.”

Carpels

Hands

Feet

facebook.com/HonorsGroup
READING SUB-TEST

Wrists

18 CTS often occurs in;

Men

Women

Men and women of all ages

Only men of all ages

19 _____ complained of pain and loss of hand function in the 1860s;

Meat packers

Assembly line workers

Jackhammer operators

Employees who spend hours at a computer

20 CTS accounts for _____.of all industrial workplace illnesses;

50%

48%

84%

60%

facebook.com/HonorsGroup
READING SUB-TEST

21 Physical therapists teach employees about;

Proper use of the musculoskeletal system.

How to avoid CTS.

How to increase efficiency at work.

Health-care needs.

22 CTS educational program includes;

Only detection of CTS symptoms in employees.

Explanation to employees on how to avoid CTS.

Exercises that employees can do to avoid CTS.

Discussion of cost involved in surgery.

facebook.com/HonorsGroup
READING SUB-TEST

ANSWERS

PART A

1. Correct Answer Is: B


2. Correct Answer Is: D
3. Correct Answer Is: A
4. Correct Answer Is: B
5. Correct Answer Is: C
6. Correct Answer Is: D
7. Correct Answer Is: A
8. Correct Answer Is: mutation of the CLN2 gene
9. Correct Answer Is: Adeno-associated viruses
10. Correct Answer Is: epilepticus
11. Correct Answer Is: vomiting and thrombocytosis
12. Correct Answer Is: mannose-6-phosphate pathway
13. Correct Answer Is: Gene therapy
14. Correct Answer Is: 2.5 × 1012 particles
15. Correct Answer Is: Myoclonus
16. Correct Answer Is: Tpp1
17. Correct Answer Is: Cln2 Cdna
18. Correct Answer Is: Primary Assessments
19. Correct Answer Is: Three
20. Correct Answer Is: Cln6

PART B & C

1. Correct Answer Is: Various ailments can lead to angina pectoris.


2. Correct Answer Is: ineffectiveness of the SCS.
3. Correct Answer Is: to provide temporary relief from pain.
4. Correct Answer Is: Metabolomics has seen a surge in popularity in recent scientific research.
5. Correct Answer Is: The curative effect of improving disease condition in treatment group is superior to the
control group.
6. Correct Answer Is: The results indicate that subjects in the two groups improved after treatment.
7. Correct Answer Is: One to four of every 1,000 people, including all age groups.
8. Correct Answer Is: Low production of ACTH.
9. Correct Answer Is: Slowly after many months
10. Correct Answer Is: Androgens
11. Correct Answer Is: Balance of sodium and potassium
12. Correct Answer Is: False
13. Correct Answer Is: Primary adrenal insufficiency.
14. Correct Answer Is: Stopping the use of corticosteroids.
15. Correct Answer Is: Is a cumulative trauma disorder.
16. Correct Answer Is: On the rise with many other cumulative trauma disorders.
17. Correct Answer Is: Wrists
18. Correct Answer Is: Men and women of all ages
19. Correct Answer Is: Meat packers
20. Correct Answer Is: 48%
21. Correct Answer Is: How to avoid CTS.
22. Correct Answer Is: Exercises that employees can do to avoid CTS.

facebook.com/HonorsGroup
READING SUB-TEST

OET 2.0 READING TASK 10

Part A

TIME: 15 minutes

Look at the four texts, A-D, in the separate Text Booklet.

For each question, 1-20, look through the texts, A-D, to find the relevant information.

Write your answers on the spaces provided in this Question Paper.

Answer all the questions within the 15-minute time limit.

Your answers should be correctly spelt.

Text A

Autism Spectrum Disorder

Autism Spectrum Disorder (ASD) develops in early childhood. Recent population analysis indicates that the
number of cases of ASD is increasing in many countries, particularly in technologically developed countries.
The U.S. Center for Disease Control research claims that, in some states, one of every 68 children (one of 42
boys) has a diagnosis of the ASD, a 30% increase from 2012 (IACC Strategic Plan for Autism Spectrum
Disorder Research, 2013.

Multiple dysfunctional reflex patterns are characteristic in two separate groups of children diagnosed with
autism: 1) those whose patterns were immature or pathological and severely dysfunctional from birth, and 2)
those that developed normally but regressed into autism at age 2 or 3 unexpectedly. Reflexes of these children
may have been delayed and immature, but not noted by specialists or parents. Their nerve system, possibly,
was not resilient enough to cope with the stress that they experienced. Alternatively, their reflexes might not
have matured and have caused the asynchronicity in their brain function development on both cortical and

facebook.com/HonorsGroup
READING SUB-TEST

extrapyramidal levels resulting in neurodevelopmental disorders beginning around 2 years of age. An initially
mild unrecognized problem can lead to more complicated deficits with age.

Text B

Individuals diagnosed with ASD show a chronic lack of sensory motor integration and delay of skills
concerning the early motor milestones. They show a wide range of immature reflex patterns such as Hands
Pulling, Hands Supporting, Hands Grasp, Crawling, Asymmetrical Tonic Neck Reflex, Symmetrical Tonic
Neck Reflex, Babkin Palmomental, Ocular-Vestibular, and other patterns. The MNRI program utilizes non-
invasive intervention to support the development of the neuro-sensory-motor aspects of those reflex patterns
through specific techniques and procedures that allow restoration of links between reflex circuit components
and the protection function of a reflex to normalize their over-freezing and fight or flight reactions seen, for
example, in tactile defensiveness or deprivation. Thus, the MNRI program works particularly with the
autonomic nervous system – its sympathetic and parasympathetic processes.

Text C

Disharmony in muscular system development and lack of regulation for muscle tone beginning in children
with ASD in their infancy results in impulsive reactions that often turns into permanent physical characteristics
and behaviors as they grow older. For example, impulsivity may lead to poor ability for goal setting, poor
focus and following instructions, deficient inner control, hyperactivity, disorganized and chaotic behavior, and
irritability and impatience. Lack of muscle tone regulation may later result in challenges in motor
programming and control, planning, and thus lead to poor motor-cognitive- behavior coordination. This poor
regulation is caused by a lack of balance in the excitation and inhibition processes in the reflex circuits,
including improper connectivity between alpha and gamma motor neurons. Clinical observations show that the
disharmony and lack of proper regulation in muscle tone in children with ASD are seen mainly in: Hypertonic
muscles in the posterior dorsal plane of the body (along the spinal column - sacrospinalis, thoracic longus,
trapezius) and with the opposite hypotonic abdominal muscles and diaphragm negatively affects development

facebook.com/HonorsGroup
READING SUB-TEST

of postural control. The child with ASD, in an attempt to release tension caused by this disproportion of
muscle tone in the back and front of their body, may often display reactivity in behavior and impulsive
movements triggering balance/equilibrium mechanisms (balancing reflex pattern), resulting in a state of being
overstimulated.

Text D

Problems in visual and auditory perception systems: The eyes of children diagnosed with ASD show a restless
state or lack of mobility and dilated pupils. They usually have a limited, narrow visual span, poor visual
attention and focus, and hyperactive peripheral vision. Their eye movements appear to freeze or jump rapidly
in saccades. Many children with ASD demonstrate an addictive tendency for computers and cell phones with
compulsive repetition of the same image, object, or program, often watching it at a very close range. The child
with ASD becomes over-focused, which over-stimulates their vestibular system and static balance. The
Pupillary Reflex in these children may become hypersensitive, over-stimulating the sympathetic system, with
either over-reactive or hypoactive motor activity. The visual system of children diagnosed with ASD copes
poorly with this visual chaos which leads to a visual processing disorder. A Bonding response in infants is
seen from their first months after birth. Bonding as a behavior trait matures during their first years of life.
Almost every child with ASD assessed presented signs of inadequate bonding – lack of attachment, tactile and
interactivity defensiveness, a tendency for self-isolation, a poor imitation, and poor learning of verbal
communication. When bonding is immature, there are problems with visual contact, focusing on the face/eyes
of their mother and other adults and poor emotional communication, inability to adequately smile, and poor
labeling of the objects in their environment.

facebook.com/HonorsGroup
READING SUB-TEST

Questions 1-7

For each question, 1-7, decide which text (A, B, C or D) the information comes from. You may use any letter
more than once.

In which text can you find information about;

1 Represents the resistance to passive movement of a joint.

Answer

2 Associated with pupillary function.

Answer

3 Utilization of information and clinical experience from neurodevelopment in different ways.

Answer

4 Development of autism.

Answer

5 Possibility of development of strange characteristics as one grows.

Answer

6 Not existing or occurring at the same time with respect to movements or reactions.

facebook.com/HonorsGroup
READING SUB-TEST

Answer

7 Primitive reflex that normally emerges during the first year of an infant's life.

Answer

Questions 8-14

Answer each of the questions, 8-14, with a word or short phrase from one of the texts. Each answer may
include words, numbers or both.

8 What MNRI will operate with?

Answer

9 How eye movements may appear?

Answer

10 What is the term which defines activation of the vestibular system which causes eye movement?

Answer

11 What is related to tendency to act on a whim, displaying behavior characterized by little or no forethought,
reflection?

Answer

12 What is the term used to define healthy stress?

facebook.com/HonorsGroup
READING SUB-TEST

Answer

13 What is known to be activated as a result of turning the head to one side?

Answer

14 What is the impact visual chaos of the children with ASD?

Answer

Questions 15-20

Complete each of the sentences, 15-20, with a word or short phrase from one of the texts. Each answer may
include words, numbers or both.

15 The term is used to describe a rapid movement of the eye between fixation points.

16 Generally, in newly born babies will be at very early stages.

17 What synchronizes hands, neck, and jaw is

18 In many of the cases, reflex in affected children may turn out to be more .

19 Almost all of the children with ASD show various signs of .

20 The complexities in can be the result of the regulation changes in muscle tone.

facebook.com/HonorsGroup
READING SUB-TEST

Part B

In this part of the test, there are six short extracts relating to the work of health professionals. For questions 1-
6, choose the answer (A, B or C) which you think fits best according to the text.

UNAIDS Programme Coordinating Board

The 31st UNAIDS Programme Coordinating Board (PCB) meeting took place in Geneva from 11-13 June
2018. There were more than 700 000 less new HIV infections estimated globally in 2016 than in 2001. The
road from 2.5 million new HIV infections in 2011 to zero new HIV infections is a long one and significant
efforts are required to accelerate HIV prevention programmes. Sustained investments for access to
antiretroviral therapy by donors and national governments have led to record numbers of lives being saved in
the past six years. In 2011 more than half a million fewer people died from AIDS-related illnesses than six
years earlier. It’s a dramatic turning point. Numbers can quantify, but alone cannot express the impact of each
averted death on the whole community, including its children. The number of people dying from AIDS-related
causes began to decline in the mid-2000s because of scaled up antiretroviral therapy and the steady decline in
HIV incidence since the peak of the epidemic in 1997. In 2011, this decline continued, with evidence showing
that the drop in the number of people dying from AIDS-related causes is accelerating in several countries.

Radix Sophorae tonkinensis

Radix Sophorae tonkinensis was crushed and screened, then taking screened powder (approximately 0.5 g) gain
a respective weight, denoted M. Trichloromethane-methanol-ammonia (40:10:1) was used to dispose of the
radix Sophorae tonkinensis samples for 30 min, and all samples were subsequently obtained from organic

facebook.com/HonorsGroup
READING SUB-TEST

solvent extraction with 30 min ultrasonic treatment. All disposed of samples were filtered, then 10 mL of
filtrate was measured to recover solvents to dry under decompression at 38°C to obtain the residue, the residue
was diluted by methanol, then transferred to a 10 mL volumetric flask. After mixing and filtering with 0.45 μm
filter membrane, Radix Sophorae samples were obtained. The blank groups were treated as the samples but
without radix Sophorae tonkinensis.

Short note on Torcetrapib

Torcetrapib, which has been in development since the early 1990s, was supposed to raise so-called good
cholesterol, and cardiologists had hoped it would reduce the buildup of plaques in blood vessels that can cause
heart attacks. This drug actually caused an increase in deaths and heart problems. Eighty-two people had died
so far in a clinical trial, versus 51 people in the same trial who had not taken it. The GABR company gave up
the project incurring a claimed loss of $1 billion investment and not much hue and cry was raised on the deaths
of the study-subjects as this was the doing of a billionaire giant manufacturer. In case of trials or experiments,
if a single death is noticed with the use of UD, a big hue and cry would be raised.

Drug Adverse Reactions

Type of ADR Number ADRS Percentage (%)

Type-A 96 103.7

Type-B 69 56.79

Type-C 23 28.39

facebook.com/HonorsGroup
READING SUB-TEST

Type G 8 7.4

Type H 4 3.7

Total 200 200

Vitamin B12

Vitamin B12 is produced by the liver and is involved in several biochemical metabolic reactions. It promotes
the repair of damaged skin mucous membranes and vascular endothelial cells, reduces spasm and occlusion of
blood vessels, improves local blood flow and prevents the deterioration of wound infection. In addition, it
reduces the excitability of pain fibres C and AG, leading to an analgesic effect. Vitamin B12 injections to the
skin in the radiation field benefit the wound by reducing irritation and pain, preventing rupture and enhancing
new epithelial resistance to radiation, thereby promoting healing of the skin. Chen et al used a vitamin B12
solution to treat radiation-induced moist dermatitis. The cure rate at 10 days was 100%, which was
significantly different from the control group.

Age wise distribution of male patients showing percentage of distribution.

Age in years Male patients Age in years Female patients

20-30 4 (7.27) 20-30 0 (0)

30-40 54 (98.18) 30-40 32 (71.11)

40-50 24 (43.63) 40-50 38

facebook.com/HonorsGroup
READING SUB-TEST

50-60 18 50-60 10

60-70 10 60-70 6 (13.32)

Total 110 (110) 70-80 4 (8.88)

Total 90 (90)

1 What this notice talks about?

AIDS-related deaths dropped by more than 50%

AIDS-related deaths increased more significantly

Efforts that can lead to curbing AIDS in various countries.

2 The given notice explains the procedure of;

Use of radix Sophorae samples.

Ultrasonic treatment - obtaining radix.

Preparing radix Sophorae tonkinensis samples.

3 What is correct about Torcetrapib?

It is known to enhance health.

Trials performed produced negative results.

The drug didn’t get approved completely as the project was dropped in the middle.

4 The table shows

facebook.com/HonorsGroup
READING SUB-TEST

Comparison of Type A, B, C and G and H drugs and their resistance quality.

Type A is known to produce more adverse reactions than all the others present in the table.

A total of 200 cases have been reported with respect to adverse reaction.

5 The notice clearly explains

Clinical trials using Vitamin B12.

Advantages of B12.

Study performed with focus on B12 usage.

6 What is correct about the given table?

The age wise male patients population ranges from 4.

18.18 patients were in the age group of 50-70 years.

32.72 patients were in the age group of 50-60 years.

facebook.com/HonorsGroup
READING SUB-TEST

Part C

In this part of the test, there are two texts about different aspects of healthcare. For questions 7-22, choose the
answer (A, B, C or D) which you think fits best according to the text.

Text 1: Ebola Virus and Marburg Virus

The Ebola virus and Marburg virus are related viruses that cause hemorrhagic fevers; illnesses marked by
severe bleeding (hemorrhage), organ failure and, in many cases, death. Both the Ebola virus and Marburg virus
are native to Africa, where sporadic outbreaks have occurred for decades. The Ebola virus and Marburg virus
both live in animal hosts, and humans can contract the viruses from infected animals. After the initial
transmission, the viruses can spread from person to person through contact with bodily fluids or contaminated
needles.

No drug has been approved to treat the Ebola virus or Marburg virus. People diagnosed with the Ebola or
Marburg virus receive supportive care and treatment for complications. Scientists are coming closer to
developing vaccines for these deadly diseases. In both the Ebola virus and Marburg virus, signs and symptoms
typically begin abruptly within the first five to 10 days of infection. Early signs and symptoms include fever,
severe headaches, joint and muscle aches, chills, sore throat and weakness. Over time, symptoms become
increasingly severe and may include nausea and vomiting, diarrhea (may be bloody), red eyes, raised rash,
chest pain and coughing, stomach pain, severe weight loss, bleeding from the nose, mouth, rectum, eyes and
ears.

The Ebola virus has been found in African monkeys, chimps and other nonhuman primates. A milder strain of
Ebola has been discovered in monkeys and pigs in the Philippines. The Marburg virus has been found in
monkeys, chimps and fruit bats in Africa. The virus can be transmitted to humans by exposure to an infected
animal's bodily fluids, including blood. Butchering or eating infected animals can spread the viruses; scientists
who have operated on infected animals as part of their research have also contracted the virus. Infected people
typically don't become contagious until they develop symptoms. Family members are often infected as they
care for sick relatives or prepare the dead for burial.

facebook.com/HonorsGroup
READING SUB-TEST

Medical personnel can be infected if they don't use protective gear such as surgical masks and latex gloves.
Medical centers in Africa are often so poor that they must reuse needles and syringes and some of the worst
Ebola epidemics have occurred because contaminated injection equipment wasn't sterilized between uses.
There's no evidence that the Ebola virus or Marburg virus can be spread via insect bites. Ebola and Marburg
hemorrhagic fevers are difficult to diagnose because many of the early signs and symptoms resemble those of
other infectious diseases, such as typhoid and malaria. But if doctors suspect that you have been exposed to the
Ebola virus or Marburg virus, they use laboratory tests that can identify the viruses within a few days.

Most people with Ebola or Marburg hemorrhagic fever have high concentrations of the virus in their blood.
Blood tests known as enzyme-linked immunosorbent assay (ELISA) and reverse transcriptase polymerase
chain reaction (PCR) can detect specific genes or the virus or antibodies to them. No antiviral medications
have proved effective in treating the Ebola virus or Marburg virus infections. As a result, treatment consists of
supportive hospital care; this includes providing fluids, maintaining adequate blood pressure, replacing blood
loss and treating any other infections that develop.

As with other infectious diseases, one of the most important preventive measures for the Ebola virus and
Marburg virus is frequent hand-washing. Use soap and water, or use alcohol-based hand rubs containing at
least 60 percent alcohol when soap and water aren't available. In developing countries, wild animals, including
nonhuman primates, are sold in local markets; avoid buying or eating any of these animals. In particular,
caregivers should avoid contact with the person's bodily fluids and tissues, including blood, semen, vaginal
secretions and saliva. People with Ebola or Marburg are most contagious during the later stages of the disease.
If you're a healthcare worker, wear protective clothing — such as gloves, masks, gowns and eye shields - keep
infected people isolated from others. Carefully disinfect and dispose of needles and other instruments; injection
needles and syringes should not be reused. Scientists are working on a variety of vaccines that would protect
people from Ebola or Marburg viruses. Some of the results have been promising, but further testing is needed.

Text 2: A Chronic Disease - Atopic Dermatitis

Atopic dermatitis is a common chronic skin disease. It is also called atopic eczema. Atopic is a term used to
describe allergic conditions such as asthma and hay fever. Both dermatitis and eczema mean inflammation of
the skin. People with atopic dermatitis tend to have dry, itchy and easily irritated skin. They may have times

facebook.com/HonorsGroup
READING SUB-TEST

when their skin is clear and other times when they have rash. In infants and small children, the rash is often
present on the skin around the knees and elbows and the cheeks. In teenagers and adults, the rash is often
present in the creases of the wrists, elbows, knees or ankles, and on the face or neck.

Atopic dermatitis usually begins and ends during childhood, but some people continue to have the disease into
adulthood. If you have ever had atopic dermatitis, you may have trouble with one or more of these: dry,
sensitive skin, hand dermatitis and skin infections.The exact cause of atopic dermatitis is unknown. Research
suggests that atopic dermatitis and other atopic diseases are genetically determined; this means that you are
more likely to have atopic dermatitis, food allergies, asthma and/or hay fever if your parents or other family
members have ever had atopic dermatitis. These diseases may develop one after another over a period of years.
This is called the “atopic march”.

Knowing that a child with a slight wheeze has had a history of atopic dermatitis, for example makes it easier to
diagnose the subtle onset of asthma. There are many things that make the itching and rash of atopic dermatitis
worse. When you learn more about atopic dermatitis and how to avoid things that make it worse, you may be
able to lead a healthier life.

If you have a reaction to something you touch, breathe or eat, you might have an allergy. Allergies can trigger
or worsen your atopic dermatitis symptoms. Common causes of allergy are: dust mites, furry and feathered
animals, cockroaches, pollen, mold, foods, chemicals. Your healthcare provider may recommend allergy
testing and food challenges to see if allergies worsen itching or rashes. Allergy testing may include skin
testing, blood tests or patch tests. Many measures can be taken to avoid things to which you are allergic.
Although many of the measures can be done for the entire home, the bedroom is the most important room to
make skin friendly. Talking with healthcare provider about what measures you can take to avoid your allergens
can be very beneficial.

Food allergies may be the cause of itching or rashes that occur immediately after eating, especially in children.
Some common food allergens include milk, eggs, peanuts, wheat, nuts, soy and seafood. Most people are
allergic to only one, two or at the most three foods. Be aware that diet restrictions can lead to poor nutrition
and growth delay in babies and children. Talk with your healthcare provider about maintaining a well-balanced
diet.

facebook.com/HonorsGroup
READING SUB-TEST

Emotions and stress do not cause atopic dermatitis, but they may bring on itching and scratching. Anger,
frustration and embarrassment can cause flushing and itching. Day to day stresses as well as major stressful
events can lead to or worsen the itch-scratch cycle.The medications used in atopic dermatitis include topical
steroids, topical immunomodulators, tar products, anti-infectives and antihistamines. Steroid medicines that are
applied to the skin are called topical steroids. Topical steroids are drugs that fight inflammation; they are very
helpful when a rash is not well controlled. Topical steroids are available in many forms such as ointments,
creams, lotions and gels. It is important to know that topical steroids are made in low to super potent
strengths. Steroid pills or liquids, like prednisone, should be avoided because of side effects and because the
rash often comes back after they are stopped.

facebook.com/HonorsGroup
READING SUB-TEST

Text 1: Questions 7-14

7 The Ebola and Marburg Viruses are native to;

America

Japan

Africa

China

8 What is right about Ebola and Marburg viruses?

Spread from person to person only.

Spread from animals to humans.

Spread from animals to animals.

Spread person to person after initial transmission from the infected animals.

9 Symptoms are typically seen within;

Five days

Ten days

Five to seven days

Five to ten days

10 In the Philippines, Ebola was discovered in;

Chimpanzees

Human primates

facebook.com/HonorsGroup
READING SUB-TEST

Non-human primates

Monkeys

11 Most known Ebola diseases occur due to;

Contamination

Bodily fluids

Contaminated needles and syringes

None

12 People with hemorrhagic fever show;

High number of viruses in their blood

Low concentrations of virus

High concentrations of antibodies

Low concentrations of antibodies

13 Pick one of the best preventive measures stated in the passage here;

Hand cleaning with medicinal soap.

Use of alcohol-based hand rubs, containing at least 60% alcohol, in absence of water & soap.

Only use of soap.

Avoiding direct contact with patients is a necessity.

14 As a healthcare worker, you should;

Keep infected people totally isolated from others.

facebook.com/HonorsGroup
READING SUB-TEST

Not reuse needles and syringes for the second time.

Wear clothing such as gowns and eye shields.

none of the above

Text 2: Questions 15-22

15 People with atopic dermatitis suffer from;

Hay fever

Asthma

Dry, itchy and irritated skin

Rashes

16 In small children, a rash is seen;

Around elbows

On the face

On the neck

Around the knees

17 People with atopic dermatitis have;

Dry skin

Skin infections

Hand dermatitis

All of the above

facebook.com/HonorsGroup
READING SUB-TEST

18 The term atopic refers to;

Allergic diseases

Asthma and hay fever.

Allergic conditions like hay fever.

Allergic conditions like asthma.

19 _____ can worsen dermatitis symptoms;

Allergies

Pollen

Dust

Mold

20 According to the information given in the passage, avoiding allergens is;

Easy

Difficult

Sometimes easy and sometimes difficult

Can’t say

facebook.com/HonorsGroup
READING SUB-TEST

21 “Allergic conditions like asthma in patients who have had a history of atopic dermatitis can be
easily diagnosed by health professionals,” this statement is;

Out of the paragraphs given

False

True

Can be true or can be false

22 Topical steroids are available in these forms:

Gel tubes

Ointments

Lotions

Ointments, creams, lotions and gels.

facebook.com/HonorsGroup
READING SUB-TEST

ANSWERS

PART A

1. Correct Answer Is: C


2. Correct Answer Is: D
3. Correct Answer Is: A
4. Correct Answer Is: A
5. Correct Answer Is: C
6. Correct Answer Is: A
7. Correct Answer Is: B
8. Correct Answer Is: autonomic nervous system
9. Correct Answer Is: freeze
10. Correct Answer Is: Ocular-Vestibular
11. Correct Answer Is: impulsivity
12. Correct Answer Is: Eustress
13. Correct Answer Is: asymmetrical tonic neck reflex
14. Correct Answer Is: visual processing disorder
15. Correct Answer Is: Saccades
16. Correct Answer Is: Bonding Response
17. Correct Answer Is: Babkin Palmomental
18. Correct Answer Is: Hypersensitive
19. Correct Answer Is: Inadequate Bonding
20. Correct Answer Is: Motor Programming

PART B & C

1. Correct Answer Is: AIDS-related deaths increased more significantly


2. Correct Answer Is: Preparing radix Sophorae tonkinensis samples.
3. Correct Answer Is: Trials performed produced negative results.
4. Correct Answer Is: Type A is known to produce more adverse reactions than all the others present in the
table.
5. Correct Answer Is: Study performed with focus on B12 usage.
6. Correct Answer Is: 32.72 patients were in the age group of 50-60 years.
7. Correct Answer Is: Africa
8. Correct Answer Is: Spread person to person after initial transmission from the infected animals.
9. Correct Answer Is: Five to ten days
10. Correct Answer Is: Monkeys
11. Correct Answer Is: Contaminated needles and syringes
12. Correct Answer Is: High number of viruses in their blood
13. Correct Answer Is: Use of alcohol-based hand rubs, containing at least 60% alcohol, in absence of water &
soap.
14. Correct Answer Is: Not reuse needles and syringes for the second time.
15. Correct Answer Is: Dry, itchy and irritated skin
16. Correct Answer Is: Around the knees
17. Correct Answer Is: All of the above
18. Correct Answer Is: Asthma and hay fever.
19. Correct Answer Is: Allergies
20. Correct Answer Is: Easy
21. Correct Answer Is: True
22. Correct Answer Is: Ointments, creams, lotions and gels.

facebook.com/HonorsGroup
READING SUB-TEST

OET 2.0 READING TASK 11

Part A

TIME: 15 minutes

Look at the four texts, A-D, in the separate Text Booklet.

For each question, 1-20, look through the texts, A-D, to find the relevant information.

Write your answers on the spaces provided in this Question Paper.

Answer all the questions within the 15-minute time limit.

Your answers should be correctly spelt.

Text A

Brain Imaging In Children

A headache is a result of disorders that affect pain-sensitive sites, such as meninges, blood vessels, paranasal
sinuses and muscles; it is also one of the most common causes of pain in children. The prevalence of severe
and frequent headaches is 25.25 per thousand in children. A large number of physicians and parents of sick
children are concerned that inter-carcinoma lesions may be due to headaches. The most important means to
diagnose the cause of headaches is to take precise medical history and para-clinical measures. Brain imaging
method, including CT scan and MRI, is one of the diagnostic methods for children with headaches. The main
reason for performing MRI and CT scans in patients with a headache is the diagnosis of curable lesions, which
can increase the patient’s longevity or improve her quality of life. Cases such as brain tumors, hydrocephalus
vascular malformations, and subdural hematoma are in this category. Another important cause for conducting
MRI in headache patients is a way to alleviate their anxiety due to the presence of a brain tumor or intracranial
disease.

facebook.com/HonorsGroup
READING SUB-TEST

Text B

Cases in which imaging should be done include important changes in the type of headache, headache
worsening, sudden development of headache or when it is stimulated by awakening of sleep, and when it is
associated with a neurological symptom. It has been reported that there are abnormal findings in the imaging
of patients, who are suspected of having a headache based on the findings of the study. Secondary pathologic
factors are more common than those in the general population of headache patients. Parents’ anxiety and their
concern about headaches in their children and the availability of imaging measures have caused most children
to experience CT scan and radiation due to the importance of headaches in children and due to limited studies
in under developing countries. Patients below 12 years of age were asked for, imaging actions, such as CT
scan and MRI, were the main inclusion criteria. The exclusion criteria included being older than 12 years of
age, lack of consent to enter the study, and defects in the medical records. Subsequently, the CT and MRI
reports of these patients were reviewed and the findings were recorded. Abnormal findings in the CT scan
include mass, cysts, infarcts, hydrocephalus, calcification, hematoma, ventricular dilation and edema.
Abnormal findings in MRI include sinusitis, retinal cysts, masses, cysts, atrophy, ventricular dilatation, age
variations, hydrocephalus, hematoma, demyelinating disease, mastoiditis, encephalomalacia, schizophyllum
and hypoplasia of corpus callosum, the prevalence of which were measured after collecting reports.

Text C

All patient information, including demographic factors and para-clinical symptoms, were recorded in a
researcher-made check list and entered into SPSS. In the descriptive part, abnormal CT scan and MRI findings
are presented as the main variable in different groups. All of the demographic and clinical characteristics of
patients were also reported based on descriptive criteria. In the analytical section, based on statistical
assumptions, parametric and nonparametric proportional tests were used. CHI-SQUARE test was used to
analyze the qualitative findings and independent T-test was used to compare quantitative data; non-parametric
Mann-Whitney was used if the initial assumptions were not as normal as they were supposed to. All tests were
examined at a 5% error level.

facebook.com/HonorsGroup
READING SUB-TEST

Text D

Total amount of 353 people were included in the study, of which 7 were excluded during the study. In the first
group, CT scan was performed on 217 patients, of which 85 subjects were girls and 132 were boys and 11.1%
were abnormal. In the second group, 136 people were subjected to MRI, of which 56 (41.1%) were female and
80 (58.8%) were male, and 24.3% were abnormal. Also, according to gender segregation, abnormal findings in
CT scan were significantly higher in boys (63% boys and 37% girls) (P = 0.03), and it was also found that
MRI findings were also significantly more common in boys in comparison to girls (66% vs. 34%) (P = 0.04).
The results of the study showed that the most common CT scan abnormal finding was mass (21%) and
hematoma (21%). Then, cysts (14%) and Ventriculomegaly (14%) were the most frequent forms of
abnormality. Prevalently found abnormality was cysts in MRI (30%). Atrophy (12%) and Ventriculomegaly
had the second and third frequencies (15%) (P> 0.05).

Questions 1-7

For each question, 1-7, decide which text (A, B, C or D) the information comes from. You may use any letter
more than once.

In which text can you find information about;

facebook.com/HonorsGroup
READING SUB-TEST

1 Research methodology.

Answer

2 Patients who underwent study.

Answer

3 Importance of detection of the causes in early stages.

Answer

4 Reasons why imaging shall be done.

Answer

5 Data segregation.

Answer

6 Condition in which there is an accumulation of cerebrospinal fluid (CSF) within the brain.

Answer

7 A small localized area of dead tissue resulting from failure of blood supply.

Answer

facebook.com/HonorsGroup
READING SUB-TEST

Questions 8-14

Answer each of the questions, 8-14, with a word or short phrase from one of the texts. Each answer may
include words, numbers or both.

8 Which age group patients were included in the study?

Answer

9 What is the reason presented for taking MRI in children with headache?

Answer

10 What is define as inflammation of the air cavities within the passages of the nose?

Answer

11 Was any defect in medical records an inclusion criteria?

Answer

12 Who shows more peculiarities in CT imaging?

Answer

13 What method was followed if the initial assumptions were abnormal?

Answer

facebook.com/HonorsGroup
READING SUB-TEST

14 What is known to be more common with respect to CT scanning?

Answer

Questions 15-20

Complete each of the sentences, 15-20, with a word or short phrase from one of the texts. Each answer may
include words, numbers or both.

15 is referred to as a solid swelling of clotted blood within the tissues.

16 The were presented in two sections of descriptive and analytical.

17 The percentage of boys who underwent CT scan is

18 can be defined as a brain condition that occurs in the fetus when the lateral ventricles
become dilated

19 A is any disease of the nervous system in which the myelin sheath of neurons is
damaged.

20 can be defined as the general physiological process of reabsorption and breakdown of


tissues.

facebook.com/HonorsGroup
READING SUB-TEST

Part B

In this part of the test, there are six short extracts relating to the work of health professionals. For questions 1-
6, choose the answer (A, B or C) which you think fits best according to the text.

Nutrition

Between 2003 to 2004 and 2016 to 2017 in the United States, the mean AHA healthy diet score improved in
both children and adults. The prevalence of an ideal healthy diet score increased from 0.2 percent to 0.6
percent in children and from 0.7 percent to 1.5 percent in adults. These improvements were largely attributable
to increased whole grain consumption and decreased sugar-sweetened beverage consumption in both children
and adults, as well as a small, non-significant trend in increased fruit and vegetable consumption. No major
trends were evident in children or adults in progress toward the targets for consumption of fish or sodium.
Between 2012 and 2017, although AHA healthy diet scores tended to improve in all race/ethnicity, income,
and education levels, there has been constant inequalities as well. with generally smaller improvements seen in
minority groups and those with lower income or education.

About one in every three US adults or 30.4 percent, do not engage in leisure time physical activity. Hispanic
and Non-Hispanic black adults were more likely to be inactive. Among students in grades 9-12, only about
27.1 percent meet the American Heart Association recommendation of 60 minutes of exercise every day. More
high school boys (36 percent) than girls (17.7 percent) reported having been physically active at least 60
minutes per day on all 7 days.

facebook.com/HonorsGroup
READING SUB-TEST

Acute Bilirubin Encephalopathy (ABE)

ABE is a term to describe the variable spectrum, from subtle to advanced manifestations of bilirubin toxicity
present in the first weeks of life (American Academy of Pediatrics Subcommittee on Hyperbilirubinemia,
2004; Van Praagh, 1961). Symptoms associated with ABE include a range of neurological
manifestations, sleeping disorder, somnolence, hypotonia, loss of the Moro reflex, followed by a stage
characterized by hypertonia of the extensor muscle groups (backward arching of the neck and backward
arching of the trunk). Additionally, fever and/or a high pitched cry may be present. Different investigators
have used different methods to characterize and define clinical manifestations of ABE in the infant.

Care For Women

Undoubtedly, encouraging women to give birth in health facilities, where there are skilled birth attendants, is
essential and has helped reduce global newborn and maternal mortality rates for decades. However, there is
room for improvement in the quality of care provided in these facilities.

To meet Sustainable Development Goal 3 of ensuring healthy lives and promoting well-being for all at all
ages, we cannot keep our focus solely on survival. High quality care for all pregnant women and their
newborns, throughout pregnancy, childbirth and the postnatal periods, is essential to ensure that mothers and
children both survive and thrive.

facebook.com/HonorsGroup
READING SUB-TEST

Wild polio

Overall the Committee was encouraged by continued progress in WPV1 eradication, with the number of cases
globally falling to an all-time low in 2017. In addition, there has been no international spread of WPV since
the fifteenth meeting in November 2017.

The Committee commended the continued high level commitment seen in sub continents and the high degree
of cooperation and coordination, particularly targeting the high risk mobile populations that cross the
international border, such as nomadic groups, local populations straddling the border, seasonal migrant
workers and their families, repatriating refugees (official and informal), and guest children (children staying
with relatives across the border). Stopping transmission in these populations is going beyond efforts and
cannot be underestimated, underlining the critical continuing need for cross border activities in surveillance
and vaccination.

facebook.com/HonorsGroup
READING SUB-TEST

ICD - Health Trends

ICD is the foundation for the identification of health trends and statistics globally, and the international
standard for reporting diseases and health conditions. It is the diagnostic classification standard for all clinical
and research purposes. ICD defines the universe of diseases, disorders, injuries and other related health
conditions, listed in a comprehensive, hierarchical fashion that allows for:

 easy storage, retrieval and analysis of health information for evidence-based decision-making;

 sharing and comparing health information between hospitals, regions, settings and countries; and

 data comparisons in the same location across different time periods.

Uses include monitoring of the incidence and prevalence of diseases, observing reimbursements and resource
allocation trends, and keeping track of safety and quality guidelines. They also include the counting of deaths
as well as diseases, injuries, symptoms, reasons for encounter, factors that influence health status, and external
causes of disease

facebook.com/HonorsGroup
READING SUB-TEST

Distribution of Ebola virus disease cases by health zone in Democratic Republic of the Congo, 1 April – 9 July
2018

Description Bikoro Iboko Wangata Total

Cases

New Suspected 3 2 0 5

New Probable 0 0 0 0

New Confirmed 0 0 0 0

Total New Cases 3 2 0 5

Cumulative Cases

Total Suspected 3 2 0 5

Total Probable 11 4 0 15

Total Confirmed 10 24 4 38

Total Number of Cases 24 30 4 58

facebook.com/HonorsGroup
READING SUB-TEST

1 According to the notice given

health rate improved from 2003-2017

many disparities present in 2012-2017 widened over time

increase in physical activity led to improved health score

2 The term which is more close to condition in which the child may reply instantly

somnolence

hypotonia

infantile reflex

3 The notice

places women at the centre of the care

explains what can be done in order to decrease mortality rate

talks of standards that ensure healthy life

4 The notice clearly explains that

disease control is a still a big problem

disease has successfully been managed

disease is widespread and is affecting population across the globe

facebook.com/HonorsGroup
READING SUB-TEST

5 The notice talks more about

how ICD works

ICD purpose and uses

actions undertaken by the ICD

6 What is correct?

Ebola virus is common among Waganta population

There has been widespread prevalence of the virus in Iboko

Virus in Bikoro is spreading at a rate closer to Waganta

facebook.com/HonorsGroup
READING SUB-TEST

Part C

In this part of the test, there are two texts about different aspects of healthcare. For questions 7-22, choose the
answer (A, B, C or D) which you think fits best according to the text.

Text 1: Viral Infection – Yellow Fever

Yellow fever is a viral infection spread by a particular species of mosquito. It's most common in the areas of
Africa and South America, affecting both travellers to and residents of those areas. In mild cases, it causes
fever, headaches, nausea and vomiting. However, it can become more serious, causing heart, liver and kidney
problems along with bleeding (haemorrhaging). Up to 50 percent of people with the more severe form of
yellow fever die of the disease. There's no specific treatment for yellow fever, but getting a yellow fever
vaccine before travelling to an area in which the virus is known to exist can protect you from the disease.
During the first three to six days after contracting yellow fever — the incubation period — there won’t be any
signs or symptoms of the disease. After this, the virus enters an acute phase and, in some cases, a toxic phase
follows which can be life-threatening.

Once the yellow fever virus enters the acute phase, you may experience signs and symptoms including: fever,
headaches, muscle aches - particularly in your back and knees - nausea, vomiting or both, loss of appetite,
dizziness, red eyes, face or tongue. These signs and symptoms usually improve and disappear within several
days. Although signs and symptoms may disappear for a day or two following the acute phase, some people
with acute yellow fever then enter a toxic phase. During the toxic phase, acute signs and symptoms return and
more severe and life-threatening ones also appear. These can include yellowing of the skin and the whites of
the eyes (jaundice), abdominal pain and vomiting - sometimes of blood - decreased urination, bleeding from
your nose, mouth and eyes, heart dysfunction (arrhythmia), liver and kidney failure, and brain dysfunction,
including delirium, seizures and coma.

Make an appointment to see your doctor four to six weeks before travelling to an area in which yellow fever is
known to occur. If you don't have that much time to prepare, call your doctor anyway. Your doctor will help
you determine whether you need vaccinations and can provide general guidance on protecting your health
while abroad. Seek emergency medical care if you've recently travelled to a region where yellow fever is

facebook.com/HonorsGroup
READING SUB-TEST

known to occur and you develop severe signs or symptoms of the disease. Even if you develop mild
symptoms, call your doctor. Yellow fever is caused by a virus that is spread by the Aedes aegypti mosquito.
These mosquitoes thrive in and near human habitations where they can breed in even the cleanest water. Most
cases of yellow fever occur in sub-Saharan Africa and tropical South America.

Humans and monkeys are most commonly infected with the yellow fever virus; mosquitoes transmit the virus
back and forth between monkeys, humans or both. When a mosquito bites a human or monkey infected with
yellow fever, the virus enters the mosquito's bloodstream and circulates before settling in the salivary glands.
When the infected mosquito bites another monkey or human, the virus then enters the host's bloodstream,
where it may cause the illness to develop.

You may be at risk of the disease if you travel to an area where mosquitoes continue to carry the yellow fever
virus. These areas include sub-Saharan Africa and tropical South America. Even if there aren't current reports
of infected humans in these areas, it doesn't necessarily mean you're risk-free. It's possible that local
populations have been vaccinated and are protected from the disease, or that cases of yellow fever just haven't
been detected and officially reported. If you're planning on travelling to these areas, you can protect yourself
by getting a yellow fever vaccine at least 10 to 14 days before travelling. Anyone can be infected with the
yellow fever virus, but older adults are at greater risk of becoming seriously ill.

Diagnosing yellow fever based on signs and symptoms can be difficult because, early in its course, the
infection can be easily confused with malaria, typhoid, dengue fever and other viral hemorrhagic fevers. To
diagnose your condition, your doctor will likely: Ask questions about your medical and travel history, collect a
blood sample for testing. If you have yellow fever, your blood may reveal the virus itself. If not, blood tests
called enzyme-linked immunosorbent assay (ELISA) and polymerase chain reaction (PCR) can also detect
antigens and antibodies specific to the virus. Results from these tests may take several days. No antiviral
medications have proved helpful in treating yellow fever and, as a result, treatment consists primarily of
supportive care in a hospital. This includes providing fluids and oxygen, maintaining adequate blood pressure,
replacing blood loss, providing dialysis for kidney failure, and treating any other infections that develop. Some
people receive transfusions of plasma to replace blood proteins that improve clotting. If you have yellow fever,
you may also be kept away from mosquitoes, to avoid transmitting the disease to others.

facebook.com/HonorsGroup
READING SUB-TEST

Text 2: Aortic Dissection or Dissecting Aneurysm

An aortic dissection is a serious condition in which a tear develops in the inner layer of the aorta, the large
blood vessel branching off the heart. Blood surges through this tear into the middle layer of the aorta, causing
the inner and middle layers to separate (dissect). If the blood-filled channel ruptures through the outside aortic
wall, aortic dissection can be fatal. Aortic dissection, also called a dissecting aneurysm, is relatively
uncommon. Anyone can develop the condition, but it most frequently occurs in men between 60 and 70 years
of age. Symptoms of aortic dissection may mimic those of other diseases, often leading to delays in diagnosis.
However, when an aortic dissection is detected early and treated promptly, your chance of survival greatly
improves.

Aortic dissection symptoms may be similar to those of other heart problems, such as a heart attack. Typical
signs and symptoms include sudden severe chest or upper back pain (often described as a tearing, ripping or
tearing sensation, that radiates to the neck or down the back), loss of consciousness (fainting), shortness of
breath, sweating, weaker pulse in one arm compared to the other etc. If you have signs or symptoms such as
severe chest pain, fainting, sudden onset of shortness of breath or symptoms of a stroke then seeking medical
assistance is imperative. While experiencing such symptoms doesn't always mean that you have a serious
problem, it's best to get checked out quickly because early detection and treatment may help to save your life.

An aortic dissection occurs in a weakened area of the aortic wall. Chronic high blood pressure may stress the
aortic tissue, making it more susceptible to tearing. You can also be born with a condition associated with a
weakened and enlarged aorta, such as Marfan syndrome or bicuspid aortic valve. Rarely, aortic dissections
may be caused by traumatic injury to the chest area, such as during motor vehicle accidents.

Aortic dissections are divided into two groups, depending on which part of the aorta is affected: Type A: This
is the more common and dangerous type of aortic dissection. It involves a tear in the part of the aorta just
where it exits the heart or a tear extending from the upper to lower parts of the aorta, which may extend into
the abdomen. Type B: This type involves a tear in the lower aorta only, which may also extend into the
abdomen. Risk factors for aortic dissection include Uncontrolled high blood pressure (hypertension), found in
at least two-thirds of all cases, Hardening of the arteries (atherosclerosis), Weakened and bulging artery (pre-

facebook.com/HonorsGroup
READING SUB-TEST

existing aortic aneurysm), aortic valve defect (bicuspid aortic valve), A narrowing of the aorta you're born with
(aortic coarctation)

People with certain genetic diseases are more likely to have an aortic dissection than other people. These
genetic diseases include Turner's syndrome, high blood pressure, heart problems, and a number of other health
conditions may be a result of this disorder. Marfan syndrome: This is a condition in which connective tissue,
which supports various structures in the body, is weak. People with this disorder often have a family history of
aneurysms of the aorta and other blood vessels. These weak blood vessels are prone to tears (dissection) and
rupture easily. Ehlers-Danlos syndrome: This group of connective tissue disorders is characterized by skin that
bruises or tears easily, loose joints and fragile blood vessels. Loeys-Dietz syndrome: This is a connective tissue
disorder marked by twisted arteries, especially in the neck. People who have Loeys-Dietz syndrome are
thought to be at risk of developing aortic dissections and aneurysms.

An aortic dissection can lead to death, due to severe internal bleeding, including into the lining around the
heart (pericardial sac), organ damage (such as kidney failure or life-threatening damage to the intestines),
strokes (possibly including paralysis), and aortic valve damage, such as causing the aortic valve to leak (aortic
regurgitation). Detecting an aortic dissection can be tricky because the symptoms are similar to those of a
variety of health problems. Doctors often suspect an aortic dissection if the following signs and symptoms are
present: sudden tearing or ripping chest pain, widening of the aorta on a chest X-ray, blood pressure difference
between the right and left arms.

facebook.com/HonorsGroup
READING SUB-TEST

Text 1: Questions 7-14

7 Yellow fever is common in;

Africa

South America

Both

Not given

8 Which of the following is not a sign of yellow fever?

Back pain

Vomiting

Nausea

Dry tongue

9 Signs/symptoms of toxic phase;

Loss of appetite

Yellowness of eyes

Brain dysfunction

B and C

facebook.com/HonorsGroup
READING SUB-TEST

10 Seizures may occur during;

Acute phase

Toxic phase

Sometimes in both the phases

Not given

11 Yellow fever, which is a viral disease, is spread by;

Aedes albopictus mosquito

Aedes aegypti mosquito

Female aedes mosquito

Male aedes mosquito

12 Mosquitoes transmit the virus from

Humans to monkeys

Monkeys to humans

Human to human

A&B

13 Travelling to areas where the disease is common is recommended after vaccination of;

facebook.com/HonorsGroup
READING SUB-TEST

10 days

12 days

14 days

10-14 days

14 Supportive care in hospitals includes;

Free medical check up

Free BP check up

Effective dialysis procedures in case of kidney failure

B and C

Text 2: Questions 15-22

15 In aortic dissection a tear develops in;

Outer layer of aorta.

Inner layer of aorta.

Middle aorta.

A blood vessel branching off the heart.

16 Dissecting aneurysm is common among;

Men

Women

Both

facebook.com/HonorsGroup
READING SUB-TEST

Children

17 Symptoms of aortic dissection include;

Chest pain and swelling

Weak pulse in both arms

Loss of consciousness.

All of the above

18 Aortic dissection can also be caused due to;

High BP

Weak aortic wall

Inborn symptoms

Traumatic injury to chest during accidents

19 The most dangerous type of aortic dissection is;

Type A

Type B

Aortic aneurysm

Aortic coarctation

20 A condition in which connective tissue is weak is called;

Turners syndrome

Loeys-Dietz syndrome

facebook.com/HonorsGroup
READING SUB-TEST

Ehlers-Danlos syndrome

Marfans syndrome

21 People with Loeys-Dietz syndrome are likely to develop;

Aneurysms

Ruptured blood vessels

Twisted arteries in the neck

Aortic complications

22 Detecting aortic dissection is;

Easy

Difficult

Impossible

Sometimes possible

facebook.com/HonorsGroup
READING SUB-TEST

ANSWERS

Part A

1. Correct Answer Is: C


2. Correct Answer Is: D
3. Correct Answer Is: A
4. Correct Answer Is: B
5. Correct Answer Is: C
6. Correct Answer Is: A
7. Correct Answer Is: B
8. Correct Answer Is: below twelve years
9. Correct Answer Is: diagnosis of curable lesions
10. Correct Answer Is: sinusitis
11. Correct Answer Is: no
12. Correct Answer Is: boys
13. Correct Answer Is: non-parametric Mann-Whitney
14. Correct Answer Is: mass and hematoma
15. Correct Answer Is: Hematoma
16. Correct Answer Is: Statistical Analyzes
17. Correct Answer Is: 61%
18. Correct Answer Is: Ventriculomegaly
19. Correct Answer Is: Demyelinating Disease
20. Correct Answer Is: Atrophy

Part B & C

1. Correct Answer Is: many disparities present in 2012-2017 widened over time
2. Correct Answer Is: infantile reflex
3. Correct Answer Is: places women at the centre of the care
4. Correct Answer Is: disease control is a still a big problem
5. Correct Answer Is: ICD purpose and uses
6. Correct Answer Is: There has been widespread prevalence of the virus in Iboko
7. Correct Answer Is: Both
8. Correct Answer Is: Dry tongue
9. Correct Answer Is: B and C
10. Correct Answer Is: Toxic phase
11. Correct Answer Is: Aedes aegypti mosquito
12. Correct Answer Is: A & B
13. Correct Answer Is: 10-14 days
14. Correct Answer Is: B and C
15. Correct Answer Is: Inner layer of aorta.
16. Correct Answer Is: Men
17. Correct Answer Is: Loss of consciousness.
18. Correct Answer Is: Traumatic injury to chest during accidents
19. Correct Answer Is: Type A
20. Correct Answer Is: Marfans syndrome
21. Correct Answer Is: Aneurysms
22. Correct Answer Is: Difficult

facebook.com/HonorsGroup
READING SUB-TEST

OET 2.0 READING TASK 12


Part A

TIME: 15 minutes

 Look at the four texts, A-D, in the separate Text Booklet.


 For each question, 1-20, look through the texts, A-D, to find the relevant information.
 Write your answers on the spaces provided in this Question Paper.
 Answer all the questions within the 15-minute time limit.
 Your answers should be correctly spelt.

Text A

Eczema / Dermatitis

The words `eczema` and `dermatitis` are often used synonymously to describe a polymorphic pattern of
inflammation, which in the acute phase is characterized by erythema and vesiculation, and in the chronic phase
by dryness, lichenification and fissuring. Contact dermatitis describes these patterns of reaction in response to
external agents, which may be acting either as irritants, where the T cell-mediated immune response is not
involved, or as allergens, where cell-mediated immunity is involved. Contact dermatitis may be classified into
the following reaction types: Subjective irritancy – idiosyncratic stinging and smarting reactions that occur
within minutes of contact, usually on the face, in the absence of visible changes. Cosmetic or sunscreen
constituents are common agents. Acute irritant contact dermatitis is often the result of a single overwhelming
exposure or a few brief exposures to strong irritants or caustic agents. Chronic irritant contact dermatitis
occurs following repetitive exposure to weaker irritants, which may be either `wet`, such as detergents, organic
solvents, soaps, weak acids and alkalis, or `dry`, such as low humidity air, heat, powders and dust.

facebook.com/HonorsGroup
READING SUB-TEST

Text B

Patch testing

The mainstay of diagnosis in allergic contact dermatitis is the patch test. This test has a sensitivity and
specificity of between 70% and 80%15. Patch testing involves the reproduction under the patch tests of
allergic contact dermatitis in an individual sensitized to a particular antigen(s). The standard method involves
the application of the antigen to the skin at standardized concentrations in an appropriate vehicle and under
occlusion. The back is most commonly used principally for convenience because of the area available,
although the limbs, in particular the outer upper arms, are also used. Various application systems are available
of which the most commonly used are Finn chambers. With this system, the investigator adds the individual
allergens to test discs that are loaded on to adhesive tape. Available are available – the TRUE and the Epiquic
tests. There are few comparative studies between the different systems. Pre-prepared tests are significantly
more reliable than operator-prepared tests. There is also some evidence that larger chambers may give more
reproducible tests. However, this may only apply to some allergens. The open patch test, not so common, is
used where potential irritants or sensitizers are being assessed. It is also useful in the investigation of contact
urticaria and protein contact dermatitis. The open patch test is usually performed on the forearm but the upper
outer arm or scapular areas may also be used. The site should be assessed at regular intervals for the first 30–
60 min and a later reading should be carried out after 3–4 days. A repeated open application test, applying the
suspect agent on to the forearm, is also useful in the assessment of cosmetics, where irritancy or combination
effects may interfere with standard patch testing. This usually involves the application of the product twice
daily for up to a week, stopping if a reaction develops.

facebook.com/HonorsGroup
READING SUB-TEST

Text C

Photopatch testing

Where photoallergic dermatitis is suspected, photopatch testing may be carried out. Very briefly, the standard
method of photopatch testing involves the application of the photo allergen series and any suspected materials
in duplicate on either side of the upper back. One side is irradiated with ultraviolet (UV) after an interval (1 or
2 days) and readings are taken in parallel after a further 2 days. The exact intervals for irradiation and the dose
of UVA given vary from centre to centre. The U.K. multicentre study into photopatch testing has now been
completed and published. It is recommended that allergens be subjected to 5 J cm2 UVA and a reading to be
taken after 2 days. The incidence of photoallergy in suspected cases was low at below 5%; however, further
readings at 3 and 4 days increased the detection rate.

Text D

There are a number of aspects, which can have their effect on the accuracy of patch testing. Principal among
these are the characteristics of the individual allergens and the method of patch testing. Some allergens are
more likely to cause irritant reactions than others. These reactions may be difficult to interpret and are easily
misclassified as positive reactions. Nickel, cobalt, potassium dichromate and carba mix are the notable
offenders in the standard series. As indicated above, pre-prepared patch tests are better standardized in terms
of the amount of allergen applied and are therefore more reproducible, but are prohibitively expensive in the
U.K. Patient characteristics are also important. It is essential that the skin on the back is free from dermatitis
and that skin disease elsewhere is as well controlled as possible. This will help to avoid the `angry back
syndrome` with numerous false positives. However, if a patient applies topical steroids to the back up to 2
days prior to the test being applied or is taking oral corticosteroids or immunosuppressant drugs, then there is a
significant risk of false negative results.

facebook.com/HonorsGroup
READING SUB-TEST

Questions 1-7

For each question, 1-7, decide which text (A, B, C or D) the information comes from. You may use any letter
more than once.

In which text can you find information about;

1 One of the most common tests.

Answer :

2 The blockage or closing of a blood vessel or hollow organ.

Answer :

3 Important factors that may influence the testing results.

Answer :

4 The condition that occurs in response to excessive itching or rubbing of the skin.

Answer :

5 Two different types of tests which help in diagnosing the disease condition.

Answer :

6 The process or fact of irradiating or being irradiated.

Answer :

facebook.com/HonorsGroup
READING SUB-TEST

7 Splitting or cracking of the skin.

Answer :

Questions 8-14

Answer each of the questions, 8-14, with a word or short phrase from one of the texts. Each answer may
include words, numbers or both.

8 Which test is known to be more helpful with respect to the determination of other irritants, which make it
difficult to carry out other tests?

Answer :

9 What are given orally or by injection and distribute throughout the body?

Answer :

10 In which process do we apply the product two times?

Answer :

11 What would be the outcome of the pre-prepared patch test, if there is dermatitis on the back of the skin?

Answer :

12 What are the common precipitants of contact dermatitis?

Answer :

13 What would be the outcome of the test, if the patient applies topical steroids two days prior to the test?

Answer :

facebook.com/HonorsGroup
READING SUB-TEST

14 What was the proportion of photoallergic incidents in suspected cases?

Answer :

Questions 15-20

Complete each of the sentences, 15-20, with a word or short phrase from one of the texts. Each answer may
include words, numbers or both.

15 as the cumulative irritant is known to create more worsening conditions.

16 The open patch is carried in order to examine the potentiality of the or irritants.

17 The chronic phase of a polymorphic pattern of inflammation is characterized by .

18 Carba mix is one of the in the standard series.

19 The standard method of testing involves the application of the .

20 As per recommendations, allergens be subjected to .

facebook.com/HonorsGroup
READING SUB-TEST

Part B

In this part of the test, there are six short extracts relating to the work of health professionals. For questions 1-
6, choose the answer (A, B or C) which you think fits best according to the text.

VKCFD

Inherited combined deficiency of the vitamin K dependent clotting factors is a very rare inherited bleeding
disorder that is caused by a problem with clotting Factors II, VII, IX, and X. VKCFD can also be acquired
later in life as a result of other disorders, or certain medications such as the blood-thinning drug Coumadin.
Acquired VKCFD is more common than the inherited form. Some newborn babies have a temporary vitamin
K deficiency, which can be treated with supplements at birth. In order to continue the chain reaction of the
coagulation cascade, these four factors need to be activated in a chemical reaction that involves vitamin K.
VKCFD is an autosomal recessive disorder, which means that both parents must carry the defective gene in
order to pass it on to their child. It also means that the disorder affects both males and females.

Factor I deficiency

It is an umbrella term for several related disorders in males and females, known as congenital Fibrinogen
defects. Afibrinogenemia (a lack of Fibrinogen) and hypofibrinogenemia (low levels of Fibrinogen) are
quantitative defects, meaning the amount of Fibrinogen in the blood is abnormal. Dysfibrinogenemia is a
qualitative defect in which Fibrinogen does not work the way it should. Hypodysfibrinogenemia is a combined
defect that involves both low levels of Fibrinogen and impaired function. Afibrinogenemia is an autosomal
recessive disorder, which means that both parents must carry the defective gene in order to pass it on to their
child. Hypofibrinogenemia, dysfibrinogenemia, and hypodysfibrinogenemia can be either recessive (both
parents carry the gene) or dominant (only one parent carries and transmits the gene).

facebook.com/HonorsGroup
READING SUB-TEST

2q37 deletion syndrome

It is a condition that can affect many parts of the body. This condition is characterized by hypotonia in infancy,
mild to severe intellectual disability and developmental delay, behavioral problems, characteristic facial
features, and other physical abnormalities.

Most babies with 2q37 deletion syndrome are born with potentially chronic fatigue, which usually improves
with age. About 25 percent of people with this condition have autism, a developmental condition that affects
communication and social interaction.

The characteristic facial features associated with 2q37 deletion syndrome include a prominent forehead, highly
arched eyebrows, deep-set eyes, a flat nasal bridge, a thin upper lip, and minor ear abnormalities. Other
features of this condition can include short stature, obesity, unusually short fingers and brachymesophalangy,
sparse hair, heart defects, seizures, and an inflammatory skin disorder called eczema. A few people with 2q37
deletion syndrome have a rare form of kidney cancer called Wilms tumor.

Sterile technique

Surgical asepsis is used to maintain sterilize. Use of effective sterile technique means that no organisms are
carried to the client. Microorganisms are destroyed before they can enter the body. Sterile technique is used
when changing dressings, administering parenteral (other than the digestive tract) medications, and performing
surgical and other procedures such as urinary catheterization. With surgical asepsis, first articles are sterilized,
and then their contact with any unsterile articles is prevented. When a sterile article touches an unsterile
article, it becomes contaminated. It is no longer sterile.

facebook.com/HonorsGroup
READING SUB-TEST

Treatment for HCV

Before the commencement of HCV treatment, it is necessary to genotype the virus as different genotypes
require different types and duration of treatment, and the protease inhibitors.

Current therapy for genotype 1 infection is a combination of PEG-IFN, RBV and a PI or nucleotide
polymerase inhibitor, which results in high rates of sustained virological response (SVR; a negative HCV
RNA test three or six months after the end of treatment). Boceprevir, simeprevir and telaprevir can also be
used. Dual therapy with PEG-IFN and RBV or sofosbuvir with RBV is used for genotypes 2 and 3 infections.
Patients with genotype 4 infection treated with treated with sofosbuvir, PEG-IFN and RBV have similar
response rates when compared with genotype 1-infected individuals. Small studies of genotypes 5- and 6-
infected patients have shown similar SVR rates to genotypes 2- and 3-infected ones.

Patients with genitourinary sarcomas

Patients with genitourinary sarcomas are relatively in a bad state, when compared with other soft tissue
regions. Prognosis is relatively poor and can be explained by the high proportion seen in high degree tumors, a
large proportion of patients with metastatic disease, large tumor and the area affected. In addition, the rarity
and heterogeneity of genitourinary sarcomas can explain the great variability in clinical progress in different
subgroups. Dissemination of urethral cancer follows the anatomic subdivision. The anterior urethra has a
lymphatic drainage system for superficial and deep inguinal region. Posterior urethra drains the lymphatic
ganglion of the external iliac artery, hypogastric, and internal obturator muscle. Late diagnosis is seen in one
third of patients with inguinal lymphatic ganglion metastasis and in 20% of those with pelvic ganglion
metastasis.

facebook.com/HonorsGroup
READING SUB-TEST

Questions 1-6

1 The notice talks about;

A How blood clots occur.

B Blood clot doesn`t form. (if the reaction doesn`t occur the way it should)

C Blocking of clotting reaction.

2 What is correct?

A All types of Factor I deficiency affect both males and females.

B The fibrogen defect in impairment leads to disorder.

C Genes can be both recessive or dominant.

3 What is referred to as ‘weak muscle’?

A Chronic fatigue

B Brachymesophalangy

C Hypotonia

4 The notice talks about;

A Every kind of procedure is used in the process of sterilization to keep sterile the objects or articles that
are to be introduced into a wound or body cavity or that is to penetrate the skin;

B General overview of sterilization techniques;

facebook.com/HonorsGroup
READING SUB-TEST

C The practices that the nurses will have to focus on;

5 It is known to prevent viral infection;

A PEG-IFN, RBV

B Boceprevir

C Sofosbuvir

6 The notice talks about;

A Rate of heterogeneity of genitourinary sarcomas.

B Survival rate and diagnosis.

C Survival rates of patients with genitourinary sarcomas.

facebook.com/HonorsGroup
READING SUB-TEST

Part C

In this part of the test, there are two texts about different aspects of healthcare. For questions 7-22, choose the
answer (A, B, C or D) which you think fits best according to the text.

Text 1: Cancer and Cervical Cancer

The body is made up of trillions of living cells. Normal body cells grow, divide, and die in an orderly fashion.
During the early years of a person`s life, normal cells divide faster to allow the person to grow. After the
person becomes an adult, most cells divide only to replace worn-out or dying cells or to repair injuries.

Cancer begins when cells in a part of the body start to grow out of control. There are many kinds of cancer, but
they all start due to out-of-control growth of abnormal cells. Cells become cancer cells because of damage to
DNA. DNA is in every cell and directs all its actions. In a normal cell, when DNA gets damaged the cell either
repairs the damage or the cell dies. In cancer cells, the damaged DNA is not repaired, but the cell doesn`t die
like it should. Instead, this cell goes on making new cells that the body does not need. These new cells will all
have the same damaged DNA as the first cell does.

No matter where cancer may spread, it is always named after the place where it started. For example, breast
cancer that has spread to the liver is still called breast cancer, not liver cancer. Likewise, prostate cancer that
has spread to the bone is metastatic prostate cancer, not bone cancer. Different types of cancer can behave very
differently. For example, lung cancer and breast cancer are very different diseases. They grow at different rates
and respond to different treatments. That is why people with cancer need the treatment that is aimed at their
particular kind of cancer. Not all tumors are cancerous. Tumors that aren`t cancer are called benign. Benign
tumors can cause problems – they can grow very large and press on healthy organs and tissues. But they
cannot grow into (invade) other tissues. Because they can`t invade, they also can`t spread to other parts of the
body (metastasize). These tumors are almost never life threatening.

The cervix is the lower part of the uterus (womb). It is sometimes called the uterine cervix. The body of the
uterus (the upper part) is where a baby grows. The cervix connects the body of the uterus to the vagina (birth
canal). The part of the cervix closest to the body of the uterus is called the endocervix. The part next to the
vagina is the exocervix (or ectocervix). The 2 main types of cells covering the cervix are squamous cells (on
the exocervix) and glandular cells (on the endocervix). The place where these 2 cell types meet is called the

facebook.com/HonorsGroup
READING SUB-TEST

transformation zone. Most cervical cancers start in the transformation zone and in the cells lining the cervix.
These cells do not suddenly change into cancer. Instead, the normal cells of the cervix first gradually develop
pre-cancerous changes that turn into cancer. Doctors use several terms to describe these pre-cancerous
changes, including cervical intraepithelial neoplasia (CIN), squamous intraepithelial lesion (SIL), and
dysplasia. These changes can be detected by the Pap test and treated to prevent the development of cancer.

Cervical cancers and cervical pre-cancers are classified by how they look under a microscope. There are 2
main types of cervical cancers: squamous cell carcinoma and adenocarcinoma. About 80% to 90% of cervical
cancers are squamous cell carcinomas. These cancers are from the squamous cells that cover the surface of the
exocervix. Under the microscope, this type of cancer is made up of cells that are like squamous cells.
Squamous cell carcinomas most commonly begin where the exocervix joins the endocervix.

Most of the other cervical cancers are adenocarcinomas. Cervical adenocarcinomas seem to have become more
common in the past 20 to 30 years. Cervical adenocarcinoma develops from the mucus-producing gland cells
of the endocervix. Less commonly, cervical cancers have features of both squamous cell carcinomas and
adenocarcinomas. These are called adenosquamous carcinomas or mixed carcinomas. Although cervical
cancers start from cells with pre-cancerous changes (pre-cancers), only some of the women with pre-cancers of
the cervix will develop cancer. The change from cervical pre-cancer to cervical cancer usually takes several
years, but it can happen in less than a year. For most women, pre-cancerous cells will go away without any
treatment. Still, in some women pre-cancers turn into true (invasive) cancers. Treating all pre-cancers can
prevent almost all true cancers.

facebook.com/HonorsGroup
READING SUB-TEST

Text 2: Hepatitis – Viral Liver Infection

Hepatitis (A, B, or C) can be caused by a virus (Viral Hepatitis), drugs, alcohol, medications, and blood
transfusions. Scientists estimate that between 3.5 and 5.3 million people in the USA are living with Hepatitis.
A blood test is required to diagnose Hepatitis infection. Hepatitis A is a viral liver infection. In most cases the
body easily defeats the virus (much like the flu, which is what you may feel like you have). Because of this it
does not lead to long term liver challenges. Hepatitis A is the most common form of Hepatitis. It is spread
through the feces of a contaminated person. This can easily be prevented by thoroughly washing hands after
using the restroom, before eating, and after changing a diaper.

Eating raw oysters and undercooked clams can increase your chances of contracting the virus. If you are
traveling in a country where Hepatitis is common make sure you wash your hands often and well, eat cooked
oysters and clams, and use an antiviral essential oil such as Lemon to help protect yourself. Hepatitis B is a
viral liver infection. Again, most adult bodies are able to fight off the virus. In this case, it is referred to as
Acute (something that does not last long) Hepatitis B. Hepatitis B is spread through contact with blood or
bodily fluids of an infected person. This can include unprotected sexual intercourse, sharing drug needles,
getting a tattoo with instruments that were not properly cleaned, or by sharing a personal item such as a razor
or toothbrush with an infected person.

A mother who is infected can pass the virus on to her baby during delivery. Again, the symptoms are flu-like
in nature, so it often goes undiagnosed. A person who has Chronic (lasting three months or more) Hepatitis B
may show no symptoms until liver damage has occurred. Hepatitis B can lead to liver damage or cancer; your
doctor may want to do a biopsy to determine the amount of damage your liver has experienced. Hepatitis C is
also a viral liver infection. A few people will contract Hepatitis C and get better. This is called Acute Hepatitis
C. Most, however, will develop Chronic Hepatitis C and go on to deal with liver damage, cirrhosis of the liver,
liver cancer, and possibly liver failure. Hepatitis C is the number one reason for liver transplants in the USA.

Hepatitis C is spread through contact with contaminated blood. This can occur by sharing a needle, receiving a
blood transfusion or organ transplant (blood and organs have been screened for Hepatitis in the USA since
1992), getting a tattoo with equipment that has not been properly cleaned, and, in rare cases, a mother can pass
the virus on to her baby during birth. Scientists are not sure, but think there may be a slim possibility that the
virus may be passed through unprotected sexual intercourse.

facebook.com/HonorsGroup
READING SUB-TEST

Symptoms generally do not occur until the virus is causing damage. Again, the symptoms are flu-like; you
may also experience jaundice (yellowish eyes and skin) after the flu-like symptoms go away. Most people
discover they are infected by having routine tests done or by donating blood or organs and the standard tests
show the infection. There is also a home test you can purchase and do if you suspect you are infected.

If you are infected with a Hepatitis virus, or if you have been in the past, one of the most important things that
you can do is strengthen your liver. The easiest way to do this is the Be Young Liver Cleanse: in the morning,
take 1 drop of Be Young Lemon essential oil, 1 drop of Be Young Peppermint essential oil, and 1 teaspoon to
1 tablespoon of fresh lemon juice, followed by a glass of water. “Be Young essential oils” are absolutely 100%
pure, EOBBD tested and guaranteed to be free of synthetics and extenders. Do not try this with an essential oil
that you are not certain has been properly cared for and tested as you do not want to increase challenges to
your liver. When properly supported, the liver has a remarkable capacity for regeneration.

facebook.com/HonorsGroup
READING SUB-TEST

9 Cancer in liver having its origin in breast is a;

A Liver cancer

B Breast cancer

C Prostate cancer

D Bone cancer

8 The DNA in cancer cells is;

A Damaged, but can be repaired.

B Not totally damaged.

C Permanently damaged, but the cell doesn`t die.

D Damaged but creates new cells with damaged DNA.

Text 1: Questions 7-14

7 When a person becomes an adult, the cells divide only to;

A Replace old cells

B Replace dying cells

C Repair injuries

D All of the above

10 Benign tumors;

A Can grow and invade the tissues.

B Can grow and press on healthy organs and tissues.

facebook.com/HonorsGroup
READING SUB-TEST

C Are cancerous.

D Are highly fatal.

11 Most cervical cancers start in;

A Squamous cells;

B Glandular cells;

C The transformation zone;

D All the above;

12 There are ........... main types of cervical cancers

A two

B three

C four

D five

13 About 80 to 90% of cervical cancers are;

A Adenocarcinoma

B Adenosquamous carcinomas

C Mixed carcinomas

D Squamous cell carcinomas

facebook.com/HonorsGroup
READING SUB-TEST

14 Pre-cancerous cells in women can go away;

A With growth in more number of new cells.

B With medications from doctors.

C Without treatment

D Can`t say

Text 2: Questions 15-22

15 Hepatitis is caused by;

A Virus

B Alcohol consumption

C Medications

D All of the above

16 Which of the following spreads through feces of a contaminated person;

A Hepatitis A

B Hepatitis B

C Both

D Can`t say

17 Most adult bodies are able to fight off this virus;

facebook.com/HonorsGroup
READING SUB-TEST

A Hepatitis A virus

B Hepatitis B virus

C Both

D Can`t say

18 Hepatitis can lead to;

A Cancer

B Severe damage to the liver cells.

C Cancerous growth in the liver.

D Not given

19 In the USA people go for liver transplantation due to;

A Hepatitis A

B Hepatitis B

C Hepatitis C

D All of the above

20 Hepatitis C spreads through;

A Sharing needles

B Blood transfusion

C Organ transplantation

facebook.com/HonorsGroup
READING SUB-TEST

D All of the above

21 A patient may experience jaundice when;

A The flu-like symptoms appear.

B The flu-like symptoms disappear.

C Eyes become yellow.

D All of the above.

22 ‘Be Young essential oils’ are;

A Free from extenders

B EOBBD

C Full of synthetics

D Not given

facebook.com/HonorsGroup
READING SUB-TEST

ANSWERS
PART A

1. Correct Answer Is: C


2. Correct Answer Is: B
3. Correct Answer Is: D
4. Correct Answer Is: A
5. Correct Answer Is: B
6. Correct Answer Is: C
7. Correct Answer Is: A
8. Correct Answer Is: repeated open application test
9. Correct Answer Is: Corticosteroids
10. Correct Answer Is: repeated open application test
11. Correct Answer Is: false positive result
12. Correct Answer Is: cosmetic and sunscreen constituents
13. Correct Answer Is: false negative result
14. Correct Answer Is: below 5%
15. Correct Answer Is: Chronic irritant contact dermatitis
16. Correct Answer Is: sensitizers
17. Correct Answer Is: dryness, lichenification and fissuring
18. Correct Answer Is: notable offenders
19. Correct Answer Is: photo allergen series
20. Correct Answer Is: 5 J cm2 UVA

PART B & C

1. Correct Answer Is: B Blood clot doesn`t form. (if the reaction doesn`t occur the way it should)
2. Correct Answer Is: A All types of Factor I deficiency affect both males and females.
3. Correct Answer Is: C Hypotonia
4. Correct Answer Is: A Every kind of procedure is used in the process of sterilization to keep sterile the
objects or articles that are to be introduced into a wound or body cavity or that is to penetrate the skin;
5. Correct Answer Is: B Boceprevir
6. Correct Answer Is: C Survival rates of patients with genitourinary sarcomas.
7. Correct Answer Is: B Breast cancer
8. Correct Answer Is: D Damaged but creates new cells with damaged DNA.
9. Correct Answer Is: D All of the above
10. Correct Answer Is: B Can grow and press on healthy organs and tissues.
11. Correct Answer Is: C The transformation zone;
12. Correct Answer Is: A two
13. Correct Answer Is: D Squamous cell carcinomas
14. Correct Answer Is: B With medications from doctors.
15. Correct Answer Is: D All of the above
16. Correct Answer Is: A Hepatitis A
17. Correct Answer Is: B Hepatitis B virus
18. Correct Answer Is: A Cancer
19. Correct Answer Is: C Hepatitis C
20. Correct Answer Is: D All of the above
21. Correct Answer Is: B The flu-like symptoms disappear.
22. Correct Answer Is: A Free from extenders

facebook.com/HonorsGroup
READING SUB-TEST

OET 2.0 READING TASK 13


Part A

TIME: 15 minutes

 Look at the four texts, A-D, in the separate Text Booklet.


 For each question, 1-20, look through the texts, A-D, to find the relevant information.
 Write your answers on the spaces provided in this Question Paper.
 Answer all the questions within the 15-minute time limit.
 Your answers should be correctly spelt.

Text A

Atrial fibrillation (AF)

Atrial fibrillation (AF) is the most common cardiac arrhythmia. It affects >33 million individuals worldwide,
and its prevalence is projected to double by 2050. AF is associated with a 5- and 2-fold increased risk of stroke
and mortality, respectively. Furthermore, AF-related strokes are associated with higher morbidity, mortality,
and health care costs compared with non-cardioembolic strokes.

The mainstay of stroke prevention remains oral anticoagulation (OAC), with vitamin K antagonists and more
recently, direct oral anticoagulants (DOAC), reducing the risk of ischemic stroke and all-cause mortality in
patients with AF. However, more than one-third of AF patients at high risk for stroke still fail to receive
effective stroke prophylaxis in contemporary practice. Although the introduction of DOAC has overcome
some of the limitations of warfarin therapy, persistent barriers including costs, ongoing bleeding risks with no
reversal agent for most DOACs, noncompliance and high discontinuation rates may preclude a broader use of
DOAC in clinical practice.

facebook.com/HonorsGroup
READING SUB-TEST

Text B

The left atrial appendage (LAA) is a remnant of the embryonic left atrium and is considered the main reservoir
for left atrial thrombi in >90% of patients with nonvalvular AF. In recent years, percutaneous LAA closure
(LAAC) has rapidly grown worldwide as an appealing alternative for the prevention of thromboembolism in
patients at high risk for stroke, with a specific focus on patients ineligible for OAC. While no specific
recommendation on LAAC was given in the 2014 American guidelines, the 2016 European guidelines for the
management of AF provided a class IIb recommendation for percutaneous LAAC in patients with AF and
contraindications for long-term OAC, based on data from the PROTECT-AF and PREVAIL trials, the only
LAAC randomized trials to date. Although none of these studies included patients ineligible for OAC, most of
the real-world registries conducted to date have focused on this target population, which currently represents
the majority of LAAC recipients. This review provides an updated overview of current transcatheter LAAC
devices and reviews the main clinical data from LAAC randomized trials and registries, focusing on
procedural and late outcomes, as well as on future directions.

Text C

facebook.com/HonorsGroup
READING SUB-TEST

WATCHMAN and WATCHMAN FLX

The WATCHMAN device (Boston Scientific, Natick, MA) was the second dedicated LAAC device and
remains the only device studied in randomized clinical trials to date. It consists of a self-expanding nitinol 10-
strut frame, with a 160 μm permeable polyethylene terephthalate membrane fabric cap facing the left atrium.
The open distal end is fixed by 10 active fixation anchors in 1 row. The tool will be of different dimensions 21,
24, 27, 30, and 33 mm. The transseptal access sheath will have crucial specifications, and may show 14F with
respect to width or breadth and is available in 3 different preformed curve shapes: anterior curve, double curve
(used in >90% of procedures), and single curve. Three proximal radio-opaque markers correspond to the
approximate level of deployment for 21, 27, and 33 device sizes, respectively.

Text D

The transseptal puncture is performed under fluoroscopic and preferably transesophageal echocardiography
(TEE; bicaval view followed by short-axis view) guidance in the inferoposterior portion of the fossa ovalis.
After transseptal puncture, a long extra-stiff J tipped 0.035-inch wire is advanced into the left upper pulmonary
vein and the transseptal sheath is exchanged over the wire for the access sheath. After removal of dilator and
guidewire, a 5F to 6F pigtail catheter is advanced through the access sheath into the left upper pulmonary vein.
By using TEE and fluoroscopic guidance, it will be easy to create some access. In the stage pertaining to it,
access sheath and pigtail are adjusted as required. Appropriate WATCHMAN device sizing is determined by
the maximum LAA ostium diameter (measured from the circumflex artery to 1-2 cm within the pulmonary
vein ridge at 0°, 45°, 90°, and 135°) and depth (from ostium to the tip of LAA). An oversizing of the device by
10 to 20 percent (corresponding to 2–4 mm) is generally recommended. A fluoroscopic right (20°–30°) and
caudal (20°–30°) projection, which usually opens the mid-distal portion of the LAA is the preferred one for the
deployment of the WATCHMAN device. After accurate LAA assessment, the delivery system is advanced
into the access sheath until the distal markers of the delivery catheter and the access sheath align. The device is
then deployed with a slow unsheathing movement.

facebook.com/HonorsGroup
READING SUB-TEST

Questions 1-7

For each question, 1-7, decide which text (A, B, C or D) the information comes from. You may use any letter
more than once.

In which text can you find information about;

1 Used for accessing the left side of the heart.

Answer :

2 Functioning of the device.

Answer :

3 A comparison with another disease.

Answer :

4 A drug that helps with blocking and managing the disease condition.

Answer :

5 Structure of the device.

Answer :

6 Made, done, or effected through the skin.

Answer :

facebook.com/HonorsGroup
READING SUB-TEST

7 Details about LAA imaging and LAAC closure techniques are simply beyond the study performed.

Answer :

Questions 8-14

Answer each of the questions, 8-14, with a word or short phrase from one of the texts. Each answer may
include words, numbers or both.

8 What is generally recommended with respect to device sizing?

Answer :

9 When will the catheter be introduced into the left upper pulmonary vein?

Answer :

10 In how many sizes, the WATCHMAN device is available?

Answer :

11 What will be the outer diameter of the transseptal access sheath?

Answer :

12 What is recommended for the effective use of the Watchman Device?

Answer :

13 What is taken into account to measure the size of the device?

Answer :

facebook.com/HonorsGroup
READING SUB-TEST

14 What is the study more focused on?

Answer :

Questions 15-20

Complete each of the sentences, 15-20, with a word or short phrase from one of the texts. Each answer may
include words, numbers or both.

15 The best defending procedure is more connected to the use of the .

16 All three are needed to be adjusted as per the levels of the device deployed.

17 With access sheath is rightly repositioned into the LAA.

18 With DOAC being introduced, the scope for the enhanced more.

19 The recommended oversizing of the device is in range of .

20 In its final stage, the device will be deployed with .

facebook.com/HonorsGroup
READING SUB-TEST

Part B

In this part of the test, there are six short extracts relating to the work of health professionals. For questions 1-
6, choose the answer (A, B or C) which you think fits best according to the text.

Nursing care of the heart transplant patient

Nursing care of the heart transplant patient is similar to the care of any cardiac surgery patient. Bleeding is a
major concern in the early postoperative period. Chest tube drainage is frequently monitored (initially every 15
minutes), as are the cardiac output, pulmonary artery pressures, and CVP. Cardiac tamponade can develop,
presenting as either a sudden event or a gradual process. Chest tubes are gently milked (not stripped) as needed
to maintain patency. Atrial dysrhythmias are relatively common following cardiac transplant. Temporary
pacing wires are placed during surgery because surgical manipulation or postoperative swelling may disrupt
the conduction system. Hypothermia is induced during surgery; postoperatively, the patient is gradually re-
warmed over a 1- to 2-hour period. Cardiac function is impaired in up to 50% of transplanted hearts during the
early postoperative period. Inotropic agents such as low-dose dopamine, dobutamine, or milrinone may be
required to bring more stability.

Cardiology Conference

With people from around the world focused on getting some answers concerning Cardiology, this is your
single most obvious opportunity to accomplish the greatest accumulation of individuals from the mending
focuses, Universities, bunch, etc. This Cardiology conference rather European Cardiology Congress in 2017
will coordinate appears, disperse information, meet with recurring pattern and potential investigators and get
name affirmation at this 3-day event. Broadly acclaimed speakers, the most recent frameworks,
methodologies, and the most current updates in Cardiology field are indications of this conference. This World
Cardiology Congress will help in frameworks organization, B2B uniting amidst specialists and academicians.

facebook.com/HonorsGroup
READING SUB-TEST

With Age Comes Weakness

The eyelids become lax with the rotation of lid margins as result flow of tear may be disrupted. The production
of tear from the lachrymal glands may decline to make the eyes dry and burning. There may be an
accumulation of fluid in the cornea turning it hazy. The muscles of iris are weak and the lens capsule becomes
stiff making it difficult to see small objects and depth perception accurately. The lens protein gets denatured
and solidifies. The anterior aspect of the uveal tract degenerates along with degeneration of important areas in
the retina.

DVT

The problem occurs when a part of the clot breaks off and travels through the bloodstream to the lungs,
causing a blockage called pulmonary embolism (PE). If the clot is small, and with appropriate treatment,
people can recover from PE. However, there could be some damage to the lungs. If the clot is large, it can stop
blood from reaching the lungs and is fatal. In addition, nearly one-third of people who have a DVT will have
long-term problems caused by the damage the clot does to the valves in the vein called post-thrombotic
syndrome (PTS). People with PTS have symptoms such as swelling, pain, discoloration, and in severe cases,
scaling or ulcers in the affected part of the body. In some cases, the symptoms can be so severe that a person
becomes disabled.

facebook.com/HonorsGroup
READING SUB-TEST

Medication Survey

In a survey of 598 communities in 18 countries, the availability of aspirin, β-blockers, ACEIs, and statins
varied widely In the low-income countries, only 1 of 30 rural and only 25 of 32 urban communities had all 4
medications available. The 4 medications were potentially unaffordable for 60% of households in low-income
countries. Availability rose to 37% (rural) and 62% (urban) in lower middle-income countries and 73% (rural)
and 80% (urban) in upper middle-income countries. Effective strategies for delivering medications are being
developed for LMIC where health system infrastructure is underdeveloped.

BiDil - Support Blacks

BiDil for in African Americans BiDil, a fixed-dose combination of two vasodilators (hydralazine and
isosorbide), is indicated as an adjunctive treatment in African Americans with heart failure. It has been shown
to reduce symptoms, decrease the number of hospitalizations, and prolong life in Blacks. The recommended
dose is one to two tablets three times per day, although the dose may be as low as 1/2 tablet three times a day
if side effects are intolerable. The approval of this combination drug has raised the ethical issue of race-
specific FDA approval.

facebook.com/HonorsGroup
READING SUB-TEST

Questions 1-6

1 As per the given notice, one of the major duties include;

A Care professional will have to look for ways to stop bleeding.

B It is important to maintain hemodynamic stability.

C Support cardiac function and circulation.

2 The given notice explains;

A Why medical professionals shall join the conference?

B What medical professionals will learn?

C How this is different from others.

3 The major result of the changes mention is

A inability to see / blindness

B ineffective accommodation and depth perception

C extra sensitivity to glare and dry eyes

4 The given notice talks about;

A Complications involved in DVT.

B Symptoms that lead to major problems.

C Result of the disease conditions.

facebook.com/HonorsGroup
READING SUB-TEST

5 According to the information given, what is correct?

A Only 40% of the household could have afforded the medications.

B 33% of households in lower middle-income countries managed to have used the mediations.

C 25% of households in upper middle-income countries got access to medications.

6 The given notice talks about;

A Bidil dosage use.

B Effectiveness of BiDil in treating heart failure.

C how it helped the community.

facebook.com/HonorsGroup
READING SUB-TEST

Part C

In this part of the test, there are two texts about different aspects of healthcare. For questions 7-22, choose the
answer (A, B, C or D) which you think fits best according to the text.

Text 1: What is Zika?

The Zika virus disease is caused by the Zika virus, which is spread to people primarily through the bite of an
infected mosquito (aedes aegypti and aedes albopictus). The illness is usually mild with symptoms lasting up
to a week, and many people do not have symptoms or will have only mild symptoms. However, a Zika virus
infection during pregnancy can cause a serious birth defect called microcephaly and other severe brain defects.

Zika is spread to people primarily through the bite of an infected aedes species mosquito (aedes aegypti and
aedes albopictus). A pregnant woman can pass Zika to her fetus during pregnancy or around the time of birth.
Also, a person with Zika can pass it to his or her sexual partners. We encourage people, who have traveled to
or live in places with Zika to protect themselves by preventing mosquito bites and sexual transmission of Zika.

Many people infected with Zika will have no symptoms or mild symptoms that last several days to a week.
However, a Zika infection during pregnancy can cause a serious birth defect called microcephaly and other
severe fetal brain defects. Current research suggests that Guillain-Barre syndrome (GBS), an uncommon
sickness of the nervous system, is strongly associated with Zika; however, only a small proportion of people
with a recent Zika virus infection get GBS. Once someone has been infected with Zika, it`s very likely they`ll
be protected from future infections. There is no evidence that past Zika infection poses an increased risk of
birth defects in future pregnancies.

Going to places where Zika is common is often not recommended. Travellers who go to places with outbreaks
of Zika may or may not get infected with Zika. Moreover, in pregnant women, if the virus is caught from such
places, it can cause microcephaly and other severe fetal brain defects.

Any pregnant women who have recently travelled to an area with Zika should talk to their doctor about their
travel, even if they don`t feel sick. Pregnant women should see a doctor if they have any Zika symptoms
during their trip or within 2 weeks after travelling. All pregnant women can protect themselves by using plenty

facebook.com/HonorsGroup
READING SUB-TEST

of prevention measures, which include but are not limited to: avoiding travel to an area with Zika; preventing
mosquito bites; and following recommended precautions against getting Zika. The most common symptoms of
the Zika virus disease are fever, rashes, joint pain, and red eyes. Other symptoms include muscle pain and
headaches. Many people infected with Zika won`t have symptoms or will have mild symptoms, which can last
for several days to a week.

Currently, there is no evidence that a woman who has recovered from the Zika virus infection (the virus has
cleared her body) will have Zika-related pregnancy complications in the future. Based on information about
similar infections, once a person has been infected with the Zika virus and has cleared the virus, he or she is
likely to be protected from future Zika infections. If you`re thinking about having a baby in the near future and
you or your partner live in or travelled to an area with Zika, talk with your doctor or another healthcare
provider. Men who have travelled to any areas with Zika or who have had a Zika infection should wait at least
6 months after travel (or 6 months after symptoms started if they get sick) before trying to conceive with their
partner. Women should wait at least 8 weeks after travel (or 8 weeks after symptoms started if they get sick)
before trying to get pregnant.

Text 2: Avian Influenza - A Virus Infections in Humans

Although avian influenza A viruses usually do not infect humans, rare cases of human infection with these
viruses have been reported. Infected birds shed the avian influenza virus in their saliva, mucus and feces.
Human infections with bird flu viruses can happen when enough of the virus gets into a person`s eyes, nose or
mouth, or is inhaled. This can happen when the virus is in the air (in droplets or possibly dust) and a person
breathes it in, or when a person touches something that has the virus on it and then touches their mouth, eyes
or nose. Rare human infections with some avian viruses have occurred most often after unprotected contact
with infected birds or surfaces contaminated with avian influenza viruses. However, some infections have been
identified where direct contact was not known to have occurred.

The reported signs and symptoms of low pathogenic avian influenza (LPAI) A virus infections in humans have
ranged from conjunctivitis to influenza-like illness (e.g., fever, cough, sore throat, muscle aches) to lower
respiratory disease (pneumonia) requiring hospitalization. Highly pathogenic avian influenza (HPAI) LPAI
H7N9 and HPAI Asian H5N1 have been responsible for most human illness worldwide to date, including the

facebook.com/HonorsGroup
READING SUB-TEST

most serious illnesses and deaths. A virus infections in people have been associated with a wide range of
illness from conjunctivitis only to severe respiratory illness (e.g. shortness of breath, difficulty breathing,
pneumonia, acute respiratory distress, viral pneumonia, respiratory failure) with multi-organ disease,
sometimes accompanied by nausea, abdominal pain, diarrhea, vomiting and sometimes neurologic changes
(altered mental status, seizures).

Detection of the avian influenza A virus infection in humans is a formidable process. It may be difficult to
assess the presence of the virus by clinical signs and symptoms alone; laboratory testing is required. The avian
influenza A virus infection is usually diagnosed by collecting a swab from the nose or throat of the sick person
during the first few days of illness. This specimen is sent to a lab; the laboratory looks for the avian influenza
A virus either by using a molecular test, by trying to grow the virus, or both. (growing avian influenza A
viruses should only be done in laboratories with high levels of protection). For critically ill patients, collection
and testing of lower respiratory tract specimens may lead to a diagnosis of avian influenza virus infection. For
some patients who are no longer very sick or who have fully recovered, it may be difficult to find the avian
influenza A virus in the specimen, using these methods. It is often termed a problem of gargantuan proportion.
Sometimes it may still be possible to diagnose the avian influenza A virus infection by looking for evidence of
the body`s immune response to the virus infection by detecting specific antibodies the body has produced in
response to the virus. This is not always an option because it requires two blood specimens (one taken during
the first week of the illness and another taken 3-4 weeks later).

Analyses of available avian influenza viruses circulating worldwide suggest that most viruses are susceptible
to oseltamivir, peramivir, and zanamivir. However, some evidence of antiviral resistance has been reported in
HPAI Asian H5N1 viruses and influenza A H7N9 viruses isolated from some human cases. Monitoring for
antiviral resistance among avian influenza A viruses is crucial and ongoing. This data directly informs WHO
antiviral treatment recommendations.

The best way to prevent an infection of the avian influenza A virus is to avoid sources of exposure. Most
human infections with avian influenza A viruses have occurred following direct or close contact with infected
poultry. People who have had contact with infected birds may be given expert-formulated influenza antiviral
drugs preventatively. While antiviral drugs are most often used to treat flu, they can also be used to prevent
infection in someone who has been exposed to influenza viruses. When used to prevent seasonal influenza,
antiviral drugs are 70% to 90% effective. A seasonal influenza vaccination will not prevent infection with

facebook.com/HonorsGroup
READING SUB-TEST

avian influenza A viruses, but can play hardball in effectively reducing the risk of co-infection with human and
avian influenza A viruses. It`s also possible to make a vaccine intellectually that can protect people against
avian influenza viruses. For example, the United States government always have access to the stockpiled
vaccine to protect against avian influenza A H5N1 vaccine. The stockpiled vaccine could be used if a similar
H5N1 virus were to begin transmitting easily from person to person. Creating a candidate vaccine virus is the
first step in producing a vaccine.

facebook.com/HonorsGroup
READING SUB-TEST

Text 1: Questions 7-14

7 According to paragraph 1, the Zika virus disease is;

A Common

B Uncommon

C Severe disease

D Without any symptoms

8 What does paragraph 2 indicate?

A How does Zika spread from person to person

B Modes of transmission of Zika

C How do people get infected with Zika?

DA&C

9 What is mentioned in paragraph 3?

A How do people get infected with Zika?

B What health problems can result from getting Zika?

C How microcephaly occurs.

D Microcephaly and birth defects.

10 According to paragraph 5, what is recommended for pregnant women?

A They should not travel to places where the Zika virus is common.

facebook.com/HonorsGroup
READING SUB-TEST

B They should talk to their partner before going to Zika-infected places.

C They should inform their doctor before travelling.

D Not given

11 In paragraph 5, how many methods of prevention from Zika are described?

A Plenty of prevention measures

B2

C3

D4

12 Can a person who is completely recovered from Zika virus infection, get Zika infection
again?

A. Yes

B. No

C. Depends from person to person

D. Not given

13 According to paragraph 6, which one of the following statements is true?

A A woman`s future pregnancies will be at risk if she is infected with the Zika virus.

B There is no evidence that infected women will not face any complications in future pregnancies.

C Any woman who is infected with Zika is advised to wait for six months before getting pregnant.

DB&C

facebook.com/HonorsGroup
READING SUB-TEST

14 According to paragraph 6, which of the following is not true?

A A woman should wait for six months before they get pregnant if they have traveled to any places with
the Zika virus.

B If the woman has been infected with the Zika virus and has cleared the virus, she is likely to be
protected from future Zika infections.

C In most of the cases, Zika complications may occur in future pregnancies.

D All above statements are true.

Text 2: Questions 15-22

15 According to paragraph 1, the virus enters the human body through.

A The nose

B The eyes

C The mouth

D Contact with an infected bird

16 According to paragraph 1, human infection with avian influenza A virus is;

A Uncommon

B Common

C Rare

D Frequent in infected places

17 According to paragraph 2, people may suffer from flu if they have been infected by

A LPAI

facebook.com/HonorsGroup
READING SUB-TEST

B HPAI

C H5N1

D LPAI H7N9

18 According to paragraph 3, what is not true?

A A molecular test is performed to see the presence of the virus in the collected specimen.

B Lab tests are aimed at growing the virus for the right detection purpose.

C A specimen for testing is usually collected right after the person gets the illness by infection.

D People who have recovered from the illness should also be diagnosed for further confirmation.

19 According to paragraph 3, the word ‘gargantuan’ may mean;

A Small-scale

B Trivial

C Very large

D Pint-sized

20 Which central idea is presented in paragraph 4?

A Treatment of the avian influenza disease.

B Effective medications for the viral disease.

C Research on effective medicine for the avian virus disease.

D None of the above.

facebook.com/HonorsGroup
READING SUB-TEST

21 According to paragraph 5, which word may give the meaning of experience?

A Exposure

B Expert

C Intellectualism

D None of the above

22 The phrase ‘to play hardball’ in paragraph 5 may mean;

A To be able to achieve the results.

B To do what it takes to complete the task.

C To work effectively.

D To be firm and determined in order to get what you want.

facebook.com/HonorsGroup
READING SUB-TEST

ANSWERS
PART A

1. Correct Answer Is: D


2. Correct Answer Is: D
3. Correct Answer Is: A
4. Correct Answer Is: A
5. Correct Answer Is: C
6. Correct Answer Is: B
7. Correct Answer Is: C
8. Correct Answer Is: Oversizing by 10 to 20 percent
9. Correct Answer Is: After removal of dilator and guidewire
10. Correct Answer Is: Five
11. Correct Answer Is: 14 F
12. Correct Answer Is: Fluoroscopic right and caudal projection
13. Correct Answer Is: Maximum diameter / LAA ostium diameter
14. Correct Answer Is: LAAC recipients
15. Correct Answer Is: oral anticoagulation (OAC)
16. Correct Answer Is: radio-opaque markers
17. Correct Answer Is: fluoroscopic guidance
18. Correct Answer Is: warfarin therapy
19. Correct Answer Is: 10% to 20%
20. Correct Answer Is: unsheathing movement

PART B & C
1. Correct Answer Is: C Support cardiac function and circulation.
2. Correct Answer Is: A Why medical professionals shall join the conference?
3. Correct Answer Is: C extra sensitivity to glare and dry eyes
4. Correct Answer Is: A Complications involved in DVT.
5. Correct Answer Is: A Only 40% of the household could have afforded the medications.
6. Correct Answer Is: B Effectiveness of BiDil in treating heart failure.
7. Correct Answer Is: C Severe disease
8. Correct Answer Is: C How do people get infected with Zika?
9. Correct Answer Is: B What health problems can result from getting Zika?
10. Correct Answer Is: D Not given
11. Correct Answer Is: C 3
12. Correct Answer Is: B. No
13. Correct Answer Is: A A woman`s future pregnancies will be at risk if she is infected with the Zika virus.
14. Correct Answer Is: C In most of the cases, Zika complications may occur in future pregnancies.
15. Correct Answer Is: B The eyes
16. Correct Answer Is: A Uncommon
17. Correct Answer Is: B HPAI
18. Correct Answer Is: D People who have recovered from the illness should also be diagnosed for further
confirmation.
19. Correct Answer Is: C Very large
20. Correct Answer Is: A Treatment of the avian influenza disease.
21. Correct Answer Is: A Exposure
22. Correct Answer Is: D To be firm and determined in order to get what you want.

facebook.com/HonorsGroup
READING SUB-TEST

OET 2.0 READING TASK 14


Part A

TIME: 15 minutes

Look at the four texts, A-D, in the separate Text Booklet.

For each question, 1-20, look through the texts, A-D, to find the relevant information.

Write your answers on the spaces provided in this Question Paper.

Answer all the questions within the 15-minute time limit.

Your answers should be correctly spelt.

Text A

The mechanism of polycythemia in primary familial and congenital polycythemia (PFCP) is due to the
truncated EpoR (genetic mutation) in which there is no inhibition of signalling pathways. In all conditions of
hypoxia HIF-1 is responsible for the polycythemia. Some patients with chronic lung disease or congenital
cyanotic heart disease do not develop polycythemia in spite of hypoxia, the mechanism of which is not very
clear. Polycythemia in smokers is due to increased blood carbon monoxide (CO). CO displaces one molecule
of O2 from hemoglobin and converts it to carboxyhemoglobin (COHb). COHb has 200 times greater affinity
than oxygen. This results in not only occupation of one of the heme groups of haemoglobin but also increase in
the oxygen affinity of the remaining heme group resulting in tissue hypoxia. Polycythemia accompanying
kidney and liver diseases and neoplastic disorders, is usually associated with increased EPO production. In
tumours EPO production is shown to be autonomous of hypoxic stimuli.

facebook.com/HonorsGroup
READING SUB-TEST

Text B

The molecular basis of post-transplantation erythrocytosis (PTE) remains unclear. It is found in 5-10% of renal
allograft recipients developing within 8-24 months following a successful renal transplantation. It resolves
spontaneously within 2 years in about 25% of patients. In congenital secondary polycythemia, mutations in the
haemoglobin can lead to increased oxygen affinity leading to decreased oxygen delivery and compensatory
polycythemia. A rare mechanism in this group is 2, 3 BPG deficiency. This compound is synthesised in red
blood cell and binds to haemoglobin reducing its affinity for oxygen. Its absence leads to increased affinity of
haemoglobin for oxygen resulting in a lifelong hypoxic stimulus and erythrocytosis. The fetal haemoglobin
has high oxygen affinity and many of the neonates may have markedly elevated hematocrits. Polycythemia
vera rises from the transformation of a single hematopoietic stem cell with a selective growth advantage that
gradually becomes the predominant myeloid progenitor. Recently a somatic mutation is detected in a gene on
chromosome 9p in a majority of polycythemia vera patients. This gene encodes for tyrosine kinase JAK. This
somatic mutation transforms this kinase into a constitutively active form and seems to be responsible for the
uncontrolled proliferation of the erythroid cells.

Text C

Clinical Approach

Symptoms of polycythemia are very nonspecific like a headache, weakness, pruritus, dizziness, sweating and
visual disturbances. Some of the patients are seen initially with complications of polycythemia like thrombosis
(cerebral, peripheral) and haemorrhage. Thrombosis may occur at unusual sites like hepatic vein.
Polycythemia may be diagnosed when Budd Chiari syndrome is being investigated. Hematocrit values above
51% in males and over 48% in females requires further evaluation.

Diagnostic criteria laid down by PVSG and WHO require demonstration of an elevated red cell mass as a
must. This is practically not possible in most centres. So, WHO has revised the criteria (2008) for the
diagnosis of PV6 .

facebook.com/HonorsGroup
READING SUB-TEST

Accordingly, there are 2 major and 3 minor criteria.

Major criteria

1. Hemoglobin level above 18.5g/dl for men and 16.5g/dl for females OR Hemoglobin or hematocrit > 99th
percentile of reference range for age, sex, or altitude of residence OR elevated red cell mass >25% above mean
normal predicted value.

2. Presence of JAK2 gene mutation (V617F) or other functionally similar.

Minor criteria

1. Bone marrow showing hypercellularity for age and trilineage growth (panmyelosis)

2. Subnormal Epo level

3. EEC (endogenous erythroid colonies)

Diagnostic combinations - Major criteria + one minor criterion and first major criterion + 2 minor criteria

Text D

Recommendations;

Low dose aspirin 75- 150 mg is recommended in all PV patients without history of major bleeding or gastric
intolerance, based on the results of the ECLAP study. Patients with PV should be properly hydrated when they
develop gastrointestinal disorders. The spent phase occurs after about 15-20 years, when the phlebotomy
requirement decreases and the patient develops anaemia. The marrow fibrosis increases and spleen becomes
greatly enlarged. The treatment during this phase is purely symptomatic including blood transfusions. Other
treatment modalities tried are splenectomy, thalidomide and marrow transplantation in younger patients. In the

facebook.com/HonorsGroup
READING SUB-TEST

future we may have new JAK2 targeted inhibitors to treat PV. Some patients may get transformed into acute
leukaemia. Any form of treatment during this phase is not at all satisfactory.

Currently, management of PV depends on the risk stratification

Age >60yrs or history


Cardiovascular risk factors
of thrombosis
Risk category

Low No No

Intermediate No Yes

High Yes

Phlebotomy is the cornerstone of low-risk patients aimed at reaching and maintaining a target hematocrit of
45% in males and 42% in females. Low dose aspirin may be added to the treatment. High-risk patients should
receive myelosuppressive treatment in addition to phlebotomy. The drug of choice is hydroxyurea.

PV may infrequently occur during childbearing years. There is an increased incidence of abortion in about
30% of cases. Pre-eclampsia is also common. It is very interesting that some of the women may even reduce
their hematocrit. Their phlebotomy requirement is also found to be decreased. The possible explanations are
the erythropoietic suppressive effect of the high estrogen levels, expansion of the plasma volume and
nutritional deficiencies. If needed, the patient should be treated with phlebotomy, low dose aspirin or
interferon. After delivery the blood count will drift back to the original polycythemic level.

facebook.com/HonorsGroup
READING SUB-TEST

Questions 1-7

For each question, 1-7, decide which text (A, B, C or D) the information comes from. You may use any letter
more than once.

In which text can you find information about;

1 Severe itching of the skin, as a symptom of various ailments.

Answer

2 Maintaining the ratio of the volume of red blood cells to the total volume of blood around five percent and 8
percent lesser than 50% in males and females.

Answer

3 Take place during pregnancy.

Answer

4 A condition arising due to shortening of the genes.

Answer

5 The presence of an abnormal excess of cells.

Answer

6 Bluish cast to the skin and mucous membrane.

facebook.com/HonorsGroup
READING SUB-TEST

Answer

7 An enzyme that can transfer a phosphate group from ATP to a protein in a cell.

Answer

Questions 8-14

Answer each of the questions, 8-14, with a word or short phrase from one of the texts. Each answer may
include words, numbers or both.

8 What is the term which refers to the use of the drug in the treatment of certain cancers?

Answer

9 What is found in a gene on chromosome 9p in patients?

Answer

10 When does a patient develop anaemia?

Answer

11 What is the condition in which bone marrow activity is decreased, resulting in fewer red blood cells, white
blood cells, and platelets?

Answer

12 What leads to the formation by peripheral blood mononuclear cells from patients with polycythemia vera?

facebook.com/HonorsGroup
READING SUB-TEST

Answer

13 What is the hormone produced by the kidney that promotes the formation of red blood cells by the bone
marrow?

Answer

14 What is the recommended for treatment?

Answer

Questions 15-20

Complete each of the sentences, 15-20, with a word or short phrase from one of the texts. Each answer may
include words, numbers or both.

15 is used with other medications or radiation therapy to treat some blood disorders.

16 mutations were found to have a high correlation with abnormal heart defects.

17 cells can undergo rapid proliferation before differentiating into maturation stages.

18 are seen in the great majority of cases of polycythaemia.

19 Polycythemia vera shows stable growth, majorly come into being from a single stem
cell.

20 In various cancerous conditions, production of is recorded to be independent of hypoxic


stimuli.

facebook.com/HonorsGroup
READING SUB-TEST

Part B

In this part of the test, there are six short extracts relating to the work of health professionals. For questions 1-
6, choose the answer (A, B or C), which you think fits best according to the text.

The American Academy of Pediatrics recommends screening for dyslipidemia in children and adolescents who
have a family history of dyslipidemia or premature CVD, those whose family history is unknown, and those
youths with risk factors for CVD, such as being overweight or obese, having hypertension or DM, or being a
smoker 1 In 2011, the NHBLI Expert Panel recommended universal dyslipidemia screening for all children
between 9 and 11 years of age and again between 17 and 21 years of age 23.

Analysis of data from NHANES 1999 to 2006 showed that the overall prevalence of abnormal lipid levels
among youths 12 to 19 years of age was 20 3%. From 2005 to 2010, among adults with high LDL- C, age-
adjusted control of LDL-C increased from 22 3% to 29 5% 25 The prevalence of LDL-C control was lowest
among people who reported receiving medical care less than twice in the previous year (11 7%), being
uninsured (13 5%), being Mexican American (20 3%), or having income below the poverty level (21 9%) 2.

facebook.com/HonorsGroup
READING SUB-TEST

DEBATABLE Concepts of Laparoscopic splenectomy

Malignant hematologic diseases

Huge splenomegaly (> 25 cm)

Malignant splenic tumor


Pericapsular inflammation

Large lymph nodes at the splenic hilum

PHT and cirrhosis

Difficulties
― Technical challenge
― Splenic mobilization
― Safe access to the splenic hilum
―Increased risk of bleeding ―Extraction ext

The AED protocol is consistent with the guidelines recommended by the American Heart Association
(Guidelines 2005 for Cardiopulmonary Resuscitation and Emergency Cardiac Care American Heart
Association; Circulation vol 112, Issue 24 Suppl. Dec. 13, 2005) and the International Liaison Committee on
Resuscitation (ILCOR)). Upon detecting a shockable cardiac rhythm, the AED advises the operator to press
the SHOCK button (9390E only) to deliver a defibrillation shock followed by performing 2 minutes of CPR.
For the Powerheart AED G3 Automatic, upon detecting a shockable rhythm, the AED will automatically
deliver defibrillation shocks followed by performing 2 minutes of CPR.

facebook.com/HonorsGroup
READING SUB-TEST

Note: In alignment with the 2005 Guidelines, the default setting for the CPR time has been set to allow for 5
cycles of 30 compressions and 2 breaths. Increasing or decreasing the CPR time setting may increase or
decrease the number of actual cycles allowed during the CPR timeout period.

The geometric mean level of triglycerides for American adults ≥20 years of age was 103 5 mg/ dL in
NHANES 2011 to 2014. Approximately 24 2% of adults had high triglyceride levels (≥150 mg/dL) in
NHANES 2011 to 2014. Among males, the age-adjusted geometric mean triglyceride level was 111 6 mg/dL
in NHANES 2011 to 2014, with the following racial/ethnic breakdown: — 113 2 mg/dL for non-Hispanic
white males — 86 7 mg/dL for non-Hispanic black males — 124 1 mg/dL for Hispanic males — 115 3 mg/dL
for non-Hispanic Asian males

Relationship of a liver index and its parameters to HCC aggressiveness

β Se(β) p. 95% C.I

(A)

Liver Index score 0.2462 0.0247 <0.001 0.1978 to 0.2945

(B)

GGTP (IU/ml) 0.0013 0.0003 <0.001 0.0007 to 0.0020

Total Bilirubin
0.0585 0.0140 <0.001 0.0311 to 0.0859
(mg/dl)

facebook.com/HonorsGroup
READING SUB-TEST

Albumin (g/dl) -0.3821 0.0554 <0.001 -0.4908 to -0.2733

Platelets (x109/L) 0.0031 0.0005 <0.001 0.0021 to 0.0041

Ultrasound is done during pregnancy to track the development of the fetus in the mother’s womb. It is not
only helpful in tracking down the development but also helps to find out any fetal anomalies. Ultrasound
reveals the heartbeat of the fetus, the radius of the head, the length of the hands and feet and also his/her
height and weight. There are various kinds of ultrasound which can be done during pregnancy namely
Transvaginal Ultrasound, 3-D Ultrasound, 4-D Ultrasound and Fetal Echocardiography. While the Sonography
reports in the first trimester provides information about the fetal heartbeat, it also examines the placenta,
uterus, ovaries, cervix, checks for multiple pregnancies, the sonography done in the second and third trimester
reveals much important criteria like placental abruption, placenta previa, characteristics of Down’s syndrome
if there are any possibilities. The ultrasound in this stage also determines whether the fetus is carrying any
form of congenital disease whether hereditary or non-hereditary or not.

facebook.com/HonorsGroup
READING SUB-TEST

1 As per the following notice, what is correct?

Dyslipidemia in children is common.

Young committee mostly gets affected by the disease.

Statistics showing the curtailment ratio of the affected people.

2 Notice on debatable concepts gives information about;

Concepts which may have direct impact on splenectomy.

Conditions which might arise after splenectomy.

Situations requiring great effort.

3 The manual gives information about

Rescue Protocol

Emergency Care Plan

Adjustable Properties of Powerheart AED G3

4 As per the given notice, GM levels of triglycerides is;

Higher among people who are 20+ years.

Common among men.

Common among women.

facebook.com/HonorsGroup
READING SUB-TEST

5 Pick the one that is incorrect

GGTP (cut-off):GGTP<100; 100 ≤ GGTP ≤ 200; GGTP>200; for scores 1, 2, 3 respectively.

Bilirubin (cut-off): Bil<1.5; 1.5 ≤ Bil ≤ 2.5; Bil>2.5; for scores 1, 2, 3 respectively.

Albumin (cut-off): Alb>3.5; 2.5 ≤ Alb < 3.5; Alb<2.5; for scores 1, 2, 3 respectively.

6 The given notice talks about

Functioning of the ultrasound.

Detection of fetus.

Examination of the fetus.

facebook.com/HonorsGroup
READING SUB-TEST

Part C

In this part of the test, there are two texts about different aspects of healthcare. For questions 7-22, choose the
answer (A, B, C or D) which you think fits best according to the text.

Text 1: Anaplasmosis

Anaplasmosis is a tick-borne disease caused by the bacterium anaplasma phagocytophilum. It was previously
known as human granulocytic ehrlichiosis (HGE) and has more recently been called human granulocytic
anaplasmosis (HGA). Anaplasmosis is transmitted to humans by tick bites, primarily from the black-legged
tick (Ixodes scapularis) and the western black-legged tick (Ixodes pacificus). Of the four distinct phases in the
tick life-cycle (egg, larva, nymph, adult), nymphal and adult ticks are most frequently associated with
transmission of anaplasmosis to humans. Typical symptoms include fever, headache, chills, and muscle aches.
Usually, these symptoms which occur within 1-2 weeks of a tick bite can’t be known and in many cases can’t
even be averted. Anaplasmosis, which often can’t be thwarted, is initially diagnosed based on symptoms and
clinical presentation, and later confirmed by the use of specialized laboratory tests. The first line treatment for
adults and children of all ages is doxycycline. Anaplasmosis and other tick-borne diseases can be obviated.

Anaplasmosis is caused by the bacterium anaplasma phagocytophilum. This organism used to be known by
other names, including Ehrlichia equi and Ehrlichia phagocytophilum, and the disease caused by this organism
has been previously described as human granulocytic ehrlichiosis (HGE). However, a taxonomic change in
2001 identified that this organism belonged to the genus anaplasma, and resulted in a change in the name of
the disease to anaplasmosis. Anaplasmosis was first recognized as a disease of humans in the United States in
the mid-1990’s, but did not become a reportable disease until 1999. The number of anaplasmosis cases
reported has increased steadily since the disease became reportable, from 348 cases in 2000, to 1761 cases in
2010. The incidence (the number of cases for every million persons) of anaplasmosis has also increased, from
1.4 cases per million persons in 2000 to 6.1 cases per million persons in 2010. The case fatality rate (i.e. the
proportion of anaplasmosis patients that reportedly died as a result of infection) has remained low, at less than
1%.

facebook.com/HonorsGroup
READING SUB-TEST

The bacterium anaplasma phagocytophilum is transmitted to humans by the bite of an infected tick. The black-
legged tick (Ixodes scapularis) is the vector of A. phagocytophilum in the northeast and upper midwestern
United States. The western black-legged tick (Ixodes pacificus) is the primary vector in Northern California.
The first symptoms of anaplasmosis typically begin within 1-2 weeks after the bite of an infected tick. A tick
bite is usually painless, and some patients who develop anaplasmosis do not remember being bitten. It can be a
serious illness that can be fatal if not treated correctly, even in previously healthy people.

The severity of anaplasmosis may depend in part on the immune status of the patient. Persons with
compromised immunity caused by immunosuppressive therapies (e.g., corticosteroids, cancer chemotherapy,
or long-term immunosuppressive therapy following an organ transplant), HIV infection, or splenectomy appear
to develop a more severe disease, and case-fatality rates for these individuals are characteristically higher than
case-fatality rates reported for the general population.

Because A. phagocytophilum infects the white blood cells and circulates in the bloodstream, this pathogen
may pose a risk to be transmitted through blood transfusions. Anaplasma phagocytophilum has been shown to
survive for more than a week in refrigerated blood. Several cases of anaplasmosis have been reported
associated with the transfusion of packed red blood cells donated from asymptomatic or acutely infected
donors. Patients who develop anaplasmosis within a month of receiving a blood transfusion or solid organ
transplant should be reported to state health officials for prompt investigation.

There are several aspects of anaplasmosis that make it challenging for healthcare providers to diagnose and
treat. The symptoms vary from patient to patient and can be difficult to distinguish from other diseases.
Treatment is more likely to be effective if started early in the course of the disease. Diagnostic tests based on
the detection of antibodies will frequently appear negative in the first 7-10 days of illness. For this reason,
healthcare providers must use their judgment to treat patients based on clinical suspicion alone. Healthcare
providers may find important information in the patient’s history and physical examination that may aid
clinical diagnosis. Information such as recent tick bites, exposure to areas where ticks are likely to be found, or
history of recent travel to areas where anaplasmosis is endemic can be helpful in making the diagnosis.

facebook.com/HonorsGroup
READING SUB-TEST

Text 2: Candidiasis

Candidiasis is a fungal infection caused by yeasts that belong to the genus Candida. There are over 20 species
of Candida yeasts that can cause infection in humans, the most common of which is Candida albicans. Candida
yeasts normally reside in the intestinal tract and can be found on mucous membranes and skin without causing
infection; however, overgrowth of these organisms can cause symptoms to develop. Symptoms of candidiasis
vary depending on the area of the body that is infected.

Candidiasis that develops in the mouth or throat is called “thrush” or oropharyngeal candidiasis. Candidiasis in
the vagina is commonly referred to as a “yeast infection.” Invasive candidiasis occurs when Candida species
enter the bloodstream and spread throughout the body. The infection is not very common in the general
population. It is estimated that between 5% and 7% of babies less than one month old will develop oral
candidiasis. The prevalence of oral candidiasis among AIDS patients, (particularly women rather than men,
although not yet an established fact) is estimated to be between 9% and 31%, and studies have documented
clinical evidence of oral candidiasis in nearly 20% of cancer patients. Candidiasis of the mouth and throat, also
known as “thrush" or oropharyngeal candidiasis, is a fungal infection that occurs when there is an overgrowth
of a yeast called Candida. Candida yeasts normally live on the skin or mucous membranes in small amounts.
However, if the environment inside the mouth or throat becomes imbalanced, the yeasts can multiply and
cause symptoms. Candida overgrowth can also develop in the oesophagus, and this is called Candida
esophagitis, or esophageal candidiasis.

Candida infections of the mouth and throat can manifest in a variety of ways. The most common symptom of
oral thrush is white patches or plaques on the tongue and other oral mucous membranes. Other symptoms
include redness or soreness in the affected areas; difficulty swallowing; cracking at the corners of the mouth
(angular cheilitis) etc.

Candida infections of the mouth and throat are infrequent among adults who are otherwise healthy. Oral thrush
presents itself most recurrently among babies less than one month old, the elderly, and groups of people with
weakened immune systems. Other factors associated with oral and esophageal candidiasis include HIV/AIDS,
cancer treatments, organ transplantation, diabetes etc. Good oral hygiene practices may sporadically help to
prevent oral thrush in people with weakened immune systems. Some studies have shown that chlorhexidine

facebook.com/HonorsGroup
READING SUB-TEST

(CHX) mouthwash can help to prevent oral candidiasis in people undergoing cancer treatment. People who use
inhaled corticosteroids may be able to reduce the risk of developing thrush by washing out the mouth with
water or mouthwash after using an inhaler.

Candida infections of the mouth and throat must be treated with prescription antifungal medication. The type
and duration of treatment depends on the severity of the infection and patient-specific factors such as age and
immune status. Untreated infections can lead to a more serious form of invasive candidiasis. Oral candidiasis
usually responds to topical treatments such as clotrimazole troches and nystatin suspension (nystatin “swish
and swallow”). Systemic antifungal medication such as fluconazole or itraconazole may be necessary for
oropharyngeal infections that do not respond to these treatments. Candida esophagitis is typically treated with
oral or intravenous fluconazole or oral itraconazole. For severe or azole-resistant esophageal candidiasis,
treatment with amphotericin B may be necessary. For healthcare providers: the most up-to-date clinical
practice guidelines for the treatment of oropharyngeal / esophageal candidiasis are available at the Infectious
Diseases Society of America.

facebook.com/HonorsGroup
READING SUB-TEST

Text 1: Questions 7-14

7 According to paragraph 1, what is not anaplasmosis?

A bacterial disease

A disease that is transmitted by tick bites

A disease in which people suffer from muscle pain

A disease that can’t be prevented

8 Which word in paragraph 1 may mean ‘removing difficulty’?

Averted

Thwarted

Obviated

None of the above.

9 Paragraph 2 deals more with the;

Taxonomy of anaplasmosis.

Classification of the anaplasmosis.

How anaplasmosis entered the US.

Annual cases of anaplasmosis in the US.

10 What is not true about anaplasmosis, according to paragraph 2?

The old name of anaplasmosis was HGE.

facebook.com/HonorsGroup
READING SUB-TEST

The causal agent of anaplasmosis was recorded to be ehrlichia equi

HGE was renamed as anaplasmosis in the year 2001

Cases of anaplasmosis became known only after 2000

11 What is not true about ticks, according to paragraph 3?

A tick bite is painless

A tick bite carries bacterium anaplasma phagocytophilum

Ticks are present throughout the US

Black-legged ticks are present across California

12 What is the central idea of paragraph 4?

Immune-compromised individuals

The effects of anaplasmosis

Fatality rate and anaplasmosis

None of the above

13 What do we find in paragraph 5?

Blood transfusion risks

Organ transplant risks

Blood transfusion and organ transplant risks associated with anaplasma species

Information about anaplasma phagocytophilum

facebook.com/HonorsGroup
READING SUB-TEST

14 According to paragraph 6, what is not true about anaplasmosis?

It is difficult to diagnose and treat anaplasmosis

Why tests for anaplasmosis appear negative

Different patients may show different symptoms

A patients medical history is often taken into consideration

Text 2: Questions 15-22

15 According to paragraph 1, the abode for candida yeasts is;

Intestinal tract

Mucous membrane

Skin

All of the above

16 According to paragraph 2, oral candidiasis is;

Prevalent among children.

Prevalent among babies.

Common among people suffering from AIDS.

Common among female patients with AIDS.

17 According to paragraph 2, candida yeasts;

grow on the mucous membrane.

facebook.com/HonorsGroup
READING SUB-TEST

grow under the skin of the mouth.

grow only inside the throat area.

grow expeditiously in the esophagus.

18 The word ‘manifest’ in paragraph 3 may mean;

To describe

To multiply

To show

To disguise

19 What does paragraph 4 indicate?

Who gets affected with oral candidiasis?

How to prevent oral candidiasis.

Risk and prevention.

None of the above.

20 According to paragraph 4, candida infections;

Occur among people affected with AIDS, diabetes etc.

Occur less common among people with a low immunity rate.

Can be prevented using off-the-shelf medical products such as CHX mouthwash.

Less prevalent among adults.

facebook.com/HonorsGroup
READING SUB-TEST

21 The word ‘sporadic’ in paragraph 4 means;

Always

Not regular

Intermittently

Every so often

22 What is the central idea presented in paragraph 5?

The treatment process of candidiasis.

Treatment and outcome.

Medication for candidiasis.

None of the above.

facebook.com/HonorsGroup
READING SUB-TEST

ANSWERS
Part A

1. Correct Answer Is: C


2. Correct Answer Is: D
3. Correct Answer Is: D
4. Correct Answer Is: A
5. Correct Answer Is: C
6. Correct Answer Is: A
7. Correct Answer Is: B
8. Correct Answer Is: thalidomide
9. Correct Answer Is: somatic mutation
10. Correct Answer Is: phlebotomy requirement decreases
11. Correct Answer Is: Myelosuppression
12. Correct Answer Is: Endogenous erythroid colony
13. Correct Answer Is: Erythropoietin (EPO)
14. Correct Answer Is: phlebotomy
15. Correct Answer Is: Hydroxyurea (hydrea)
16. Correct Answer Is: Jak - 2 Kinase
17. Correct Answer Is: Myeloid Progenitor
18. Correct Answer Is: Endogenous Erythroid Colonies (eec)
19. Correct Answer Is: Hematopoietic
20. Correct Answer Is: Erythropoietin

Part B & C

1. Correct Answer Is: Statistics showing the curtailment ratio of the affected people.
2. Correct Answer Is: Conditions which might arise after splenectomy.
3. Correct Answer Is: Rescue Protocol
4. Correct Answer Is: Higher among people who are 20+ years.
5. Correct Answer Is: Albumin (cut-off): Alb>3.5; 2.5 ≤ Alb < 3.5; Alb<2.5; for scores 1, 2, 3
respectively.
6. Correct Answer Is: Functioning of the ultrasound.
7. Correct Answer Is: A disease that can’t be prevented
8. Correct Answer Is: Obviated
9. Correct Answer Is: Annual cases of anaplasmosis in the US.
10. Correct Answer Is: Cases of anaplasmosis became known only after 2000
11. Correct Answer Is: Ticks are present throughout the US
12. Correct Answer Is: Immune-compromised individuals
13. Correct Answer Is: Blood transfusion and organ transplant risks associated with anaplasma species
14. Correct Answer Is: Why tests for anaplasmosis appear negative
15. Correct Answer Is: All of the above
16. Correct Answer Is: Common among people suffering from AIDS.
17. Correct Answer Is: grow expeditiously in the esophagus.
18. Correct Answer Is: To show
19. Correct Answer Is: Risk and prevention.
20. Correct Answer Is: Less prevalent among adults.
21. Correct Answer Is: Not regular
22. Correct Answer Is: Treatment and outcome.

facebook.com/HonorsGroup
READING SUB-TEST

OET 2.0 READING TASK 15

Part A

TIME: 15 minutes

Look at the four texts, A-D, in the separate Text Booklet.

For each question, 1-20, look through the texts, A-D, to find the relevant information.

Write your answers on the spaces provided in this Question Paper.

Answer all the questions within the 15-minute time limit.

Your answers should be correctly spelt.

Text A

Cytokines

Cytokines are messengers secreted by various cell types in the body in response to a wide variety of
physiological stimulus. Cytokines aid in normal physiological processes such as growth, differentiation,
hematopoiesis, as well as several inflammatory and immune responses. The cytokines as present in minimal
levels in hepatic circulation during normal physiological status and are necessary for hepatic homeostasis.
However, the cytokines have been observed to play an active role in mediating the inflammatory progression
of NAFLD as characterized by apoptotic and necrotic lesion in liver leading to fibrosis. The cytokines
involved in hepatic inflammation, are categorized under several subfamilies – Tumor necrosis factor-α (TNF-
α), Tumor growth factor β (TGF-β), Interleukins and chemokines.

facebook.com/HonorsGroup
READING SUB-TEST

Text B

Tumor Necrosis Factor α

TNF-α is a pro-inflammatory cytokine secreted by several cell types such as neutrophils, macrophages, T and
B lymphocytes, endothelial cells, mast cells, fibroblasts etc. In liver, hepatocytes and Kupffer cells are the
major contributors of TNF- α. TNF-α plays a central role in the initiation of inflammatory cascade and its
progression from steatosis to steatohepatitis. Experiments with mice models for obesity have shown the
importance of TNF-α in NAFLD, where anti- TNF-α drug therapy showed promising results. Increased free
fatty acid level in obesity stimulates the hepatocytes to secrete TNF-α, which elicits the free fatty acid induced
expression of inflammatory genes. A positive correlation has been observed between TNF-α level in serum
and degree of fibrosis in patients with NAFLD. A study with pentoxifylline - an inhibitor of TNF-α has shown
suppressive effect on elevation of serum transaminases and triglycerides in experimental NAFLD induced rats.
The study also demonstrated that NAFLD induced TNF-α expression stimulates endoplasmic reticulum stress,
which further mediate the progression of steatosis to fibrosis. c-Jun N-terminal Kinase 1 (JNK1) a stress
activated protein kinase is activated by TNF-α which leads to initiation of an autocrine/paracrine loop resulting
in enhanced TNF- α production in liver. TNF-α activates inhibitory kappa b kinase β (IKKβ) which
phosphorylates IKB resulting in the translocation of NFκB into nucleus.

facebook.com/HonorsGroup
READING SUB-TEST

Text C

Transforming growth factor β

TGF-β is a cytokine secreted by hepatocytes and Kupffer cells in response to degradation changes in liver.
TGF-β activates the resting stellate cells by transforming them into active myofibroblasts, which secrete
extracellular matrix protein to initiate the fibrosis process. Earlier studies have shown up regulated expression
of TGF-β following experimentally induced hepatic damage such as in CCl4 poisoning. Elevated TGF-β
mRNA expression levels have been found in patients with liver fibrosis. Earlier studies have shown that TGF-
β is an early marker for the progression of steatohepatitis. Hence, detection of TGF-β level would be helpful in
marking NASH stage of NAFLD. Studies have observed polymorphisms in TGF-β1 gene in obese NAFLD
patients with advanced stages of hepatic fibrosis. Xiao and Ho have recently reported that administration of
Epigallocatechin gallate (EGCG) reduced hepatic severity in NAFLD by suppression of TGF/SMAD pathway.
Studies with TGF-β receptor II deficient mice showed protective effect against experimentally induced
NAFLD with methionine and choline deficient (MCD) diet, which was mediated through smad2 activation.

Text D

Interleukins

Interleukin-6 is a proinflammatory cytokine which has been implicated in metabolic syndrome. IL-6 also plays
several other functions such as inducer of immune response, hematopoiesis and oncogenesis. Certain studies
have reported IL-6 to be antihepatotoxic and mediate hepatic regeneration after partial hepatectomy in mice.
IL-6 has also been considered to reduce hepatic oxidative stress and to curtail mitochondrial dysfunction.
However, studies have reported the role of IL-6 as an acute phase inflammatory mediator leading to secretion
of inflammatory serum proteins. Hence, the possibility of its role in pathogenesis of NAFLD cannot be
ignored. A positive correlation has been found in patients with NASH and circulating IL-6 level. Studies with
IL-6 knockout mice models have shown reduced severity when subjected to experimental NAFLD.
Yamaguchi et al, has shown that inhibition of IL-6 receptor with Tocilizumab, enhanced hepatic steatosis but

facebook.com/HonorsGroup
READING SUB-TEST

protected against extensive hepatic damage in MCD diet induced NASH. Recently, Hamirani et al, have
observed a positive correlation between C reactive protein (CRP) and IL-6 levels and increased liver fat
accumulation in NAFLD patients as verified with CT-scans.

Questions 1-7

For each question, 1-7, decide which text (A, B, C or D) the information comes from. You may use any letter
more than once.

In which text can you find information about;

1 Biological cell that synthesizes the extracellular matrix and collagen.

Answer

2 Arranged in a radiating pattern like that of a star.

Answer

3 Production or formation, development, and differentiation of blood cells.

Answer

4 Evokes a specific functional reaction in an organ or tissue.

facebook.com/HonorsGroup
READING SUB-TEST

Answer

5 Hepatoprotection in most of the cases d.

Answer

6 Impairment of the normal processes of synthesis and elimination of triglyceride fat.

Answer

7 The tendency towards a relatively stable equilibrium between interdependent elements, especially as
maintained by physiological processes.

Answer

Questions 8-14

Answer each of the questions, 8-14, with a word or short phrase from one of the texts. Each answer may
include words, numbers or both.

8 What plays an active role in mediating the inflammatory progression of NAFLD

Answer

9 What is the term used to indicate the thickening and scarring of connective tissue?

Answer

10 What does the TNF-α level in serum indicate in patients with NAFLD?

facebook.com/HonorsGroup
READING SUB-TEST

Answer

11 What reduced the hepatic severity in NAFLD patients?

Answer

12 What does IL-6 secrete, as an acute phase inflammatory mediator?

Answer

13 What is known to be enhancing or contributing more to TNF- α?

Answer

14 What is the terms used to show the derivative, which decreases the viscosity of blood?

Answer

facebook.com/HonorsGroup
READING SUB-TEST

Questions 15-20

Complete each of the sentences, 15-20, with a word or short phrase from one of the texts. Each answer may
include words, numbers or both.

15 relates to a hormone which has effect only in the vicinity of the gland secreting it.

16 It can be said that disturbances in the normal functioning of the ER often lead to
cell response.

17 also plays is reported to initiate immune response.

18 delivered results as expected

19 is characterized by inflammation of the liver with concurrent fat accumulation in liver.

20 denotes a cell-produced substance that has an effect on the cell by which it is secreted.

facebook.com/HonorsGroup
READING SUB-TEST

Part B

In this part of the test, there are six short extracts relating to the work of health professionals. For questions 1-
6, choose the answer (A, B or C), which you think fits best according to the text.

Variety of challenges by Obstetricians and Gynecologists

The biggest challenge lies in managing the care of patients of all ages from puberty through menopause and
beyond; treating both acute and chronic health conditions; and of course, managing risk through the pregnancy
and childbirth process. The United States has a higher ratio of maternal deaths than at least 40 other countries,
even though it spends more money per capita on maternity care than any other. The lack of a comprehensive,
confidential system of ascertainment of maternal death designed to record and analyse every maternal death
continues to subject U.S. women to the unnecessary risk of preventable mortality. Maternal deaths must be
reviewed to make motherhood safer.

The status of maternal deaths in the United States was part of a larger report on the global, regional, and
national levels and causes of maternal mortality from 1990-2015. The findings suggest that only 16 countries
will achieve a target of a 75% reduction in the maternal mortality ratio (or number of maternal deaths per
100,000 live births) by 2017.

facebook.com/HonorsGroup
READING SUB-TEST

Case Study

The mother of the other patient was a white 30-year-old housewife. This was her second pregnancy, it was
planned and there had been an ovarian infection during prenatal and she had been carrying twins, but one fetus
had died. Delivery was vaginal, birth weight was 1,260 g, gestational age was 30 weeks, SNAPE-PE was 0
and sex was male. The child presented petechiae at birth and persistent tachypnea for several days. At 1 year
and 6 months he was still excreting CMV in urine, had normal neurological development for his corrected age
and his sight and hearing examination findings were normal.

Complications

Some complications are common to every expecting mother while others can be specific to limited expecting
females only. According to the statistics, it is found that 90% of the females in some or the other stages of
pregnancy have knowledgeable complications. While some difficulties are easy on the body like mild nausea
and morning sickness but other complications like asthma, diabetes, thyroid diseases and hypertension
necessities to be taken care of under the excellent guidance of gynecologists and obstetrics. Some of the
medical complications develop to be fatal to the fetus as well as the mother. Women suffering from other
medical problems like HIV, Urinary Tract Infections, Ectopic Pregnancies and it’s should go for immunization
and medical assistance from time to time to keep the intensity of the complications under control. Some of the
medical complications like Diabetes and Hypertension are so chronic that continue to persist in the body even
after the liberation of the baby. Shunning smoking, drinking alcohol and maintaining a healthy lifestyle
decreases the chances of having complications in pregnancy.

facebook.com/HonorsGroup
READING SUB-TEST

ELECTIVE SPLENECTOMY

ITP : 57 %

Congenital spherocytosis : 12 %

Hemolytic anemia : 10 %

Hodgkin’s disease : 5 %

AIDS related thrombocytopenia : 3 %

Lymphoma : 3 %

Leukemia: 2.5 %

Others (sarcomas, splenic metastases,…)

2. SPLENECTOMY IN EMERGENCY:

Trauma

facebook.com/HonorsGroup
READING SUB-TEST

Screening For NAFLD

A recent, cost-effective analysis using a Markov model suggested that screening for NASH in individuals with
diabetes is not cost-effective at present, because of disutility associated with available treatment. Given that
liver biochemistries can be normal in patients with NAFLD, they may not be sufficiently sensitive to serve as
screening tests, whereas liver ultrasound or TE are potentially more sensitive, but their utility as screening
tools is unproven. Some experts recently have called for “vigilance” for chronic liver disease (CLD) in patients
with type 2 diabetes, but not routine screening.

Liver index and its relationship to patient survival

Score

1 2 3

GGTP (IU/ml) <100 100 - 200 >200

Total Bilirubin (mg/dl) <1.5 1.5 - 2.5 >2.5

Albumin (g/dl) >3.5 2.5 - 3.5 <2.5

Platelets (x109/l) <100 100 - 150 >150

facebook.com/HonorsGroup
READING SUB-TEST

Questions 7-6

1 The given notice talks about;

US Death Rate with respect to maternity cases.

Scope and importance.

How to deal with preventable mortality?

2 As per the case study, pick the right statement;

The baby was born with the infection.

The infection surfaced itself when the child turns 1 year 6 months.

The child was normal at the age of 1 year and 6 months.

3 Pick the right statement;

Improper care during pregnancy can lead to complications.

Continous examination is necessary to thwart complications.

Major complications will always lead to death.

4 The notice gives information about;

Indications of splenectomy.

Percentage of disease conditions.

Various conditions, which arise due to Splenectomy.

facebook.com/HonorsGroup
READING SUB-TEST

5 According to Markov model:

Screening for NASH in Primary Care shall be simple.

Screening for NASH in Diabetic Care or Clinics shall be simple.

Use of ultrasound is not fully recommended.

6 Pick the incorrect statement;

Bil (cut-off): Bil<1.5; 1.5 ≤ Bil ≤ 2.5; Bil>2.5; scores 1, 2, 3 respectively.

Alb (cut-off):Alb>3.5; 2.5 ≤ Alb ≤ 3.5; Alb<2.5; scores 1, 2, 3 respectively.

Platelets (cut-off):Plt<100; 100 ≤ Plt ≤ 150; Plt ≥150; scores 1, 2, 3 respectively

facebook.com/HonorsGroup
READING SUB-TEST

Part C

In this part of the test, there are two texts about different aspects of healthcare. For questions 7-22, choose the
answer (A, B, C or D) which you think fits best according to the text.

Text 1: Brain Imaging Technologies

Brain imaging has greatly advanced in the last 20 years, due to a better understanding of the electromagnetic
spectrum and radiofrequency waves, in relation to protons in individual molecules within the cells of the brain.
New technologies allow non-invasive spatial mapping, (morphology), and observations of processes within the
brain during set tasks. By sequencing scanned sections of the brain, activity between neurons in different parts
of the brain can be observed and monitored. More recent technologies using a higher frequency resolution can
identify the distribution of individual metabolites (large complex molecules), and pharmaceutical drugs. There
are a number of scanning techniques, their purpose and limitations are described below.

A computerised tomography (CT) scans use X-rays to show the structure of the brain, with details such as
blood perfusion, (plates a and b); the resultant images are two dimensional and of comparatively low
resolution, however, the quality has been much improved since 1998. With improved technology, the single
section has now become as multisection and the speed has increased eight times, giving well-defined 3-D
pictures. A CT scan may reveal underdeveloped parts of the brain or sites of injury from impact, tumours,
lesions or infection. Before a CT scan, the patient may drink but is asked not to eat for four hours beforehand,
and not to take strenuous exercise. A CT brain scan, the preferable scanning method by doctors, will take about
30 minutes and the patient must lie still for the duration.

An MRI scanner uses a strong magnetic field and radio waves to create pictures of the tissues and other
structures inside the brain, on a computer. The magnetic field aligns the protons (positively charged particles)
in hydrogen atoms, like tiny magnets. Short bursts of radio waves are then sent to knock the protons out of
position, and as they realign, (relaxation time), they emit radio signals which are detected by a receiving device
in the scanner. The signals emitted from different tissues vary, and can, therefore, be distinguished in the
computer picture. An MRI scanner can create clear detailed pictures of the structure of the brain and detect any

facebook.com/HonorsGroup
READING SUB-TEST

abnormalities or tumours. Sometimes a dye, or tracer, such as gadolinium may be introduced via a vein in the
arm, to improve contrast in the image. Images can be enhanced by differences in the strength of the nuclear
magnetic resonance signal recovered from different locations in the brain.

Functional magnetic resonance imaging (fMRI) can show which part of the brain is active, or functioning, in
response to the patient performing a given task, by recording the movement of blood flow. All atoms and
molecules have magnetic resonance, emitting tiny radio wave signals with movement, because they contain
protons. Different molecules have different magnetic resonance and two components of blood are tracked to
observe brain activity. Haemoglobin in the blood carries oxygen; oxyhaemoglobin, around the brain and when
it is used up, it becomes deoxyhaemoglobin. Where the oxygen is being ‘used up’ shows the site of activity in
the brain. The picture is made by monitoring the ratio of the tiny wave frequencies between these two states
whilst the patient carries out a task, e.g. tapping a finger, which highlights the area of the brain functioning to
carry out this task.

Positron emission tomography (PET) scanning produces a three-dimensional image of functional processes in
the brain, (not just the structure). PET is a nuclear medicine imaging technique, which requires the patient to
receive a small injection of radioactive material (a sugar tracer; fluorodeoxyglucose), into the bloodstream.
The radioactive material causes the production of gamma-rays, these are a form of electromagnetic radiation
like X-rays, but of higher energy. The radioactive material is transported around the body and into the brain. A
ring of detectors outside the head is used to detect pairs of gamma rays emitted indirectly by the positron-
emitting radionuclide (tracer), in each part of the brain under examination.

The single photon emission computed tomography records the signals from gamma rays, (singly, rather than
when the emissions are opposite at 180º), using two or more synchronised gamma cameras, and the multiple 2-
D images are computed, tomographically reconstructed, to 3-D. A section may be examined from several
angles, but is slightly less clear than a PET image. A SPECT scanner is less expensive than a PET scanner and
uses longer-lived, more easily obtained radioisotopes.

facebook.com/HonorsGroup
READING SUB-TEST

Text 2: A safer way to detect heart disease

Researchers have used a specialized type of MRI to detect 88% of cases of coronary artery disease in a group
of patients with chest pain. The results suggest that the imaging technique can detect heart disease as
accurately as conventional methods, but with much less risk. Coronary artery disease is the most common form
of heart disease and the leading cause of death in the United States. It occurs when fat and calcium accumulate
in the arteries that supply blood to the heart. Over time, less blood reaches the heart and heart muscle dies. If
the plaque blocks the arteries completely, a heart attack occurs.

Currently, the best way to detect the disease is through a coronary angiography. A physician threads a tube into
the heart, releases a dye, and uses X-ray images to look for decreased blood flow. But there’s a small risk in
this procedure that the tube will pierce an artery, resulting in bleeding, or else scrape plaque from artery walls,
which, once the chunks of plaque are in the bloodstream, can lead to a heart attack or stroke. Other,
noninvasive tests such as cardiac ultrasounds are less risky, but not as accurate. Ultrasound images can be poor
in patients with other conditions such as obesity, requiring doctors to resort to invasive tests.

While an MRI allows doctors to image the body using magnets and radio waves, until recently it could not
produce clear images of dynamically voyaging objects, such as a beating heart. In the past two years, though,
stronger magnets, more powerful computers, and new software have improved MRI scanning. “Recent
developments allow us to acquire images of the heart in motion,” says Ricardo Cury, director of clinical
cardiac MRI at Massachusetts General Hospital in Boston and leader of the study. Doctors can now watch the
heart beating in real-time and the images are now sharp. “It’s like opening up the heart and looking at it
directly,” says Renato Santos, a cardiologist at Wake Forest University Baptist Medical Center. “Until
recently, MRI was a research tool,” says Santos. “Now it’s really a clinical tool…ready for prime time.”

Cury combined two cardiac MRI tests to improve the technique’s ability to diagnose coronary artery disease.
In his study, published in the July issue of Radiology, researchers at MGH, Harvard Medical School, and
Beneficencia Portuguesa Hospital in Sao Paulo, Brazil, examined 46 patients. They began with an MRI stress
test, injecting a harmless dye and medicine that stresses the heart. As the heart pumped, they used MRI to look

facebook.com/HonorsGroup
READING SUB-TEST

for decreased blood flow or evidence that the heart was working abnormally. Next, they examined still MRI
images of the heart for damaged areas or evidence of prior heart attacks. If patients were abnormal in one or
both tests, the doctors deduced blocked arteries.

Cury’s results reinforce those of an earlier, unrelated study in which researchers at Duke University used the
same techniques to successfully diagnose coronary artery disease in 100 patients. The results are good news for
patients. The MRI exam is short and painless. By using it to triage people with chest pain but actually without
any disease, physicians might save such patients from unnecessary invasive procedures. In cases where the
heart disease is evident, MRI can help doctors decide what to do next, for instance, whether surgery to clear or
bypass a blocked artery is necessary. After surgery, doctors can use MRI to monitor arteries for future
blockages noninvasively. Cury says that the 12 percent of cases misdiagnosed in his study are less than other
noninvasive tests, and in certain cases artifacts of the study’s design. He adds that MRI’s accuracy will
increase as doctors learn to make better diagnoses from MRI images. “Obviously 100% is ideal,” says Santos.
“I think MRI is going to get us closer to that than our traditional methods.”

Text 1: Questions 7-14

7 According to paragraph 1, technology;

Has made it easier to detect what is going on in a person’s brain

Helps define how neurons act and multiply

Makes the mapping of the brain a very simple task

Can help identify how big complex parts are distributed

8 According to paragraph 2, what is true about CT?

It works on the principles of X-rays

It provides two-dimensional images

facebook.com/HonorsGroup
READING SUB-TEST

It can help identify which parts of the brain are developed and which are not

It will always just take half an hour

9 The word ‘itty-bitty’ in paragraph 3 may means;

Effective

Gigantic

Small

Strong

10 According to paragraph 3, what is not true about an MRI?

During the scan, the tissues produce different signals and thus easily get identified.

Dye may be used to improve the quality of the image produced by the scan.

Protons emit radio signals.

The receiving device collects the emitted protons.

11 According to paragraph 4, a fMRI scan;

Produces more efficient images of the brain than an MRI.

Can detect which parts of the brain are active during movement.

Obtains the picture by closely analysing the frequencies of the emitted waves.

Not given

12 According to paragraph 5, a PET is;

An advanced technique like an MRI.

facebook.com/HonorsGroup
READING SUB-TEST

A non-magnetic technique for the detection of brain functioning.

A technique in which a patient is given a radioactive material injection.

Much more powerful than other techniques.

13 According to paragraph 6, what is true about a SPECT?

It produces quality images like an MRI

It is similar to a PET in imaging

It relies on gamma rays for detection

Not given

14 According to the information provided, which technique is considered the best of all by
doctors?

MRI

fMRI

PET

SPECT

Text 2: Questions 15-22

15 Paragraph 1 talks more about;

A breakthrough in imaging.

How heart diseases can be detected.

How new techniques are more efficient than traditional techniques.

How a heart attack occurs.

facebook.com/HonorsGroup
READING SUB-TEST

16 According to paragraph 1, a simple cause of a heart attack is;

Heart doesn’t receive enough blood.

Fat gets deposited in arteries.

Arteries carry less blood to the heart.

All of these

17 What risk is mentioned in paragraph 2?

The tube may puncture the artery.

The tube can cut off plaque.

The artery walls may release more plaque into the bloodstream.

All of these

18 Paragraph 2 talks more specifically about;

Techniques that are risky.

Why conventional techniques should not be used.

How expensive techniques are.

How risky CA can be.

19 According to paragraph 3, what is true about MRI scanning?

MRI scans in previous years were less effective.

It was not possible to capture motion-based images.

MRI scans of the new generation are more effective with power imaging qualities.

facebook.com/HonorsGroup
READING SUB-TEST

use of powerful computers and software have made MRI scanning more effective.

20 According to paragraph 3, what is possible with MRI now?

It is easy to see the heart, live in action

It is easy to track heart beating

It is easy to get more quality-based images

All of these

21 According to paragraph 4, which one of the following statements is true?

Cury led a team of doctors and researchers at MGH.

Cury detected blocked arteries.

The team of doctors compared 46 case studies.

Researchers began with the stress testing.

22 In paragraph 5, the word “triage” may mean;

To help

To examine

To show

To provide comfort

facebook.com/HonorsGroup
READING SUB-TEST

ANSWERS
Part A

1. Correct Answer Is: B


2. Correct Answer Is: C
3. Correct Answer Is: A
4. Correct Answer Is: A
5. Correct Answer Is: D
6. Correct Answer Is: D
7. Correct Answer Is: A
8. Correct Answer Is: cytokines
9. Correct Answer Is: fibrosis
10. Correct Answer Is: degree of fibrosis
11. Correct Answer Is: Epigallocatechin gallate
12. Correct Answer Is: inflammatory serum proteins
13. Correct Answer Is: hepatocytes and Kupffer cells
14. Correct Answer Is: Pentoxifylline
15. Correct Answer Is: Paracrine
16. Correct Answer Is: Stress
17. Correct Answer Is: Il-6
18. Correct Answer Is: Tnf-α “drug Therapy
19. Correct Answer Is: Steatohepatitis
20. Correct Answer Is: Autocrine

Part B & C

1. Correct Answer Is: Scope and importance.


2. Correct Answer Is: The baby was born with the infection.
3. Correct Answer Is: Continous examination is necessary to thwart complications.
4. Correct Answer Is: Indications of splenectomy.
5. Correct Answer Is: Screening for NASH in Diabetic Care or Clinics shall be simple.
6. Correct Answer Is: Platelets (cut-off):Plt<100; 100 ≤ Plt ≤ 150; Plt ≥150; scores 1, 2, 3
respectively
7. Correct Answer Is: Can help identify how big complex parts are distributed
8. Correct Answer Is: It can help identify which parts of the brain are developed and which are not
9. Correct Answer Is: Small
10. Correct Answer Is: The receiving device collects the emitted protons.
11. Correct Answer Is: Can detect which parts of the brain are active during movement.
12. Correct Answer Is: A technique in which a patient is given a radioactive material injection.
13. Correct Answer Is: It relies on gamma rays for detection
14. Correct Answer Is: MRI
15. Correct Answer Is: A breakthrough in imaging.
16. Correct Answer Is: All of these
17. Correct Answer Is: The tube may puncture the artery.
18. Correct Answer Is: Techniques that are risky.
19. Correct Answer Is: It was not possible to capture motion-based images.
20. Correct Answer Is: It is easy to get more quality-based images
21. Correct Answer Is: Researchers began with the stress testing.
22. Correct Answer Is: To examine

facebook.com/HonorsGroup

You might also like